0% found this document useful (0 votes)
29 views

Math 2

The document is a textbook on higher mathematics for engineers that covers various topics including indefinite integrals. The first lecture defines an indefinite integral as the collection of all antiderivatives of a given function. It discusses properties of indefinite integrals such as adding constants and changing variables. Common integrals are also presented, such as integrals of polynomials, trigonometric functions, and exponential functions.
Copyright
© © All Rights Reserved
Available Formats
Download as PDF, TXT or read online on Scribd
0% found this document useful (0 votes)
29 views

Math 2

The document is a textbook on higher mathematics for engineers that covers various topics including indefinite integrals. The first lecture defines an indefinite integral as the collection of all antiderivatives of a given function. It discusses properties of indefinite integrals such as adding constants and changing variables. Common integrals are also presented, such as integrals of polynomials, trigonometric functions, and exponential functions.
Copyright
© © All Rights Reserved
Available Formats
Download as PDF, TXT or read online on Scribd
You are on page 1/ 256

AZERBAIJAN STATE OIL AND INDUSTRY

UNIVERSITY

LECTURES IN HIGHER
MATHEMATICS FOR ENGINEERS
(PART II)
TEXTBOOK

BAKU -2017

1
AZERBAIJAN STATE OIL AND INDUSTRY
UNIVERSITY

AZIMOVA G.M

LECTURES IN HIGHER
MATHEMATICS FOR ENGINEERS
(PART II)
TEXTBOOK

Confirmed by Azerbaijan State


Oil and Industry University.
Order № 01-I/33
“14” september

BAKU-2017

2
CONTENTS

Introduction……………………………………………… 4
Lecture 1. Indefinite Integral…………………………. 5
Lecture 2. Integration of Rational Functions………… 19
Lecture 3. Integration of Trigonometric Functions….. 38
Lecture 4. Definite Integral…………………………… 50
Lecture 5. Applications of Definite Integral…………. 71
Lecture 6. Improper Integrals……………………….... 87
Lecture 7. Numerical Series………………………….. 101
Lecture 8. Alternating Series…………………………. 117
Lecture 9. Functional Series………………………….. 126
Lecture 10. Differential Equations……………………. 139
Lecture 11. Differential Equations of Second-Order…. 156
Lecture 12. Nonhomogeneous Linear Differential
Equations…………………………………. 171
Lecture 13. Probability……………………………….... 188
Lecture 14. Bernoulli’s Formula. The Most Probable
Number of Occurrences of an Even……… 216
Lecture 15. Random Variable and the Law of its
Distribution…………………..................... 229
Bibliography……………………………………………… 254

3
Introduction
This book is the second part of the book based on
Lectures given by the author over a number of years to
engineering students at Azerbaijan State Oil and Industry
University.
The content of the present book covers the following
topics: indefinite, definite and improper integrals; numerical
and functional series; differential equations; elements of
probability theory.
The study of these sections of mathematics will help
future engineers to expand the scope of application of their
knowledge.
When preparing the book the author tried to select the
most important mathematical facts and present them so that the
student could acquire the necessary mathematical conception
and apply mathematics to other branches of science.
Much attention is paid to the solution of problems by
methods based on presented theory. The solving problems is
very useful for active learning of the material being studied.
As in the part I of the book, here at the end of each
lecture students are also offered tests for independent work.

4
Lecture 1
Indefinite Integral
Antiderivative
When differentiating a function we solve the problem of
finding of the derivative of a given function. Now we want to
solve the inverse problem: given the derivative of a function, to
find this function.
Definition. The function F (x) is called an antiderivative or a
primitive of the given function f (x) in a given interval if
F / ( x)  f ( x) or dF ( x)  f ( x)dx .
Let us consider some examples.
x5
1. The function F ( x)  is an antiderivative of the function
5
f ( x)  x 4 in the interval (, ) , since
/
 x5  1 5 / 1
 
F ( x)      x   5 x 4  x 4  f ( x) .
/

 5  5 5
2. The function F ( x)  sin 2 x is a antiderivative of the function
f ( x)  sin 2 x in the interval (, ) , since
 /
F / ( x)  sin 2 x  2  sin x  (sin x) /  2  sin x  cos x  sin 2 x  f ( x).
.
It is easy to see, from these two examples, that the
x5
derivatives of  C and sin 2 x  C , where C is an arbitrary
5
constant, are also equal respectively to x 4 and sin 2 x . These
examples indicate that a given function has infinitely many
antiderivatives because the constant C can take on arbitrary
values.
Thus we see that the problem of finding an antiderivative
has infinitely many solutions.

5
Properties of an antiderivative
1. If the function f (x) possesses an antiderivative F (x) , then
it possesses infinitely many antiderivatives, all the
antiderivatives being contained in the expression F ( x)  C ,
where C is an arbitrary constant.
2. Any two primitives differ by a constant.
Proof. Let F1 and F2 be two antiderivatives of f. It means that
F1/  f and F2/  f . Let us consider the difference F1  F2
and find its derivative. We have
( F1  F2 ) /  F1/  F2/  f  f  0 .
It follows that F1  F2  C , where C is a constant.
3. If the function f possesses an antiderivatives F and the
function g possesses an antiderivatives G, then the sum
f  g possesses an antiderivative F  G .
Proof. Indeed since F /  f and G /  g we have
( F  G) /  F /  G /  f  g .
4. If the function f possesses an antiderivatives F, then the
function kf , where k is a constant, possesses an antiderivative
kF .
Proof. Indeed we have
(kF ) /  k ( F ) /  k  f .
5. If the function f (x) possesses a primitive F (x) , then the
function f (t ) possesses a primitive F (t ) .
6. If the function f (x) possesses a primitive F (x) , then the
1
function f (ax  b) possesses a primitive F (ax  b) , where a
a
and b are constant and also a  0 .
Proof. According to the rule for finding the derivative of a
composite function we have

6
/
1  1 /
 F (ax  b)   F (ax  b)  a  f (ax  b) .
a  a

Indefinite Integral and its Properties


Definition. The collection of all antiderivatives of a function
f (x) is called the indefinite integral and is denoted by the
symbol  f ( x)dx .
 f ( x)dx  F ( x)  C
Here  is the integral sign, f (x) is the integrand, f ( x)dx is
the element of integration, and x is the variable of integration.
The process of finding an indefinite integral is called
integration of a function.
To find the indefinite integral of a function means to find all its
antiderivatives (for which it is sufficient to determine one of
them). This is the reason why we speak of indefinite
integration as it is not indicated which of the antiderivatives is
meant.
The graph of an antiderivative of a function f (x) is called an
integral curve of the function y  f (x) . It is clear that any
integral curve can be obtained by a translation (parallel
displacement) of any other integral curve in the vertical
direction. Therefore, geometrically, the indefinite integral is
represented by the set (collection) of all integral curves
obtained by continuous parallel displacement of all integral
curves obtained by continuous parallel displacement of one of
them in the vertical direction (Fig.1.1).

7
y

o x

Fig.1.1

Properties of an indefinite integral (rules of integration)

1.  f ( x)dx
/
 f ( x) .
Proof. By the definition of the indefinite integral we have
(  f ( x)dx) /  ( F ( x)  C ) /  F / ( x)  C /  F / ( x)  f ( x) .
2. d  ( f ( x)dx )  f ( x)dx .
Proof. According to the definitions of the differential and the
indefinite integral we have
d (  f ( x)dx)  (  f ( x)dx) / dx  f ( x)dx
3.  f / ( x)dx  f ( x)  C or  df ( x)  f ( x)  C .
Thus, the signs of integration and differentiation mutually
cancel out. The result of computing an indefinite integral can
always be verified by finding the derivative of the result.
4. A constant factor in the integrand can be taken outside the
sign of the integral:
 Cf ( x)dx  C  f ( x)dx , C=constant .
5. The integral of a sum of a finite number of functions is equal
to the sum of the integrals of these functions:
8
  f ( x)  f ( x)dx   f ( x)dx   f ( x)dx ,
1 2 1 2

  f  f  ...  f dx   f dx   f dx  ...   f


1 2 n 1 2 n dx .
6. On the invariance of integration formulas.
If  f ( x)dx  F ( x)  C and u   (x) any function possessing a
continuous derivative, then  f (u )du  F (u )  C .
Proof. From  f ( x)dx  F ( x)  C it follows that
F / ( x)  f ( x) .
For the composite function F (u )  F ( ( x)) its differential is
dF (u)  F / (u)du  f (u)du .
Therefore  f (u)du   dF (u)  F (u)  C .
Table of Basic Integrals
1.  dx  x  C
 x  1
2.  x dx   C for   1
 1
1
3.  dx  ln x  C
x
4.  sin xdx   cos x  C
5.  cos xdx  sin x  C
1
6.  cos 2
x
dx  tan x  C
1
7.  sin x dx   cot x  C
2

8.  tan xdx   ln cos x  C


9.  cot xdx  ln sin x  C
10.  e x dx  e x  C
9
ax
11.  a x dx  C
ln a
dx
12.   arcsin x  C
1 x2
dx x
13.   arcsin C
a x2 2 a

dx
14. 1 x 2
 arctan x  C
dx 1 x
15.   arctan  C
a x
2 2
a a
dx 1 xa
16.  2  ln C
x a 2
2a x  a
dx
17.   ln x  x 2  a 2  C
x a
2 2

Basic Methods of Integration


The basic methods of integration are the following methods:
1. The decomposition method
2. Integration by parts.
3. Integration by change of variable (by substitution)..

The Decomposition Method


A given integral can often be represented in the form of a sum
of tabular integrals (i.e. the integrals from the table). Then we
perform term wise integration and thus obtain the answer. Let
us consider several examples.

10
1.  a 0 
 a1 x  a 2 x 2  ...  a n x n dx 
 a 0  dx  a1  xdx a 2  x 2 dx ...  a n  x n dx 
1 1 1
 a0 x  a1 x 2  a 2 x 3  ...  a n x n 1  C
2 3 n 1
2.  3x  2 sin x  5e dx   3x dx   2 sin xdx   5e x dx 
2 x 2

x3
 3 x 2 dx  2 sin xdx  5 e x dx  3   2  ( cos x)  5e x  C 
3
 x 3  2 cos x  5e x  C

sin 2 x 1  cos 2 x
3.  tan 2 xdx   dx   cos 2 x dx 
cos 2 x
 1  1
   1dx   dx   dx  tan x  x  C .
 cos x 
2
cos 2 x

Integration by parts
The method of integration by parts is implied by formula
for differentiating a product of two functions. Let
u  u (x) and v  v(x) be functions of x possessing continuous
derivatives. We have
d (uv)  udv  vdu
whence
udv  d (uv)  vdu
Integrating both sides of the latter equality, we get
 udv   d (uv)   vdu
that is,
 u dv  uv   vd u (*)
This is the formula of integration by parts which can be
rewritten as follows
11
 u ( x)v ( x)dx  u( x)v( x)   v( x)u ( x)dx
/ /

We do not write down the arbitrary constant appearing in the


integration of d (u ) and include it into the arbitrary constant in
the second integral on the right-hand side of equality.
Let us consider some examples.
1. Let us find  xe x dx .
We put x  u, e x dx  dv, whence
du  dx , and v   e x dx  e x .
Now using the formula of integration by parts (*) we get
 xe dx  xe   e dx  xe  e  C
x x x x x

2. Let us find  x cos xdx .


Here we put x  u , cos xdx  dv , whence
du  dx and v   cos xdx  sin x
By formula (*) we obtain
 x cos xdx  x  sin x   sin xdx  x  sin x  cos x  C
x
2
3. Take the integral cos xdx .
Here we put x 2  u, cos xdx  dv, whence
du  2xdx, and v  sin x .
According to the formula (*) we have
 x cos xdx  x  sin x  2 x sin xdx .
2 2

The last integral is again found with the aid of integration by


parts. Finally we obtain
 x cos xdx  x  sin x  2 x cos x  2 sin x  C .
2 2

4. Let us find the integral  x ln xdx .


We put ln x  u, xdx  dv
whence
12
1 x2
du  dx, and v   xdx  .
x 2
Then by the formula (*) we get
x2 1 x2 x2
 x ln xdx  2 ln x  2  xdx  2 ln x  4  C .
5. Now let us consider the integral  x n ln xdx (n  1)
Putting ln x  u, x n dx  dv we find
1 x n 1
du  dx and v   x n dx 
x n 1
and thus we have
x n 1 x n 1
 x ln xdx  n  1 ln x   n  1 d ln x 
n

x n 1 1 x n 1 x n 1
n 1
 ln x  x n
dx  ln x   C.
n 1 n 1 (n  1) 2

6. Consider the integral  e x cos xdx .


We put cos x  u, e x dx  dv whence
du   sin xdx and v   e x dx  e x .
By formula (*) we obtain
 e cos xdx  e cos x   e sin xdx
x x x

Let us again apply integration by parts. On putting


sin x  u, e x dx  dv we find du  cos xdx , v  e x and hence
e sin xdx  e x sin x   e x cos xdx
x

Now substituting the expression thus obtained into the result of


the first integration we get
 e cos xdx  e cos x  e sin x   e cos xdx
x x x x

13
On transposing the integral from the right-hand side to the left
we find
1 x
 e cos xdx  2 e (cos x  sin x)  C .
x

Integration by change of variable


A change of variable in an indefinite integral is performed by
means of substitutions of two types:
I. x   (t ) , where  (t ) is a monotonic, continuously
differentiable function of the new variable t. In this case, the
integration is carried out by the formula
 f x dx   f [ (t )]   (t )dt ;
/.

II. u   (x) , where u is a new variable. With the aid of this


substitution, the integration is performed by the formula
 f [ ( x)]  ( x)dx   f (u )du
/.

Let us consider several examples.


1. Find the integral  (5 x  2) 8 dx .
1
Solution. Putting 5x  2  t , we get 5dx  dt , i.e. dx  dt ,
5
which yields
1 8 1 1 1
 (5x  2) 
dx t dt   t 9  C  (5 x  2) 9  C .
8

5 5 9 45
2. Find the integral  cos(4 x  1)dx .
1
Solution. We put 4 x 1  t whence 4dx  dt , i.e. dx  dt .
4
Therefore
1 1 1
 cos(4 x  1)dx  4  cos tdt  4 sin t  C  4 sin( 4 x  1)  C .
3. Find the integral  sin x cos xdx .
3

14
Solution. Putting sin x  u we get du  d (sin x)  cos xdx and
hence
u4 1
     C  sin 4 x  C .
3 3
sin x cos xdx u du
4 4
4. Find the integral  e cos xdx .
sin x

Solution. Putting sin x  u we obtain du  cos xdx and


therefore
 e cos xdx   e du  e  C  e  C .
sin x u u sin x

arctan x
5. Find the integral  1 x2
dx .
1
Solution. Putting arctan x  u we get du  and hence
1  x2
arctan x u2 1
 1  x 2 dx   udu  2  C  2 arctan x  C .
2

6. Find the integral  x 2 3 4  3x 3 dx


Solution. We put 4  3x 3  u whence  9 x 2 dx  du i.e.
1
x 2 dx   du . Thus,
9
1
1 3 1 3
 x 4  3x dx   4  3x x dx   9  u du   9  u du 
2 3 3 3 3 2

4
3 4
1u 1 1
  C   u 3  C   ( 4  3 x 3 )3 4  3 x 3  C.
9 4 12 12
3
Recurrence Formula for Finding the Integral
dx
In   2
(x  a 2 )n
The desired formula is obtained by means of integration by
parts. We have
15
dx 1 a2  x2  x2 1 a2  x2
In    2  ( x 2  a 2 ) n dx  a 2  ( x 2  a 2 ) n dx 
(x  a )
2 2 n
a
1 x2 1 1 x

a2  ( x 2  a 2 ) n dx  a 2 I n1  a 2  x  ( x 2  a 2 ) n dx .
x
Let us put u  x, dv  dx ; then du  dx and
(x  a 2 )n
2

xdx 1
v   ( x 2  a 2 ) n d ( x 2  a 2 ) 
(x  a )
2 2 n
2
1 1
  2
2(n  1) ( x  a 2 ) n1
Consequently
1 1  x 1 dx 
I n  2 I n 1  2 
a a  2(n  1)( x  a )
2 2 n 1
  2 n 1 
2(n  1) ( x  a ) 
2
,

or
1 x 1
In  I  2
2 n 1 2 n 1
 2 I n 1
a 2a (n  1)( x  a )
2
2a (n  1)
that is
x 1 2n  3
In  2 2 n 1
 2 I n 1 .
2a (n  1)( x  a )
2
a 2n  2
dx 1 x
For n  1 we have I 1   2  arctan  C .
x a 2
a a
Putting n  2 , we obtain an expression for the integral I n in
terms of the elementary functions:
1 x 1
I2  2  2  2  I1 
2a x  a 2
2a
1 x 1 1 x 
 2 2  2  arctan  C  .
2a x  a 2
2a  a a 
16
Assuming now n  3 , we find the integral I 3 (the integral
I 2 having been found). In this way we can find I n for any
positive integer n .

TEST
(1  x)2
1. Find  x x
dx

2 x 2  12 x  6
A)  C B) 2 x 2 12 x  C C) 3 x  5x 2  C
3 x
2x  6
D) C E) x  3 x 2  C
3 x
x
2. Find  dx
x4
A) ln x  4  C B) x  4 ln x  4  C C) x  C
D) ln x  ln x  4  C E) x  4  C
3. Find  x sin 2 xdx
A) 2 x sin x  cos 2 x  C B) x sin 2 x  C
1 1
C) sin 2 x  x cos 2 x  C D) x cos 2 x  C E) x  sin x  C
4 2
4. Find  xe dx
x

A) C  e  x ( x  1) B) e x  x  C C)  x  e  x  C
D) x  C E) e  x  C
5. Find  x arctan xdx
x2 1 x2 1 x
A) C B) arctan x   C
2 2 2
x2 x
C) arctan  C D) x  arctan x  C E) arccos  C
2 2

17
dx
6. Find 1 x 1
A) x 1  C B) ln 1  x  1  C C) ln x 1  C
1
D) 2 x  1  2 ln(1  x  1)  C E) C
x 1
x
7. Find  x( x  1)dx .
x
A) ln x 1  C B) ln x  C C) ln C
x 1
D) ( x  1)  C E) 2 arctan x  C
dx
8. Find e x
1  e 2 x
.

A) C  arcsin e  x B) arccos e 2 x  C C) e x  C
D) arctan e  x  C E) e 2 x  C
9. Find  x cos x 2 dx
1
A) cos x  C B) sin x  C C) sin x 2  C
2
1
D) cos x 2  C E) arccos x  C
5
dx
10. Find  x .
e (3  e  x )
1
A) ln e x  C B) e x  e  x  C C) ln 3x  C
3
D) C  ln(3  e  x ) E) 3  e x  C
Answers:
1.A 2.B 3.C 4.A 5.B 6.D 7.E 8.A 9.C 10.D

18
Lecture 2
Integration of Rational Functions (Rational Fractions)

Definition. A rational function (rational fraction) is a


fraction of the form
P( x)
Q( x)
where P(x) and Q(x) are polynomials.
Definition. A rational fraction is said to be proper if the
degree of the numerator P(x) is less than the degree of the
denominator; otherwise the fraction is said to be improper.
Let us consider the integral of the form
P( x)
 Q( x)dx
P( x)
If is an improper, then it can be represented in the form
Q( x)
of a sum of a polynomial and a proper fraction:
P( x) M ( x)
 L( x)  .
Q( x) Q( x)
Consequently,
P( x) M ( x)
 Q( x)dx   L( x)dx   Q( x) dx
The integration of the polynomial
L( x)  a0  a1 x  ...  a n x n
does not present any difficulties, and hence the problem
reduces to integrating the proper fraction in which the degree
of the numerator M (x) is less that of the denominator Q(x) .
The term partial (elementary) fractions is used for proper
fractions of the following types:
A
I. ;
xa
19
A
II. , where n is a positive integer;
( x  a) n
Ax  B p2
III. , where  q  0 , that is the quadratic
x 2  px  q 4
trinomial x 2  px  q does not possess real roots;
Ax  B
IV. , where n is a positive integer, and
( x  px  q) n
2

p2
 q  0.
4
In all the four cases it is assumed that A, B, p, q, a are real
numbers. The fractions we have enumerated will be called,
respectively, partial fractions of types I, II, III and IV.
M ( x)
It turns out that the proper rational fraction can be
Q( x )
represented as a decomposition into partial fractions.
Thus we need to know how to find the integrals of the partial
fractions.
Let us consider the integrals of the partial fractions of the first
three types. We have
A
I.  dx  A  ln x  a  C
xa
Indeed, putting x  a  t we find dx  dt and hence
A A 1
 x  a dx   t dt  A t dt  A ln t  C  A ln x  a  C
A A 1
II.  dx    C
( x  a) n
n  1 ( x  a) n 1
Here we do the same: x  a  t  dx  dt and we get

20
A A n
 ( x  a) n dx   t n dt  A t dt 
A 1 A 1
  n1  C    C
n 1 t n  1 ( x  a) n1
dx 2 2x  p
III.  2  arctan C
x  px  q 4q  p 2
4q  p 2
Indeed, for this special case of the partial fraction of type III we
obtain
2
 p p2
x  px  q   x    q 
2
or x 2  px  q  t 2  a 2 ,
 2 4
p 4q  p 2 p2
where t  x  , a (here  q  0 ), whence
2 2 4
dx dt 1 t
 x 2  px  q   t 2  a 2  a arctan a  C 
2 2x  p
 arctan .
4q  p 2
4q  p 2
dx
Example. Find the integral  2 .
x  8 x  32
Solution. We have
dx dx
 x 2  8 x  32   ( x  4) 2  16 
d ( x  4) 1 x4
  arctan C
( x  4)  16 4
2
4
We will show now how to integrate, in a general form, partial
fractions of type III.
Ax  B p2
It is required to find  2 dx , where  q  0 . Let
x  px  q 4
us isolate the derivative of the denominator from the numerator
21
of the fraction. To do that, we will represent the numerator in
the form
A Ap
Ax  B  (2 x  p)  B 
2 2
Then
A Ap
(2 x  p)  B 
Ax  B 2 2 dx 
 x 2  px  q dx   x  px  q
2

A 2x  p  Ap  dx
  dx   B   2
2 x  px  q
2
 2  x  px  q
In the first integral the numerator is the derivative of the
denominator; therefore,
2x  p d ( x 2  px  q)
 x 2  px  q dx   x 2  px  q 
du
  ln u  C  ln( x 2  px  q)  C
u
(here x 2  px  q  0 )
The second integral, as has been indicated, can be found by the
formula
dx 2 2x  p
 x 2  px  q  4q  p 2 arctan 4q  p 2  C
Thus, finally we have
Ax  B A
 x 2  px  q dx  2 ln(x  px  q) 
2

2 B  Ap 2x  p
 arctan C .
4q  p 2 4q  p 2
(3x  1)dx
Example. Find the integral  2 .
4 x  4 x  17
Solution. We have

22
3 3
(8 x  4)  1 
(3x  1)dx 8 2
 4 x 2  4 x  17   4 x 2  4 x  17 dx 
3 8x  4 1 dx
  2 dx   2 
8 4 x  4 x  17 2 4 x  4 x  17
3 d (4 x 2  4 x  17) 1 1 dx
     
8 4 x  4 x  17
2
2 4 2 17
x x
4
3 du 1 dx 3 1 dt
     ln u    2 
8 u 8  1
2
8 8 t  22
x  4
 2
3 1 1 t
 ln( 4 x 2  4 x  17)    arctan  C 
8 8 2 2
3 1 2x 1
 ln( 4 x 2  4 x  17)   arctan C .
8 16 4

Now let us show how to integrate the partial fractions of type


IV. It is required to find the integral
Ax  B p2
 ( x 2  px  q) n dx , where
4
 q  0.

Let us isolate the derivative of the denominator from the


numerator of the fraction:
A Ap
(2 x  p)  B 
Ax  B 2 2
 ( x 2  px  q) n dx   ( x 2  px  q) n dx 
A 2x  p  Ap  dx
  dx   B   2
2 ( x  px  q)
2 n
 2  ( x  px  q) n

23
The first integral on the right-hand side is easily found with the
aid of the substitution x 2  px  q  t whence
(2 x  p)dx  dt . We have
2x  p dt 1
 ( x 2  px  q) n dx   t n   (n  1)t n1  C 
1
 C
(n  1)( x  px  q) n 1
2

The second integral on the right-hand side must be transformed


as follows:
2x  p dx
 ( x 2  px  q) n    2 n
.
p 
2
 p 
 x     q   
 2  4  
p
Putting now x   t , dx  dt and introducing the
2
p2
designation q   a 2 we get
4
dx dt
 ( x 2  px  q) n   t 2  a 2 n .
Thus we see that integration of an elementary fraction of type
IV can be carried out with the aid of the recurrence formula.
Thus the integral of every rational fraction (rational function)
can be reduced to integrals of partial fractions of the indicated
types. Besides, the integral of a rational fraction is always
expressible in terms of elementary functions such as rational
functions, the logarithmic function and the arctangent.
2x  3
Example. Find the integral  2 dx .
( x  2 x  5) 2
Solution. We have

24
2x  3 2x  2  1
 ( x 2  2 x  5) 2 dx   ( x 2  2 x  5) 2 dx 
.
2x  2 1
 2 dx   dx
( x  2 x  5) 2 [( x  1) 2  4] 2
We perform the change of variable
x 2  2 x  5  t , (2 x  2)dx  dt
in the first integral and put x  1  z, dx  dz in the second
integral. We get
2x  3 dt dz
 (x 2
 2 x  5) 2
dx   2   2
t ( z  4) 2

1 1 z 1 1 z 
    arctan  C 
t 24 z  4 24 2
2
2 
Returning now to the old variable, we get
2x  3 1 1 x 1
 ( x 2  2 x  5) 2 dx   x 2  2 x  5  8  x 2  2 x  5 
1 x 1 x7 1 x 1
 arctan C   arctan C
16 2 8( x  2 x  5) 16
2
2
In the conclusion we formulate the main stages of integration
the rational fractions.
P( x)
Before integrating the rational fraction , we must perform
Q( x)
the following algebraic transformations and calculations:
I. If we are given an improper rational fraction, we have to
isolate its entire rational function (a polynomial), that is,
represent in the form
P( x) M ( x)
 L( x) 
Q( x) Q( x)
M ( x)
where L(x) is a polynomial and is a proper rational
Q( x )
fraction.
25
II. Factor the denominator of the fraction into linear and
quadratic multipliers:
p2
Q( x)  ( x  a) n ...(x 2  px  q) k ... , where q  0.
4
III. Decompose the proper rational fraction into partial
fractions:
M ( x) A1 A1 A
  n 1
 ...  n  ...
Q( x) ( x  a ) n
( x  a) xa
B1 x  C1 B x  C2 B x  Ck
...   2 2 k 1
 ...  2 k  ...
( x  px  q)
2 k
( x  px  q) x  px  q
IV. Compute the undetermined coefficients
A1 , A2 ,..., An , B1 , C1 , B2 , C2 ,...Bk , Ck ,... for which purpose it is
necessary to reduce the latter equality to the common
denominator, equate the coefficients in the same degrees of x
on the right-hand and left-hand sides of the identity obtained
and solve the system of linear equations with respect to the
sought-for coefficients.
Thus, as a result, integration of a rational fraction will reduce
to finding integrals of a polynomial and of partial rational
fractions.
Let us consider several examples.
x3
1. Find the integral  3 dx .
x x
Solution. The given proper rational fraction can be represented
as a sum of partial fractions of type I:
x3 x3 A B C
    .
x  x x( x  1)( x  1) x x  1 x  1
3

First of all we must find the unknown coefficients A, B and C .


Clearing the equation of fractions, we obtain.
x  3  A( x 2  1)  Bx( x  1)  Cx( x  1) .
Consequently, combining the terms of like degrees, we get
26
x  3  ( A  B  C) x 2  (B  C) x  A .
Since it is an identity, the coefficients in like powers of x on
both sides must be equal:
x  3  0  x 2  x  3  ( A  B  C) x 2  (B  C) x  A
Comparing the coefficients in like powers of x, we get the
system of equations
A  B  C  0

B  C  1
 A  3

From which we find A  3, B  1 and C  2 .
There is a still simpler method of determined the coefficients
A, B and C ; putting in succession x  0, x  1, x  1 , we
obtain  3   A,  2  2 B, and  4  2C , .that is, again
A  3, B  1 and C  2 .
Thus, decomposition of a rational fraction into partial fractions
has the form
x3 3 1 2
  
x  x x x 1 x 1
3

and therefore
x3 dx dx dx
 x 3  x dx  3 x   x  1  2 x  1 
x3
 3  ln x  ln x  1  2 ln x  1  C  ln C
( x  1)( x  1) 2
x2 1
2. Find the integral  ( x  1) 3 ( x  3)dx .

27
Solution. The factor ( x  1) 3 is associated with the sum of three
A B C
partial fractions   , and the factor
( x  1) 3 ( x  1) 2 x  1
D
x  3 , with the partial fraction . Therefore
x3
x2 1 A B C D
    .
( x  1) ( x  3) ( x  1)
3 3
( x  1) 2
x 1 x  3
Clearing the equation of fractions, we get
x 2  1  A( x  3)  B( x  1)( x  3)  C ( x  1) 2 ( x  3)  D( x  1) 3
.
Consequently,
x 2  1  A( x  3)  B( x 2  2 x  3) 
 C ( x 3  x 2  5 x  3)  D( x 3  3x 2  3x  1)
Combining the terms of like degrees, we get
x 2  1  (C  D) x 3  ( B  C  3D) x 2 
 ( A  2 B  5C  3D) x  3 A  3B  3C  D
Comparing the coefficients in like powers of x, we get the
system of equations
C   D
C  D  0 B  4 D  1
 B  C  3D  1 

   A  16D  2
 A  2 B  5C  3D  0 
3 A  3B  3C  D  1 D   5
 32
1 3 5 5
from which we find A  , B , C , D .
2 8 32 32
The unknowns A, B, C and D in the decomposition could have
been determined in another way. The real roots of the
denominator are the numbers 1 and -3. Putting x=1 we get
28
1
2=4A, i.e. A= . For x  3 , we have 10  64D , i.e.
2
5 5 3
D   , whence C  and B  .
32 32 8
Thus, finally decomposition of a rational fraction into partial
fractions has the form
x2 1 1 3 5 5
   
( x  1) ( x  3) 2( x  1)
3 3
8( x  1) 2
32( x  1) 32( x  3)
Thus, we get
x2 1 1 dx 3 dx 5 dx
 ( x  1) 3 ( x  3) dx  2  ( x  1) 3  8  ( x  1) 2  32  x  1 
5 dx 1 3 5 x 1
     ln C
32 x  3 4( x  1) 2
8( x  1) 32 x  3
x3  2x
3. Find the integral  ( x 2  1) 2 dx .
Solution. The given proper rational fraction can be represented
as a sum of partial fractions of types III and IV:
x 3  2x Ax  B Cx  D
 2  2
( x  1)
2 2
( x  1) 2 x 1
Clearing the equation of fractions, we obtain
x 3  2 x  Ax  B  (Cx  D)( x 2  1)
or
x 3  2 x  Ax  B  Cx 3  Dx 2  Cx  D .
Combining the terms of like degrees, we get
x 3  2 x  Cx 3  Dx 2  ( A  C ) x  B  D .
Comparing the coefficients in like powers of x, we get the
system of equations

29
C  1
D  0


 A  C  2
B  D  0
from which we find A  3, B  0, C  1, D  0 .
Consequently, decomposition of a rational fraction into partial
fraction has the form
x 3  2x  3x x
 2  2 .
( x  1)
2 2
( x  1) 2
x 1
Therefore, we get
x3  2x  3xdx xdx 3 d ( x 2  1)
 ( x 2  1) 2 dx   ( x 2  1) 2   x 2  1   2  ( x 2  1) 2 
1 d ( x 2  1) 3 1
  2   ln( x 2  1)  C.
2 x 1 2( x  1) 2
2

x 3  3x 2  5 x  7
4. Find the integral  x2  2
dx .
Solution. Since the integrand is an improper rational fraction,
first of all we have to isolate its entire rational function i.e. a
polynomial:
x 3  3x 2  5 x  7 x2  2
- 3
x  2x x3
3 x 2  3x  7
-
3x 2 6
3x  1
Thus, we have
x 3  3x 2  5 x  7 3x  1
 x 3 2 .
x 2
2
x 2
It follows that
30
x 3  3x 2  5 x  7  3x  1 
 dx    x  3  2 dx 
x 2
2
 x 2
3x  1
  xdx  3 dx   2 dx 
x 2
1 xdx dx
 x 2  3x  3 2  2 
2 x 2 x 2
1 3 d ( x 2  2) dx
 x 2  3x   2  2 
2 2 x 2 x 2
1 3 1 x
 x 2  3x  ln( x 2  2)  arctan C .
2 2 2 2
Integration of Irrational Functions
 
I. Integrals of the form  R x, m x , n x ,... dx, where R is
rational function with respect to the variable of integration x
and the radicals of x entering into it. Let us denote by k the
least common multiple of all the exponents m, n,...
LCM( m, n,... )=k; then
1 r1 1 r2
 ,  , ...
m k n k
and therefore all the quotients
k k
 r1 ,  r2 , ...
m n
are integers.
By means of the change of variable x  t k , dx  kt k 1dt.

 
1 1 k
m
x  xm  tk m  t m  t r1 ,

 t 
1 1 k
n
xx n k n
 t  t r2 ,
n

.........................................
the indicated integral can be transformed into an integral of a
rational function
31
 Rx,   
x , n x ,... dx   R t k , t r1 , t r2 kt k 1 dt   R1 (t )dt
m

where R1 (t ) is a rational function.


dx
Example. Find the integral  .
x 3 x
Solution. We put x  t 61 ; then x  t 3 , 3 x  t 2 , dx  6t 5 dt
and
dx 6t 5 dt t3 t3 11
 x  3 x  t3  t2  t 1
  6 dt  6  t  1 dt 
 dt   t3 t2 
6  (t 2  t  1)dt     6    t  ln t  1   C
 t  1 3 2 
Now returning to the original variable, we get
dx
 x  3 x  2 x  3 x  6 x  6 ln x  1  C
3 6 6

 ax  b n ax  b 
II. Integrals of the form  R x, m , , ...dx .
 cx  d cx  d 
In this case the integral is rationalized by the substitution
ax  b
 t k , where k is the least common multiple of all the
cx  d
exponents m, n,... , LCM( m, n,... )=k; then all the quotients
k k
 r1 ,  r2 , ... are integers and
m n
dt k  b
x  R1 (t ), dx  R1/ (t )dt ,
a  ct k

ax  b  ax  b  m
  t   t  t 1 ,
1 1 k
m  k m m r

cx  d  cx  d 
ax  b  ax  b  n
  t   t n  t 2 ,... .
1 1 k
n  k n r

cx  d  cx  d 

32
Thus, as a result, the indicated integral can be transformed into
an integral of a rational function
 ax  b ax  b 
 R x, m
cx  d
,
cx  d
, ...dx 
n

 
 

  R R1 t , t , t ,... R1 t dt   R 2 t dt
r1 r2

where R2 (t ) is a rational function.


dx
Example. Find the integral 
1  2x  4 1  2x
.

Solution. Here m  2, n  4 ; therefore k=LCM(2,4)=4


We apply the substitution 1  2 x  t 4 , then
x  1  t 4 , dx  2t 3 dt and hence,
1
2
dx  2t 3 dt t 2 dt t 2 11
 1  2x  4 1  2x  t 2  t
  2  t  1  t  1 dt 
 2

 t 2 1 1   1 
 2  dt   dt   2  (t  1)dt   dt  
 t 1 t 1   t 1 
 t 2  2t  2 ln t  1  C .
Now returning to the original variable by means of the
substitution t  4 1  2 x we get
dx
 1  2 x  4 1  2 x   1  2 x  2 1  2 x  2 ln 1  2 x  1  C
4 4

.
dx
III. Integrals of the form  .
ax 2  bx  c
By isolating a complete square from a quadratic trinominal,
such integrals reduce to the tabular integrals, of the form (13)
or (17) (see the table).

33
dx
Example. Find the integral  x 2  4 x  13
.

Solution. Let us reduce the quadratic trinomial to the form


x 2  4 x  13  ( x  2) 2  9 .
Then we will have
dx d ( x  2) dt
 2   
x  4 x  13 ( x  2)  9
2
t 9
2

 ln t  t 2  9  C  ln x  2  x 2  4 x  13  C
Ax  B
IV. Integrals of the form  ax 2  bx  c
dx .

This integral can be split into two by separating the part of the
numerator proportional to the derivative of the radicand. In
other words, let us expand the integral into a sum of two
integrals as follows
A Ab
(2ax  b)  B 
Ax  B
 ax 2  bx  c dx  
2a 2a dx 
ax  bx  c
2

A (2ax  b)dx  Ab  dx

2a  B
ax 2  bx  c 

2a  ax 2  bx  c

A d (ax 2  bx  c)  Ab  dx

2a  B
ax 2  bx  c 

2a  ax 2  bx  c
.

The first integral we have obtained is directly found (as an


integral of a power function)
d (ax 2  bx  c) du
 ax 2  bx  c   u  2 u  C  2 ax  bx  c  C
2

whereas the second integral has already been discussed.

34
3x  5
Example. Find the integral 
4 x 2  8x  5
dx

Solution. Let us isolate in the numerator the derivative of the


radicand:
3
(8 x  8)  2
3x  5 8
 dx   dx 
4 x 2  8x  5 4 x 2  8x  5
3 d (4 x 2  8 x  5) dx
   
8 4 x 2  8x  5 x  2x 
2 5
4
3 5
 4 x 2  8 x  5  ln x  1  x 2  2 x   C .
4 4

TEST
dx
1.Find  x(x  1) .

x 1
A) ln x  C B) ln x 1  C C) ln C
x
1
D) x  C E) ln x  5  C
5
dx
2.Find  ( x  1)(2 x  3) .
1 2x  3
A) ln  C B) ln 2 x  ln x  C C) ln 2 x  3  C
5 x 1
D) ln (2 x  3)( x  1)  C E) 2 x  3  C
dx
3. Find x 2
 2x  3
.

35
1 2 1 x 1
A) x 3C B) arctan C
2 2 2
1
C) arccos x 2  C D) arcsin x  C E) 2 x  3  C .
2
dx
4. Find  4x  3  x2
.

1
A) 4x  3  C B)arctan x 2  C
4
C) arccos x 2  C D) arcsin( x  2)  C E) arccos4 x  C

xdx
5. Find  2x 2
 3x  2
.

A)
1
5

ln ( x  2) 2 2 x  1  C  1
B) ln 2 x 1  C
5
C) ln x  2  C D) ln x  C E) 2 x  3x  C
2

( x  2) 2 dx
6.  x( x  1) 2 .
9 1 x
A) 4 ln x  3 ln x  1  C ln C
B)
x 1 4 x 1
1 x
C) ( x  1) 2  C D) x  C E) arctan C
5 x 1
dx
7. Find  x( x 2
 1)
.

A) ln x 2 1  C B) arctan x  C
x
C) arccos x  C D) ln  C E) ln x  C
x 2 1
dx
8. Find x 4
 x2
.

36
x4 1 1 x 1
A) ln C B)  ln C
5 x 2 x 1
1 x 1
C) ln C D) x 4  x 2  C E) arctan x  C
2 x 1
dx
9. Find  .
8  6x  9x 2
1
A) arctan x  C B) 8  6 x  9 x 2  C
8
x 1 1 3x  1
C) arccos x  C D) arcsin  C E) arcsin C
3 3 3
dx
10. Find  x4 x
.

A) x C B) 2 x  44 x  4 ln(1  4 x )  C
x
C) 1  x 4  C D) ln  C E) 4
x C
x 1
Answers:
1.C 2.A 3.B 4.D 5.A 6.A 7.D 8.B 9.E 10.B.

37
Lecture 3
Integration of Trigonometric Functions

I. Integrals of the form  R(sin x, cos x)dx , where R is a


rational function.
Integrals of the indicated form can be reduced to integrals of
rational functions with the aid of the so-called universal
x
trigonometric substitution t  tan .
2
As a result of this substitution we have
x x x
2  sin  cos 2 tan
sin x  2 2  2  2t ;
x x x 1 t 2
cos 2  sin 2 1  tan 2
2 2 2
x x x
cos 2  sin 2 1  tan 2
2  1 t ;
2
cos x  2 2
x x x 1 t 2
cos 2  sin 2 1  tan 2
2 2 2
x x
tan  t   arctan t  x  2 arctan t ;
2 2
2dt
dx  .
1 t2
Thus, we get
 2t 1  t 2  2dt
 R(sin x, cos x)dx   R 1  t 2 , 1  t 2   1  t 2   R1 (t )dt
 
where R1 (t ) is a rational function.
1
Example. Find the integral  dx .
1  sin x

38
Solution. The integrand depends rationally on sin x . Using the
x
substitution t  tan , we get
2
1 1 2dt dt
 1  sin x dx   
2t 1  t 2
 2
(1  t ) 2

1
1 t2
2 2
 C   C .
1 t x
1  tan
2
dx
Example. Find the integral  .
sin x
Solution. We have
2dt
dx 1  t 2  dt  ln t  C  ln tan x  C .
 sin x  2t  t

2
1 t 2

dx
Example. Find the integral  .
3 sin x  4 cos x  5
Solution. Here the integrand depends rationally on sin x and
x
cos x . Therefore, we apply the substitution t  tan ; then
2
2dt
dx 1 t2
 3 sin x  4 cos x  5   2t 1 t2

3  4 5
1 t2 1 t2
dt dt 2
 2 2  2  C .
t  6t  9 (t  3) 2
t 3
Returning to the old variable, we get

39
dx 2
 3 sin x  4 cos x  5   x
C.
tan  3
2
x
The universal substitution t  tan often leads to very
2
complicated expressions containing rational fractions. In
certain particular cases it is better to use some other
substitutions such as t  sin x , t  cos x or t  tan x , which we
are going to consider here.
I. If R(sin x, cos x) is an even function with respect to sin x and
cos x , that is, if R( sin x, cos x) = R(sin x, cos x) , then the
integral can be rationalized by means of the substitution
t  tan x .
1
Example. Find the integral  dx .
cos 4 x
Solution. Using the substitution t  tan x , we get
dx   tan 2 x  1d tan x 
1 1 1
 cos 4
x
dx   2
 2
cos x cos x
t3 1
  (t  1)dt   t  C  tan 3 x  tan x  C.
2

3 3
sin 2 x
Example. Find the integral  cos6 x dx .
Solution. We have
sin 2 x sin 2 x 1 1
 cos 6 x  cos 2 x  cos 2 x  cos 2 x dx 
dx 

  tan 2 x  (1  tan 2 x)  d tan x 


t3 t5
  t (1  t )dt   (t  t )dt    C 
2 2 2 4

3 5
1 1
 tan 3 x  tan 5 x  C .
3 5
40
dx
Example. Find the integral  sin
x  2 sin x  cos x  cos 2 x
2
.
Solution. The integrand is even with respect to sin x and cos x .
Setting t  tan x , we obtain
tan x tan x t
sin x  tan x  cos x    ;
1 1  tan 2 x 1 t 2
cos 2 x
1 1
cos x   ;
1  tan 2 x 1 t2
dt
x  arctan t ; dx  .
1 t 2
It follows that
dx
 sin
x  2 sin x  cos x  cos 2 x
2

dt
 2 1 t2 
t 2t 1 1
  
1 t2 1 t2 1 t2 1 t
2

dt d (t  1)
  
t  2t  1
2
(t  1) 2  ( 2 ) 2
1 t 1 2 1 tan x  1  2
 ln C  ln C .
2 2 t 1 2 2 2 tan x  1  2

II. If R(sin x, cos x) is an odd function with respect to sin x ,


that is, if R( sin x, cos x)   R(sin x, cos x) , the integral can
be rationalized by means of the substitution t  cos x .
(sin x  sin 3 x)dx
Example. Find the integral  .
cos 2 x

41
Solution. Since the integrand is odd with respect to the sin x ,
we put t  cos x . If follows that
sin 2 x  1  cos 2 x  1  t 2 ,
cos 2 x  2 cos 2 x  1  2t 2  1, .
dt  d cos x   sin xdx .
Thus we have
(sin x  sin 3 x)dx (1  sin 2 x) sin xdx (2  t 2 )(dt )
 cos 2 x
  cos 2 x
  2t 2  1 
t2  2 1 2t 2  4 1 2t 2  1  3
 2 dt   2
2 2t  1 2  2t 2  1
 dt 
2t  1
1 3 dt t 3 d ( 2t )
  dt   2    
2 2 2t  1 2 2 2 ( 2t ) 2  1
t 3 2t  1 1 3 2 cos x  1
  ln  C  cos x  ln C .
2 2 2 2t  1 2 2 2 2 cos x  1

III. If R(sin x, cos x) is an odd function with respect to cos x ,


that is, if R(sin x, cos x)   R(sin x, cos x) , then the integral
can be rationalized by means of the substitution t  sin x .
(cos3 x  cos5 x)dx
Example. Find the integral  .
sin 2 x  sin 4 x
Solution. Here the integrand is odd with respect to the cos x .
Therefore, we use the substitution t  sin x . Then
cos2 x  1  sin 2 x  1  t 2 , dt  d sin x  cos xdx .
Thus we have
(cos 3 x  cos 5 x)dx cos 2 x(1  cos 2 x) cos xdx
 sin 2 x  sin 4 x  
sin 2 x  sin 4 x

.
(1  t 2 )(2  t 2 )
 dt .
t2  t4
42
Note that
(1  t 2 )(2  t 2 ) 2  3t 2  t 4 t 2  t 4  2  4t 2
  
t2 t4 t2 t4 t2 t4
2  4t 2 2(1  t 2 )  6t 2 2 6
 1 2 4  1  1 2  .
t t t (1  t )
2 2
t 1 t 2
Therefore,
(1  t 2 )(2  t 2 )  2 6 
 t 2  t 4 dt   1  t 2  1  t 2 dt 
2 2
 t   6 arctan t  C  sin x   6 arctan(sin x)  C.
t sin x

II. Integrals of the form  sin m x  cos n xdx


Here we will consider two cases which are of especial
significance.
Case 1. At least one of the exponent m and n is an odd positive
number. If n is an odd positive number, then we use the
substitution sin x  t ; now if m is an odd positive number, the
substitution cos x  t .
Example. Find the integral  sin 8 x  cos 7 xdx .
Solution. Putting sin x  t , cos xdx  dt , we obtain.
 sin x  cos xdx   sin x  cos x  cos xdx 
8 7 8 6

  sin 8 x(1  sin 2 x) 3 d sin x 


  t 8 (1  t 2 ) 3 dt   t 8 (1  3t 2  3t 4  t 6 )dt 
  (t 8  3t 10  3t 12  t 14 )dt 
t9 t 11 t 13 t 15
  3  3  C 
9 11 13 15
1 3 3 1
 sin 9 x  sin 11 x  sin 13 x  sin 15 x  C.
9 11 13 15
43
Example. Find the integral  sin 5 x  cos 6 xdx .
Solution. Setting cos x  t ,  sin xdx  dt , we get
 sin x  cos xdx   sin x  cos x  sin xdx 
5 6 4 6

2
  (1  cos x) cos x sin xdx    (1  t )  t dt 
2 2 6 2 6

   (1  2t  t )t dt    (t  2t  t )dt 
2 4 6 6 8 10

t7 t 9 t 11 1 2 1
  2   C   cos 7 x  cos 9 x  cos11 x  C.
7 9 11 7 9 11

Case 2. Both exponents m and n are even positive numbers. In


this case the integrand must be transformed by mean of the
formulas
1
sin x  cos x  sin 2 x ;
2
1
sin 2 x  cos 2 x  sin 2 2 x ;
4
1  cos 2 x 1  cos 2 x
sin 2 x  ; cos 2 x  .
2 2
Example. Find the integral  sin 2 x  cos 2 xdx .
Solution. We have
1 1 1  cos 4 x
 sin
x  cos 2 xdx   sin 2 2 xdx   dx 
2

4 4 2
1 1 1
  (1  cos 4 x)dx  x  sin 4 x  C
8 8 32
Example. Find the integral  sin 4 x  cos 4 xdx .
Solution. We have

44
2
1 2 
 sin x  cos xdx   (sin x  cos x) dx    4 sin 2 x  dx 
4 4 2 2 2

1  1  cos 4 x 
2
1
   dx   (1  2 cos 4 x  cos 2 4 x)dx 
16  2  64
1 1 1 1  cos 8 x

64  dx   cos 4 xdx 
32 64  2
dx 

1 1 1 1
 x sin 4 x  x sin 8 x  C 
64 128 128 1024
3 1 1
 x sin 4 x  sin 8 x  C .
128 128 1024
Example. Find the integral  sin x  cos xdx .
2 4

Solution. We have
 sin x  cos xdx   (sin x  cos x) cos xdx 
2 4 2 2 2

1  1  cos 2 x
   sin 2 2 x  dx 
4  2
1 1
  sin 2 2 xdx   sin 2 2 x  cos 2 xdx 
8 8
1 1  cos 4 x 1 1
  dx   sin 2 2 x  d (sin 2 x) 
8 2 8 2
1 1 1
  dx   cos 4 xdx   sin 2 2 xd (sin 2 x) 
16 16 16
1 1 1
 x  sin 4 x  sin 3 2 x  C .
16 64 48

III. Integrals of the form  tan m xdx and  cot m xdx


These integrals can be found with the aid the formula
1 1
tan 2 x  2
 1 or cot 2 x  1,
cos x sin 2 x
45
by means of which the degree of the tangent or cotangent is
consecutively lowered
Example. Find the integral  tan 5 xdx .
Solution. We have
 1 
 tan xdx   tan 3 x  1dx   tan 3 xd tan x   tan 3 xdx 
5

 cos x 
2

 1 
4
tan x tan 4 x tan 2 x
   tan x  1dx    ln cos x  C.
 cos x 
2
4 4 2

Example. Find the integral  cot 8 xdx .


Solution. We have
 1 
 cotxdx   cot 6 x 2  1dx    cot 6 xd cot x   cot 6 xdx 
8

 sin x 
 1 
7
cot x cot 7 x
   cot 4 x 2  1dx     cot 4 xd cot x 
7  sin x  7
cot 7 x cot 5 x  1 
  cot 4 xdx      cot 2 x 2  1dx 
7 5  sin x 
7 5
cot x cot x
    cot 2 xd cot x   cot 2 xdx 
7 5
cot x cot 5 x cot 3 x  1 
7
      2  1dx 
7 5 3  sin x 
7 5 3
cot x cot x cot x
    cot x  x  C .
7 5 3

IV. Integrals of the form


 sin mx cos nxdx,  cos mx cos nxdx,  sin mxdx sin nxdx
These integrals can be found with the aid of the following
trigonometric formulas

46
1
sin   cos   sin(   )  sin(   ),
2
cos   cos   cos(   )  cos(   ) ,
1
2
sin   sin   cos(   )  cos(   ) .
1
2
by means of which make it possible to represent a product of
trigonometric functions as their sum.
Example. Find the integral  sin 3x  cos 7 xdx .
Solution. We have

 sin 3x  cos 7 xdx  2  sin 10x  sin(4 x)dx  2  sin 10xdx 


1 1

1 1 1

2  sin 4 xdx   cos10x  cos 4 x  C .
20 8
Example. Find the integral  cos x  cos 3x  cos 5 xdx .
Solution. We have

 cos x  cos 3x  cos 5 xdx  2  cos 4 x  cos 2 xcos 5 xdx 


1

1 1
  cos 4 x  cos 5 xdx   cos 2 x  cos 5 xdx 
2 2
  cos 9 x  cos x dx   cos 7 x  cos 3x dx 
1 1
4 4
1 1 1 1
 sin 9 x  sin x  sin 7 x  sin 3x  C.
36 4 28 12

47
TEST
dx
1.Find  1  sin x .
x 2 x
A) ln tan C B) C C) tan  C
2 x 2
1  tan
2
x
D) arctan  C E) arcsin x  C
2
sin 3 x
2.Find  dx .
cos4 x
1 1 1 1
A) 3
  C B) 4
 C C) C
3 cos x cos x cos x sin 3 x
1 1 1
D)    C E) tan x  C
x cos x sin x
dx
3. Find  .
cos x sin 3 x
A) ln tan x  C B) cos x  C C) ln cos x  C
1 1
D) ln tan x  2
 C E) C
2 sin x 2 sin 3 x
4. Find  sin 2 3 xdx .
1 1
A) sin x  C B) sin 6 x  C
6 12
1 1 1 1
C) x  sin 6 x  C D) cos 3x  C E) cos 6 x  C
2 12 2 12
5. Find  sin 3 x cos 2 xdx .
1 1 1 1
A) cos 5 x  x  C B) sin 3 x  C C)  C
5 3 x sin x
1 1 1
D) sin 5 x  sin 3 x  C E) cos 5 x  cos 3 x  C
5 5 3
48
6. Find  tan 3 xdx .
1 1
A) tan 2 x  ln cos x  C B) tan 3 x  C
2 2
1 1
C) ln cos x  C D) tan 3 x  C E) cos 5 x  C
3 2
7. Find  cot xdx .
3

1
A) ln sin x  C B)  cot 2 x  ln sin x  C
2
1
C)  tan 2 x  C D) ln cos x  C E) x  cot x  C
2
8. Find  (1  2 cos x) 2 dx .
A) 2 cos x  x  C B) x  sin x  C C) 3x  4 sin x  sin 2 x  C
1
D) x  cos x  sin x  C E) 3x  sin 2 x  C
2
9. Find  cos 3 xdx .
1 1
A) sin x  C B) cos 2 x  C C) cos 2 x  C
2 3
sin 3 x
D) sin x   C E) cos x  C
3
1  sin x
10. Find  dx .
cos x
A) ln(1  sin x)  C B) ln sin x  C C) x  sin x  C
D) 1  sin x  C E) cos x  C
Answers:
1.B 2.A 3.D 4.C 5.E 6.A 7.B 8.C 9.D 10.A.

49
Lecture 4
Definite Integral
There are various problems of geometry and physics leading to
the notion of the definite integral: determining the area of a
plane figure, computing the work of a variable force, finding
the distance travelled by a body with a given velocity, and
many other problems.
Definition. Suppose a function f (x) is defined on a closed
interval [a,b]. Let us divide the interval [a,b] into n arbitrary
parts by points a  x0  x1  x2  ...  xn  b , chose an arbitrary
point  k in each subinterval xk 1 , xk  and find the length of
every such subinterval: xk  xk  xk 1 .
The integral sum for the function f (x) on the closed interval
[a, b] is the sum of the form
n

 f (
k 1
k )xk  f (1 )x1  f ( 2 )x2  ...  f ( n )xn

Each term (summand) of the integral sum is equal to the area of


the rectangle with the base xk and the altitude f ( k ) and
which is obviously equal to f ( k )xk (Fig.4.1).

y  f (x)
f (k )

a  x0 x1 x2 xk 1 xk xn1 xn  b
o x
Fig.4.1
50
The integral sum is the area of the entire step figure consisted
of these rectangles.
The definite integral of the function f (x) on the closed
interval [a, b] is the limit of the integral sum under the
condition that the length of the greatest subinterval tends to
zero:
b n

 f ( x)dx  lim
max xk 0
 f (
k 1
k )xk .
a
The numbers a and b are called, respectively, the lower limit
and the upper limit of integration. As in the case of the
indefinite integral, the function f (x) is called the integrand,
the expression f ( x)dx is called the interval the element of
integration, the interval [a, b] is called the interval of
integration, and the variable x is the variable of integration.
Definition. A function f (x) for which the definite integral
exists is called integrable.
The process of forming the definite integral shows that the
b
symbol  f ( x)dx is a certain number.
a
Its value only depends

on the integrand f (x) and on the numbers a and b determining


the interval of integration.

Geometrical interpretation of the definite integral

Let us suppose that f ( x)  0 on the interval [a, b] .


Definition. A figure bounded by the graph of a function
y  f (x) , by the straight lines y  0, x  a and x  b is called
a curvilinear trapezoid (Fig.4.2).

51
y
y=f(x)

x=a x=b

o a y=0 b x

Fig.4.2
If f ( x)  0 on [a, b] , then, in geometrical interpretation, the
b
definite integral  f ( x)dx
a
is the area of a curvilinear

trapezoid Fig.4.2:
b
A   f ( x)dx .
a

Basic Properties of a Definite Integral

1. A definite integral does not depend on the notation of the


variable of integration, i.e.
b b b


a
f ( x)dx   f (t )dt   f (u )du  ...
a a
b
2.  dx  b  a .
a

It is clear that any integral sum for the function f ( x)  1 is


equal to b  a :
52
n n n

 f (
k 1
k )x k  1  x k   x k  x1  x 2  ...  xn 
k 1 k 1

 x1  x0  x 2  x1  ...  x n  xn 1  x n  x0  b  a
3. If the upper and the lower limits of the definite integral are
interchanged the integral only changes its sign:
b a


a
f ( x)dx   f ( x)dx .
b
4. If the limits of integration coincide then the integral is equal
to zero, i.e
a

 f ( x)dx  0
a
5.For any a, b, c there holds the formula
b c b


a
f ( x)dx   f ( x)dx   f ( x)dx
a c
(*)

Proof. Let a  c  b .
Since the limit of the integral sum does not depend on the way
the interval [a, b] is divided into parts, we can divide the
interval so that the point C is always a point of division. Then
the integral sum can be represented in the form
n

 f (
k 1
k )xk   / f ( k )xk   // f ( k )xk
k k
where the first sum on the right-hand side includes all the terms
corresponding to the points of division of the interval [a, c] and
the second sum includes all the terms corresponding to the
points of division of the interval [c, b] . The first and the second
sums are, respectively, integral sums for the function f (x)
corresponding to the intervals [a, c] and [c, b] .
If the number of the points of division increases indefinitely
and the length of the greatest subinterval tends to zero for the

53
whole interval [a, b] the same obviously takes place for the
intervals [a, c] and [c, b] ; therefore, passing to the limit as
max xk  0 , we get
n
lim
max xk  0
 f (
k 1
k )x k 

 lim
max xk 0

k
/
f ( k )x k  lim
max xk 0
k
//
f ( k )x k

The first limit on the right-hand side is equal to the integral


from a to c and the second to the integral from c to b, and thus
the required equality is obtained:
b c b

 f ( x)dx   f ( x)dx   f ( x)dx


a a c
Equality (*) also holds in the case when the point c lies outside
the interval [a, b] to the right of it, i.e. a  b  c , or to the left
of it, i.e. c  a  b .
Indeed, let a  b  c . As has been proved, since b lies between
a and c we have
c b c


a
f ( x)dx   f ( x)dx   f ( x)dx
a b
whence
b c c


a
f ( x)dx   f ( x)dx   f ( x)dx ,
a b
Now, interchanging the limits of integration in the second
integral on the right-hand side we obtain
b c b

 f ( x)dx   f ( x)dx   f ( x)dx


a a c
which is what we had to prove.
It can similarly be proved that equality (*) is valid for
c  a  b.
54
6. The integral of a sum of a finite number of functions is equal
to the sum of the integrals of these functions:
b b b

  f ( x)  g ( x)dx   f ( x)dx   g ( x)dx ,


a a a
b b b

  f ( x)  ...  f
a
1 n ( x)dx   f1 ( x)dx  ...   f n ( x)dx .
a a
Proof. According to the definition of the integral, we have
b n

  f ( x)  g ( x)dx  maxlim
a
x 0
  f ( k )  g ( k )x k
k
k 1

n n

 lim  f ( k )x k   g ( k )x k  
max xk  0
 k 1 k 1 
n n b b
lim
max x k  0
 k 1
f ( k )x k  lim
max xk 0
 g ( k )x k   f ( x)dx   g ( x)dx .
k 1 a a

7. A constant factor in the integrand can be taken outside the


integral sign:
b b

 Cf ( x)dx  C  f ( x)dx
a a

where C is constant.
Proof. According to the definition of the integral, we have
b n

 Cf ( x)dx  lim
max x k  0
 Cf (
k 1
k )x k 
a
n b
C lim
max x k  0

k 1
f ( k )x k  C  f ( x)dx .
a
b

8. If f ( x)  0 on [a, b] then  f ( x)dx  0 .


a

Proof. Let f ( x)  0 on [a, b] . Then in the integral sum all the


summands are nonnegative, and hence

55
n

 f (
k 1
k )xk  0 .

It follows that
b n

 f ( x)dx  lim
max xk 0
 f (
k 1
k )xk  0
a
since the limit of a nonnegative quantity cannot be negative.
b b
9. If f ( x)  g ( x) on [a, b] then  f ( x)dx   g ( x)dx .
a a

Proof. We have
b
f ( x)  g ( x)  f ( x)  g ( x)  0    f ( x)  g ( x)dx  0 
a
b b b b
  f ( x)dx   g ( x)dx  0   f ( x)dx   g ( x)dx .
a a a a

10. If m and M are, respectively, the least and the greatest


values of the function f (x) in the interval [a, b] ( a  b ) then
b
m(b  a)   f ( x)dx M (b  a) .
a
Proof. We have m  f ( x)  M for any x  [a, b] .
According to the previous property we can integrate the
inequality and hence
b b b

 mdx   f ( x)dx  Md x
a a a
b b b
m dx   f ( x)dx M  d x
a a a
b
m(b  a)   f ( x)dx M  (b  a) .
a
The geometrical meaning of the inequalities we have just
proved is the following (Fig.4.3):
56
The area of a curvilinear trapezoid is greater than the area of
the rectangle with altitude equal to m and is less than the area
of the rectangle with altitude equal to M.

o a b x

Fig.4.3

11. If at every point x  [a, b] the inequalities


 ( x)  f ( x)   ( x)
are fulfilled then
b b b

  ( x)dx   f ( x)dx   ( x)dx .


a a a
This means that an inequality between functions implies an
inequality of the same sense between their definite integrals,
or, briefly speaking, that it is allowable to integrate
inequalities.

Estimate of a Definite Integral


There holds the following theorem:

57
Theorem. The absolute value of the definite integral does not
exceed the integral of the absolute value of the integrand
function
b b

 f ( x)dx   f x  dx .
a a

Proof. For any x we have


 f ( x)  f ( x)  f ( x) .
It follows that
b b b
  f ( x) dx   f ( x)dx   f ( x) dx
a a a
that is
b b

 f ( x)dx  
a a
f ( x) dx.

Mean Value Theorem


The definite integral possesses the following important
property:
Theorem. Let f (x) be a continuous function on [a, b] .
Then there exists at least one point x    (a, b) such that
b

 f ( x)dx  f ( )(b  a) .
a

Proof. If the function f (x) is constant, i.e. f ( x)  C the


formula is obvious
b b

 Cdx  C  dx  C (b  a) .
a a

Let us suppose f (x) is nonconstant, then according to the


property 10 of the definite integral we have

58
b
1
b  a a
m f ( x)dx  M

and, hence,
b
1
b  a a
f ( x)dx  

where  is a number lying between the least value m and the


greatest value M of the function f (x) on [a, b] .
Since f (x) is continuous on [a, b] according to well-known
property of continuous function the function f (x) assumes all
the intermediate values between m and M. Therefore, there
exist at least one point   (a, b) at which f ( )   i.e.
b
1
b  a a
f ( x)dx  f ( )

In geometrical interpretation the mean value theorem shows


that the definite integral of a continuous function is equal to the
area of some rectangle with altitude f ( ) (Fig.4.4)

 

o a  b x
Fig.4.4

59
Integral with Variable Upper Limit

Let us consider the function of the form


x
F ( x)   f (t )dt
a
which is called the integral with variable upper limit. Here
the lower limit is fixed (constant), and the upper limit x is
variable.
For our further aims it is important to study some properties of
this function F (x) .

Continuity of the Integral with Variable Upper Limit

Theorem. If f (x) is integrable on [a, b] , then the function


F (x) is continuous on [a, b] .
Proof. Let us take an arbitrary x  [a, b] and give the argument
x an increment x .
Let us consider the difference F ( x  x)  F ( x) . We have
x  x x

F ( x  x)  F ( x)   f (t )dt  f (t )dt 


a a
x x  x x x  x
.
  f (t )dt   f (t )dt  f (t )dt   f (t )dt
a x a x

Now estimating the expression F ( x  x)  F ( x) we get


x  x x  x x  x
F ( x  x)  F ( x)  x
f (t )dt  x
f (t ) dt  x
f (t ) dt

Since the function f (x) is integrable on [a, b] then the function


is bounded on [a, b] , that is
f (t )  C for any t  [a, b] , C =constant .
60
Taking it into account, we obtain
x  x x  x
F ( x  x)  F ( x)   Cdt  C  dt  C  x .
x x

It follows that F ( x  x)  F ( x)  0 as x  0 , and, hence


lim F ( x  x)  F ( x) which means that the function F (x) is
x 0

continuous at x.

Differentiability of the Integral with Variable Upper Limit

Theorem. If a function f(x) is continuous on [a, b] then the


derivative of the integral with respect to its upper limit is equal
to the integrand:
/
x 
F ( x)    f (t )dt   f ( x) .
/

a 
In other words: an integral with variable upper limit is an
antiderivative of its integrand.
F
Proof. We have to prove that lim  f ( x) .
x 0 x

For the further it should be noted that


x  x x  x x  x
1 1
x dt  x 
x x
dt  1  f ( x ) 
x x
f ( x)dt .

We have
F F ( x  x)  F ( x) 1 x  x
x

x
  f t dt .
x x
F
Let us consider the difference  f (x) .
x

61
x  x
F 1
x
 f ( x) 
x  f (t )dt  f ( x) 
x

1   1 x  x
x  x x  x

x 
1
 f (t )  f ( x)dt .
x x
  f (t ) dt   f ( x ) dt 
x   x x
Since the function f (x) is continuous, it is clear that
x  0  t  x  f (t )  f ( x)  f (t )  f ( x)  0 .
It follows that for any   0 f (t )  f ( x)   and hence
x  x
F
  f (t )  f ( x)dt 
1
 f ( x) 
x x x
x  x

 f t   f x  dt 
1 1
     x  
x x
x
F
or  f (x)  
x
F
which means that lim  f ( x), i.e. F / ( x)  f ( x) .
x  0 x

Theorem. Any function continuous on [a,b] possesses an


antiderivative.

The Newton-Leibniz Theorem


Theorem. The definite integral is equal to the difference of the
values of any antiderivative of the integrand corresponding to
the upper and lower of integral.
b

 f ( x)dx  (b)  (a) , where ( x)  f ( x) .


a
In other words:
The definite integral is equal to the increment of any
antiderivative of the integrand gained on the interval of
integration.

62
This result is one of the most important theorem in
mathematics. It is called the Newton-Leibniz theorem, and
the equality
b

 f ( x)dx  (b)  (a)


a
is referred to as the Newton-Leibniz formula.
Proof. Let us consider the function
x
F ( x)   f (t )dt
a
and let (x) be arbitrary antiderivative of the function f (x) .
Since any two antiderivatives differ by a constant we get
F ( x)  ( x)  C
or
x

 f (t )dt  ( x)  C
a
Here putting x  a we obtain
a

 f (t )dt  (a)  C
a
or 0  (a)  C , C   ( a )

and hence
x

 f (t )dt  ( x)  (a) .
a
Now setting x  b we finally get
b

 f (t )dt  (b)  (a)


a
which is what we had to prove.

Methods of Evaluating Definite Integral

1.Newton-Leibniz formula:
63
b

 f ( x)dx  F ( x)  F (b)  F (a) ,


b
a
a

where F (x) is an antiderivative of f (x) , i.e. F / ( x)  f ( x) .


2. Integration by parts:
b b

 udv  uv   vdu ,
b
a
a a
where u  u (x) , v  v(x) are continuously differentiable
functions on the interval [a,b].
Proof. We have
d (uv)  vdu  udv
Integrating both sides of this equality from a to b we obtain
b b b

 d (uv)  v(du)   udv


a a a
whence
b b b

 udv  d (uv)   vdu


a a a
or
b b

 udv  uv a   vdu .
b

a a
3. Change of variable in the definite integral:

b 

 f ( x)dx   f [ (t )]  
/
(t )dt (1)
a

where x   (t ) and  / (t ) are continuous on interval [  ,  ],


 ( )  a,  (  )  b, and f [ (t )] is a function continuous on
[  ,  ].
Proof. Let us suppose F (x) is an antiderivative of f (x) . Then

64
 f ( x)dx  F ( x)  C , (2)

 f [ (t )]   (t )dt  F[ (t )]  C .
/
(3)

From (2) we get:


b

 f ( x)dx  F ( x)  F (b)  F (a) .


b
a
a
From (3) we obtain:

 f [ (t )]   (t )dt ]F[ (t )   F[ ( )   ( )]  F (b)  F (a)
/

Comparing the equalities we get the formula (1).


4. If f (x) is an odd function, i.e. f ( x)   f ( x) , then
a

 f ( x)dx  0 .
a
Proof. We have
a 0 a

a
 f ( x)dx  
a
f ( x)dx   f ( x)dx .
0
Now, changing the variable of integration in the first integral
on the right-hand side by means of the substitution x  t we
get
a a a a a


a
f ( x)dx   f (t )dt   f ( x)dx   f (t )dt   f ( x)dx  0 .
0 0 0 0
5. If f (x) is an even function, i.e. f ( x)  f ( x) , then
a a

 f ( x)dx  2 f ( x)dx
a 0
Proof. We do the same here

65
a 0 a a a

 f ( x)dx   f ( x)dx   f ( x)dx   f (t )dt   f ( x)dx 


a a 0 0 0
a a a
.
  f (t )dt   f ( x)dx  2   f ( x)dx
0 0 0
Let us consider several examples.
2

x
2
1. Evaluate the integral dx .
1
Solution. We have
2
2 3 (1) 3 8 1
2
x3
       3.
2
x dx
1
3 1
3 3 3 3

2
2. Evaluate the integral  cos xdx .
0

Solution. We have


2

 cos xdx  sin x
0
2
0
 sin
2
 sin 0  1  0  1 .

4
dx
3. Evaluate the integral  cos
0
2
x
.

Solution. We have


4
dx 
0 cos2 x  tan x 04  tan 4  tan 0  1  0  0 .
1

 xe
x
4. Calculate the integral dx .
0
Solution. We use here the method of integration by parts.
We put u  x, dv  e  x dx , whence du  dx, v  e  x .
Then
66
1 1

 xe dx   xe   e dx 
x x x 1

0
0 0
1 e2
 e 1  e  x  2e 1  1  .
0 e

2

e
x
5. Calculate the integral cos xdx .
0
Solution. We use here the method of integration by parts.
We set u  e x , dv  cos xdx , whence du  e x dx, v  sin x .
Then
  
2  2  2

 e cos xdx  e  sin x 02   e cos xdx  e 2   e sin xdx .


x x x x

0 0 0

Applying here the same method, we put e  u, sin xdx  dv , x

whence du  e x dx, v   cos x . Then we obtain


 
2   2

 e cos xdx  e 2  e  cos x 02   e cos xdx 


x x x

0 0

.
 2
 e 2  1   e x cos xdx .
0
It follows that
 
2
e 2 1
0 
x
e cos xdx .
2
e
ln 2 x .
6. Calculate the integral 1 x dx
Solution. We use here the method for change of variable.
dx
We put ln x  t , then  dt .
x
67
It is clear that if x  1, then t  0 ; if x  e, then t  1 .
Consequently,
e 1 1
ln 2 x t3 1 1
0 x  0   0  .
2
dx t dt
3 0
3 3
8
xdx
7. Evaluate the integral 
3 1 x
.

Solution. We set 1  x  t ; then x  t 2  1, dx  2tdt .


If x  3 , then t  2 ; if x  8 ,then t  3 . Therefore,
3
(t 2  1)  2tdt
8 3 3
xdx t3
   2 (t 2  1)dt  2   2t 2 
3

3 1 x 2 t 2
32
2 2 32 2
 (27  8)  2(3  2)   19  2   10 .
3 3 3 3

 sin
3
8. Calculate the integral x cos 2 dx .

Solution. Here the integrand is an odd function. Consequently

 sin x cos2 dx  0 .
3


TEST
1.Evaluate each of the following integrals:
1
1. 
0
1  x dx .

1 2( 8  1) 8 1 7
A) B) C) D) E)
8 3 3 27 72
1
dx
2.  (11  5x)
2
3
.

5 7
A)3 B)4 C) 5 D) E)
72 72
68
y 1
9
3.
4
y 1
dy .

2 1 2
A) 7 B) 7 C)7 D)8 E) 9
3 3 3
1
4.  (e x  1) 4 e x dx .
0

A) (e  1) 3 B) (e  1) 4 C) 0,2(e  1) 5 D)1 E) 0,2

1
dx
5. x
0
2
 4x  5
.

1 1 1
A) arctan B) arctan C) arctan D)1 E) 0
7 6 5
1

 xe
x
6. dx .
0

2 1
A)1 B) 1 C)0 D) E) e  2
e e
e 1
7.  ln(x  1)dx .
0

1
A) ln 2 B) ln 3 C) 1 D) ln 5 E) ln
2

2
8.  x cos xdx .
0

  
A) B) 0 C)  D)  1 E) 1
2 2 3
9
x
9. 
4 x 1
dx .

69
A) ln 4 B) 6  ln 3 C) 2 ln 2 D) 7  2 ln 2 E) 7
8
xdx
10.  .
3 1 x

1 35 32
A)8 B) 5 C) D)10 E)
3 3 3
Answers:
1.B 2.E 3.A 4.C 5.A 6.B 7.C 8.D 9.D 10.E

70
Lecture 5
Applications of Definite Integral
1. Computing the Area of a Plane Figure

The area of the curvilinear trapezoid (Fig.5.1) bounded


by the curve y  f ( x) ( f ( x)  0), and the straight lines
y  0, x  a and x  b can be calculated by the formula
b
A   f ( x)dx
a

y y=f(x)

o a b x
Fig.5.1

If f ( x)  0 on a, b then the area of a plane figure


(Fig.5.2) can be found from the formula
b b
A   f ( x) dx    f ( x)dx
a a

o a b x

Fig.5.2
f(x)<0
71
The area of the figure (Fig.5.3) bounded by the curves
y  f (x) and y  g ( x) ( f ( x)  g ( x)) and by the straight lines
x  a and x  b can be found from the formula
b
A    f ( x)  g ( x)dx
a

y=f(x)
0
y=g(x) 0
0
o a x
0b
0
Fig.5.3
Let us consider some special cases of calculating the area
of plane figures:

b b
A   f ( x)dx   g ( x)dx (Fig.5.4)
a c

y
f

o a c b x

Fig.5.4

72
c b
A    f ( x)  g ( x)dx   g ( x)  f ( x)dx (Fig.5,5)
a c

f(x) g(x)

g(x) f(x)

o a c b x

Fig.5.5

c b
A    f ( x)   ( x)dx   g ( x)   ( x)dx (Fig.5.6)
a c
y

g(x)
f(x)

φ(x)
o a c b x

Fig.5.6

73
If the line y  f (x) is specified by the parametric
equations x   (t ), y   (t ) , then the area of the plane figure
bounded by this line, the straight lines x  a , x  b and the
interval a, b  of the x-axis is expressed by the formula
t2

A   (t ) / (t )dt,
t1

where t1 and t 2 can be found from the equations


a   t1 , b   t 2  ( t   0 for t  t1 , t )
2

Let us consider some examples.


1. Find the area of the figure (Fig.5.7) bounded by the parabola
y  4 x  x 2 and the x-axis.
Solution. We have
4 x  x 2  0  x  0 and x  4

o 4 x
Fig.5.7

and hence, the parabola intersects the Ox axis at the points


(0,0) and (4;0). Therefore
4
4
 2 x3  32
A   (4 x  x )dx  2 x    ( sq. units )
2

0  3 0 3

74
2. Find the area of the figure (Fig 5.8) bounded by the curves
y  x and y  x 2 .

y
y=x2

y x

o 1 x

Fig.5.8

Solution. We have x  x 2  x  x 4  x  0, x  1
Consequently,
1
1
 2 32 1 3  1
A   ( x  x )dx   x  x   ( sq. units)
2

0 3 3 
0
3
x2 y2
3. Find the area A bounded by the ellipse 2  2  1 .(Fig,5.9)
a b
y
b

-a o 1 a x

b
b
Fig.5.9 Solution.
b
75 b
It is known that the ellipse is specified by the parametric
equations
x  a cos t , y  b sin t
Since  1  cos t  1 then  a  x  a ;
if x  a  cos t  1  t   ;
if x  a  cos t  1  t  0.
Consequently,
0 0

A  2 b sin t (a cos t ) dt  2 b sin t (a sin t ) dt 


/

 
0 
 2ab sin 2 tdt  2ab sin 2 tdt 
 0

1  cos 2t 
 2ab dt ab (1  cos 2t )dt 
0 2 0

 1 
 ab t  sin 2t   ab.
 2 0
4. Find the area of the figure (Fig.5.10) bounded by the curve
y  cos x and straight lines y  0, x  0 and x   .
Solution. It is clear that

o 
2
 x

Fig.5.10

76

A   cos x dx
0
where
 
 cos x for 0  x  2
cos x  
 cos x for   x   .
 2
Consequently,
 
2  2 
A   cos x dx   cos x dx   cos xdx   cos xdx 
0  0 
2 2

     
 sin x 02  sin x    sin  sin 0    sin   sin  
2  2   2
 1  1  2.
2. Computing the Arc Length of a Plane Curve
If the curve y  f (x) is smooth on the interval [a,b] (Fig.5.11)

y
M1 M2 Mi-1 Mi Mn-1
A B

o a x1 x2 xi-1 xi b x

Fig.5.11

77
(that is, the derivative y /  f / ( x) is continuous), then the
length of the corresponding arc of the curve can be found by
the formula
b
L   1  [ y / ] 2 dx .
a
Proof. Let y  f (x) be a smooth curve. We will find the
length of the arc contained between the vertical straight lines
x  a and x  b (Fig.5.11).
Let us take on the arc AB the points A, M 1 , M 2 ,..., M n1 , B,
and draw the chords AM1 , M 1 M 2 , ... , M n1 B. The lengths of
these chords we denote as l1 , l 2 ,..., l n . As a result we get
the broken line inscribed in the arc AB. The length of the
n
broken line is equal to Ln   li .
i 1
The limit
n
L  lim Ln
n 
or L  lim
max li 0
 l
i 1
i

is called the arc length of a plane curve.


Let us introduce the designation
y i  f ( xi )  f ( xi 1 ) .
Then we have
2
 y 
l i  x  y  1   i   x i
2 2

 x i 
i i

According to the Lagrange’s theorem we get


y i f ( x i )  f ( x i 1 )
  f / ( i ) where xi 1   i  xi
x i x i  x i 1

and consequently l i  1  f / ( i )  x i 
2

78
Therefore, the length of the broken line can be expressed by the
formula
 
n
Ln   1  f / ( i )  x i
2

i 1
Here, passing to the limit, we get

 
n b

 1  f / ( i )  x i   1  [ f / ( x)] 2  dx
2
L lim
max li 
i 1 a
or
b
L   1  [ y / ] 2 dx .
a
If the curve x   (t ), y   (t ) is represented parametrically
where  (t ) and  (t ) (  t   ) are continuously
differentiable functions, the arc length of the curve
corresponding to monotonic variation of the parameter t from
 to  can be calculated by the formula
 2
 / (t ) 
L   1   /    (t )dt    / (t )   / (t ) dt .
/ 2 2
   
   (t )  
Example. The curve described by a fixed point of a circle
rolling without sliding upon a straight line is called a cycloid
(Fig.5.12)

o 2 x

Fig.5.12

79
The cycloid is specified by the parametric equations
x  a(t  sin t ), y  a(1  cos t ).
One arc of the cycloid connecting two consecutive points of
contact with Ox is traversed as t runs through the interval
[0; 2  ].
Let us take a cycloid
x  a(t  sin t ), y  a(1  cos t ).
and find the length of its arc corresponding to the variation of t
from 0 to 2 .
Solution. We have
x /  a(1  cos t ), y /  a sin t.
2
L  
0
a 2 (1  cos t ) 2  a 2  sin 2 t dt 

2
 a  1  2 cos t  cos 2 t  sin 2 t dt 
0
2 2
t
a  2(1  cos t )dt  a  2  2 sin 2 dt 
0 0
2
2 2
t t
 2a  sin dt  2a  2cos 
0
2 20
 4a cos   4a cos 0  4a  4a  8a .

3. Computing the Volume of a Body


by Cross Section Areas
Suppose that we have a body bounded by a closed surface and
that we know the areas of its any parallel cross-sections by
planes perpendicular to a fixed straight line; for instance, this
line may be the Ox axis. Let it be necessary to compute the
volume of the body. The location of the cross-section is
determined by the abscissa x of the point of its intersection
80
with the Ox axis. With the change of x, the cross-section area
changes. Consequently, the cross-section area is some function
which we denote as S(x). We will assume that the area of the
cross-section is a continuous function S(x) where x is the
abscissa of the point of intersection of the cutting plane with
the Ox axis.
In addition, suppose that the whole solid is contained between
two planes perpendicular to the Ox axis and cutting it at points
a and b (a<b). Now we divide the interval [a,b] into n
subintervals with the aid of the points a  x0 , x1 , x2 ,..., xn  b .
Through these points we draw the planes perpendicular to the
x-axis. These planes will cut the body into n layers. Let us
denote the volume of each layer as Vi (Fig.5.13).

a x x b x
1 2
Fig.5.13
Thus the volume of the n-step body is expressed by the sum
n
Vn  V1  V2  ...  Vn   Vi .
i 1
The volume of the layer is approximately equal to the volume
of a cylinder whose height is equal to xi  xi  xi1 , and the
base coincides with the cross-section of the body,
corresponding to some abscissa ci ( xi 1  ci  xi ) and

81
consequently, has the area S(ci). The volume of a right cylinder
is equal to the product of the area of its base by the height.
Then we have
V1  S (ci )xi .
Therefore, replacing each layer by its corresponding right
cylinder, we get
n
Vn  S (c1 )x1  S (c2 )x2  ...  S (cn )xn   S (ci )xi .
i 1

The limit of this sum (which is an integral sum for the function
S(x) on the interval [a,b]) as n   and max xi  0 is the
desired volume:
n b

V  lim V  maxlim
n  n x 0
 S (ci )xi   S ( x)dx .
i i 1 a

4. Computing the Volume of a Body of Revolution


We now consider the volume of a body of revolution. Let us
consider a curvilinear trapezoid bounded by the curve
y  f (x) and straight lines y  0, x  a, x  b ( Fig5.14).

o a x b x

Fig5.14
If the curvilinear trapezoid is rotated around the x-axis then the
cross-section with abscissa x is a circle with radius equal to the
corresponding ordinate of the curve y  f (x) .

82
In this case
S ( x)   y 2   f 2 ( x) .
Therefore the volume of a body of revolution can be found by
the formula
b b
V    y 2 dx   f 2 ( x)dx .
a a

Example. Find the volume of a body bounded by surface of


revolution of a parabola y 2  x around the x-axis and the
plane x=h (Fig.5.15).
Solution. We have
h
h h
x2  h2
V    y dx   xdx  
2
 .
0 0
2 0
2

o h x

Fig.5.15

Example. The graph of y  sin 2 x, 0  x  is rotated
2
3600 around the x-axis. Find the volume of the body of
rotation.
Solution.

83
  

 
2 2 2
V   y 2dx    sin 2 x dx    sin 2 xdx 
2

0 0 0

 1 2  1 1
   cos 2 x      (1)     .
 2 0  2 2
If the figure bounded by the curves y  f (x) and
y  g (x) ( 0  f ( x)  g ( x) ) and straight lines x  a, x  b is
rotated around the x-axis, then the volume of a body of
revolution can be found by the formula

 
b
V    g 2 ( x)  f 2 ( x) dx .
a
Example. Find the volume of the solid formed by the rotation
of the figure bounded by the lines y 2  x and x 2  y around
the x-axis (Fig.5.16).
Solution. We have
y2  x x 4  x  x( x 3  1)  0

x2  y x  0; x  1

Applying the formula we get


   
1 1
V    ( x ) 2  ( x 2 ) 2 dx    x  x 4 dx 
0 0
1
 x2 x5   1 1  3
        0,3
2 5 0  2 5  10

84
y
y x

y=x2

o 1 x

y2=x

Fig.5.16

TEST
1.Find the area of the figure bounded by the curves
y 2  8x  16 and y 2  24x  48
2 32 10
A) 10 B) 6 C) 6 D) 2 6 E) 6
3 3 3
2. Find the area of the figure bounded by the parabola
y  3  2 x  x 2 and the x  axis .
2 2 1 4
A) 10 B)10 C) D) E)
3 3 3 5
3. Find the area of the figure bounded by the curves
y 2  2 x  4 and x  0 .
17 16
A) B)0,5 C) D)15 E) 0,2
3 3
4. Find the area of the figure bounded by the curves y  x 2 and
x3
y .
3
1 1 1 1
A) 2 B) 2 C) 2 D) 2 E) 2
7 6 5 4

85
5. Find the area of the figure bounded by the parabola
y  6 x  x 2 and the x-axis.

A) 4 B)34 C)30 D)36 E) 5


6. Find the area of the figure bounded by the line y  x and
the parabola y  2  x 2 .

1 1 1
A) 4 B)4 C)5 D) 5 E) 1
2 2 2

7. Find the length of the arc of the curve y 2  x 3 from x  0


to x  1.
13 13 13 13  8
A) B) C)1 D)5 E) 7
8 27
8. Find the length of the arc of the curve y  ln sin x from
 
x to x  .
3 2
A) ln 5 B) ln 4 C) ln 2
D) ln 7 E) ln 3
t3
9. . Find the length of the arc of the curve x   t ,
3
y  t  2 from t  0 to t  3 .
2

A)8 B)9 C)12 D)11 E) 10


3

10. Find the length of the arc of the curve y  x , if 2

0 x4
80 10  8 80 80 10
A) B) C) 10 D) 5 E)
27 27 27
Answers:
1.B 2.A 3C 4.D 5.D 6.A 7.B 8.E 9.C 10.A

86
Lecture 6
Improper Integrals
Up to now, when speaking of definite integrals we assumed
that the interval of integration was finite and closed and that
the integrand was continuous.
Improper integrals are:
(1) integrals with infinite limits of integration;
(2) integrals of unbounded functions.

Improper Integrals with Infinite Limits

We introduce the following definition



Definition. The improper integral  f ( x)dx
a
of the function

f (x) over the interval [a; ) is the limit of the integral


b

 f ( x)dx as b   provided
a
that this limit exists:

 b

 f ( x)dx  lim  f ( x)dx


a
b
a
If the limit exists and is finite, the improper integral is called
convergent; now if the limit does not exist or is equal to
infinity, it is called divergent.
Similarly,
b b



f ( x)dx  lim  f ( x)dx
a 
a
and
 b

 f ( x)dx  lim  f ( x)dx .



a  
b   a

87
Examples.


e
x
1. Compute the improper integral dx .
0
Solution. We have
 b b

 e dx  lim  e dx  lim (e ) 


x x x
b b 0
0 0

 1
 lim (e b  e0 )  lim 1  b   1 .
b b
 e 


e
x
Consequently, the improper integral dx is equal to 1 and
0
hence, it is convergent.
1
dx
2. Compute the improper integral x

2
.

Solution. We find
1
1
dx 1
dx  1  1
 x 2  alim
 
a x
2
 lim
a 
    lim1    1
 x  a a a 
that is, the improper integral converges.

dx
3. Compute the improper integral  .
 1  x
2

Solution. The integrand is even, therefore


 
dx dx
1  x 2 0 1  x 2 .
 2

Then we have
 b
dx dx
0 1  x 2 b 0 1  x 2  blim
 arctan x 0 
b
lim



 lim (arctan b  arctan 0)  lim arctan b  .
b b 2

88

dx
Thus, finally we get 1 x

2
 ,

that is, the improper integral is convergent.



4. Compute the improper integral  cos xdx
0
(or establish its

divergence).
Solution. We have
b b
lim  cos xdx  lim sin x  lim (sin b  sin 0)  lim sin b .
b b 0 b b
0

The limit lim sin b does not exist since sin b oscillates
b  

between -1 and 1 as b   . Consequently, the improper


integral is divergent.

Comparison Tests for Convergence or Divergence of


Improper Integrals with Infinite Limits

To investigate the convergence of improper integrals, we use


one of the comparison tests. To this end we will prove the
following theorems (comparison tests).
Theorem 1. Let the inequalities
0  f ( x)   ( x)
hold for all the values of x  a .

Then, if the integral   ( x)dx
a
is convergent the integral



 f ( x)dx is convergent as well.


a

Proof. Let us suppose that the integral   ( x)dx is convergent
a
and is equal to M. It is clear that
89
b 

  ( x)dx    ( x)dx  M
a a
for any b

and, hence
b

  ( x)dx  M
a
for any b.

According to the property on integrating inequalities, we get


b b
f ( x)   ( x)   f ( x)dx    ( x)dx M .
a a
b
It follows that I (b)   f ( x)dx is a bounded function, and
a

therefore the integral  f ( x)dx is convergent. The theorem has
a
been proved.
Theorem 2. Let the inequalities
0  f ( x)   ( x)
hold for all the values of x  a .

Then, if the integral  f ( x)dx
a
is divergent the integral



  ( x)dx is also divergent.


a

Proof. Now suppose that the integral  f ( x)dx is divergent.
a
b
Then the increasing function  f ( x)dx
a
tends to   as

b   . But since
b b

  ( x)dx   f ( x)dx ,
a a

90
b
the function   ( x)dx also tends to
a
  as b   and hence,

the integral   ( x)dx
a
is divergent, which is what we had to

prove.

dx
Example 1. Test the integral x
a
p
for convergence.

Solution. If p=1 we have by definition


 A
dx dx
a x  Alim 
A
 lim ln x a  lim (ln A  ln a )  
 x A A
a
since ln A   as A   . Consequently, at p=1 the

dx
integral  is divergent.
a
x
If p  1 we have by definition
 A
dx
A
dx
A
 1 
a
x p
 lim
A 
a
x p
 lim
A 
a
x  p dx  lim 
A  p  1

x  p 1  
a
 1 1 
 lim  A p1  a  p 1  
A  p  1  p 1
 
1 1 1 1
 lim p 1   p 1 .
 p 1 A  A p 1 a
1
Assume that p  1 , then p  1  0 and lim p 1  0 .
A  A

It follows that at p  1 the integral converges and



dx 1 1
x a
p
  p 1 .
p 1 a

91
1
Assume now that p  1 , then p  1  0 and lim   .
A  A p 1
This means that at p  1 the integral diverges.
Therefore, finally we get
 1 1

dx   p 1 at p  1
a x p   p  1 a
  at p  1

Thus, we have
dx converges at p  1


a x p  diverges at p  1.

dx
Example 2. Test the integral 1 x
1
8
for convergence.

Solution. The integrand


dx
f ( x) 
1  x8
1
is smaller over the interval of integration than  ( x)  since
x8
1 1
 ,
1  x8 x8

dx
and the integral x
1
8
is convergent (see the previous example).

Therefore, using the comparison test (theorem 1), it follows


that, the given integral converges too.

dx
Example3. Test the integral 
1 1  x  3 1  x2
for convergence.

Solution. It is clear that the inequality

92
1 1 1
 
1 x  1 x
3 2 1 2 7

x x
2 3
x6

dx
holds for all x  1, and the integral 
1
7
is convergent since
6
x
7
p   1 . Consequently, applying the comparison test
6
(theorem 1), we get that the given integral is also convergent.

x
Example 4. Test the integral  1  xdx
1
for convergence.

Solution. It is obvious that the inequality


x x 1
 
1 x x  x 2 x

dx
holds for all x>1 and the integral 
1 x
is divergent because

1
p   1 . Therefore, according to the comparison test
2
(theorem 2), the given integral is also divergent.

Theorem 3. If the integral 
a
f ( x) dx of the absolute value of

the function f (x) is convergent the integral  f ( x)dx
a
is also

convergent.

In this case the integral  f ( x)dx
a
is said to be absolutely

convergent. Therefore, theorem 3 means that the absolutely


convergent integral is also convergent.

93
 
cos x sin x
Example 5. The integrals 0 1  x 2 dx and 0 1  x 2 dx are
absolutely convergent because the absolute values of the
1
integrands do not exceed the positive function and the
1 x2

dx
integral  is convergent. Indeed, we have
0 1 x
2

cos x cos x 1
 
1 x 2
1 x 2
1 x2
and, similarly,

sin x 1

1 x 2
1 x2
By definition

dx
b
dx 
0 1  x 2 b a 1  x 2  blim
 arctan x 0  lim arctan b 
b
lim
 b  2
and hence, the integral is convergent.
Now, applying the comparison test (theorem 1), we get that the
integrals
 
cos x sin x
0 1  x 2 dx and 0 1  x 2 dx
are convergent, and it means that the integrals
 
cos x sin x
0 1  x 2 dx and 0 1  x 2 dx
are absolutely convergent.
It is easy to prove the following theorem.

94

Theorem 4. If the integral 

f ( x) dx is convergent, the
 
integrals 

f ( x) cos xdx and  f ( x) sin xdx

are absolutely

convergent.
Proof. Indeed, it clear that the absolute values of the integrands
obviously do not exceed f (x)
f ( x) cos x  f ( x)  cos x  f ( x)
f ( x) sin x  f ( x)  sin x  f ( x) .
Therefore, according to the comparison test (theorem 1) we get
that both given integrals are absolutely convergent, which is
what we had to prove.

It should be noted, the fact that the integral  a
f ( x) dx is
divergent does not enable us to judge upon the convergence of

the integral  f ( x)dx
a
because it can be either divergent or

convergent.

Definition. The improper integral  f ( x)dx
a
is said to be

conditionally (not absolutely) convergent if the integral


 


a
f ( x)dx is convergent and the integral a
f ( x) dx is
divergent.

sin x
Example 6. Let us consider the Dirichlet integral
0
x
dx . It 
can be shown that the Dirichlet integral is conditionally
convergent because this integral is convergent and its
numerical value is found by means of some special techniques:
95

sin x 
0
x 
dx 
2
while the integral of the absolute value of the integrand is
divergent:

sin x
0 x dx   .
Thus, the Dirichlet integral is conditionally divergent.

Improper Integrals of Unbounded Functions

Let y  f (x) be a continuous function for all x  [a, b) (i.e.,


a  x  b ) having an infinite discontinuity at x  b . It is clear
that the ordinary definition of the definite integral is
inapplicable here because the function f (x) is unbounded in
[a,b]. Let us first take the ordinary definite integral
b 
I ( )   f ( x)dx
a
where   0

and then make  tend to zero: lim I ( ) . Then I ( ) either tends


 0
to a finite limit or has no finite limit (in the latter case it either
tends to infinity or has no limit at all).
We introduce the following definition
b
Definition. The improper integral  f ( x)dx of
a
a function

f (x) continuous for a  x  b and unbounded as x  b is the


b
limit of the integral  f ( x)dx
a
as   0 (  0) provided this

limit exists:
b b 

 f ( x)dx  lim
a
  f ( x)dx ,
0
a
  0.

96
If the limit exists we say that the improper integral is
convergent. If this limit does not exist the equality becomes
meaningless, and the improper integral written on the left is
said to be divergent.
Similary, if the function f (x) has an infinite discontinuity only
at x  a of the interval [a,b] and is continuous for all
x  (a, b] (i.e., a  x  b ) then, we put by definition
b b


a
f ( x)dx  lim
 0  f ( x)dx,
a 
where   0 .

provided this limit exists.


Definition. If the function f (x) possesses an infinite
discontinuity at a point c  (a, b) and is continuous at
a  x  c and c  x  b , then, by definition
b c  b

 f ( x)dx  lim
a
  f ( x)dx  lim
0
a
  f ( x)dx .
0
c
b
The improper integral  f ( x)dx (where
a
f (c)  , a  c  b ) is

said to be convergent, if both limits exist on the right-hand


side of the equality, and divergent, if either of them does not
exist.
Now let us consider some examples.
1
dx
Example 7. Compute the improper integral  .
0
x
1
Solution. The integrand f ( x)  at the point x  0 is
x
unbounded and, therefore, we have
1 1 1
dx dx
0 x  0  x  lim
 lim
 0
ln x  lim(ln1  ln  )   ,
  0

that is, the improper integral diverges.

97
Examples. Calculate the following improper integrals:
a a a
dx dx
1.   lim   lim 2 x  lim(2 a  2  )  2 a .
0 x  0  x  0   0

1 1
2. 
dx
 lim
dx
 1  x  lim  ln(1  x) 1

0
1  x  0 0
 0 0

 lim  ln   ln 1   lim ln    .
 0  0
That is, the integral is divergent
1
dx
3.  . In this example the integrand has infinite
1 1  x
2

discontinuities at both end points of the interval of integration.


We have
1
1 1
dx dx
  lim  lim arcsin x 
 0  0
1 1 x2  0 1 1 x2  0 1

 lim arcsin(1   )  arcsin( 1   ) 


 0
 0

 lim arcsin(1   ) lim arcsin( 1   ) 


 0  0

  
 arcsin 1  arcsin( 1)      .
2  2

TEST
1. Compute the following improper integrals with infinite
limits (or establish their divergence)

dx
1. x
1
4

98
1 1 1 1
A) B) C) D) E) diverges
3 2 4 5



e
 ax
2. dx (a  0)
0

1 1 1
A) 6 B) C) D) diverges E)
5 2a a

2 xdx
3. x

2
1
1
A) 2 B) diverges C) D)0,6 E) 7
2

dx
4. x

2
 2x  2
A)  B) 2 C) diverges D) 3 E) 4

 xe
 x2
5. dx
0

A) 0,8 B) diverges C) 0,5 D) 0,6 E) 0,7


ln x
6. 
2
x
dx

1 1 1 1
A)diverges B) C) D) E)
2 3 4 5

99
7.Compute the following improper integrals of unbounded
functions (or establish their divergence)
1
dx

0 1 x2
 
A)  B) C) D) 2 E)0
2 3
2
dx
8. x
0
2
 4x  3
A)0 B) 1 C) diverges D) 5 E) 6

2
xdx
9. 
1 x 1
8 1
A) 1 B) C) 5 D) 2 E) 0
3 3
1
10.  x ln xdx
0

1 1 1
A)  1 B)  C)diverges D)  E) 
2 4 5

Answers:
1)A 2)E 3)B 4)A 5)C

6)A 7)B 8)C 9)B 10)D

100
Lecture 7
Numerical Series
Let us consider an infinite sequence of numbers u1 , u2 ,..., un ,... .
Definition. An expression
u1  u2  ...  u n  ...
is called a numerical series.
Here, the numbers u1 , u 2 ,... are called the terms of the series.
A series is briefly written as

u
n 1
n

and u n is called the n-th term or the general term of the series.
Let us introduce the following notations:
S1  u1
S 2  u1  u 2
S3  u1  u2  u3
.........................
S n  u1  u2  ...  un
The sum S n is the sum of the first n terms of the series. This
sum S n is called the nth partial sum of the series.
Definition. If the sequence of partial sums S n of the given
series has a finite limit as n   , i.e.
lim S n  S
n 

the series is said to be convergent and the number S is called


the sum of the series.
In this case we write

S   un  u1  u2  ...  un  ...
n 1

101
If the sequence S n does not tend to any limit, the series is said
to be divergent.
It should be noted that a series can be divergent in the
following two case:
(1) when the sequence S n tends to infinity, i.e.
lim S n   or lim S n   or lim S n  
n  n  n 

(2) when the sequence S n has neither finite nor infinite limit.
In both cases the series is said to have no sum.
Definition. Let us consider a series

u
n 1
n  u1  u 2  ...  un  ...

A series of the form



rn  u
k n 1
k  u n1  un2  ...

is called the n th remainder of the series.

Necessary Condition for Convergence of a Series

Theorem. (Necessary Condition) If a series is convergent its


general term tends to zero as n   :

u
n 1
n is convergent  lim un  0 .
n

Proof. We have
S n  u1  u2  ...  un1  un  S n1  un
whence
un  S n  S n1 .
If the series is convergent we have
lim S n  S and lim S n1  S .
n  n 

Since

102
lim u n  lim ( S n  S n 1 )  lim S n  lim S n 1  S  S  0
n  n  n  n 

we get
lim u n  0
n 
which is what we had to prove.
The necessary condition we have proved implies the following
sufficient test for divergence of a series.
Theorem. If the general term u n does not tend to zero as
n   the series is divergent.

lim u n  0   u n is divergent .
n
n 1
1 2 n
For example, the series   ...   ... is divergent
2 3 n 1
because its general term does not tend to zero:
n
lim u n  lim 1 0 .
n  n  n  1

It should be noted that the fact that lim u n  0 is not sufficient


n 

for the convergence of the series.


For example, let us consider the so-called harmonic series:

1 1 2 1
n 1 n
 1    ...   ... .
2 3 n
1
Here we have lim u n  lim  0 but the series is divergent. Let
n  n  n

us prove that the harmonic series is divergent. It can be shown


that in this case lim S n   . To this end we will replace some
n 

terms of the series by smaller numbers and then show that the
sum of the smaller summands tends to infinity.
We have

103
1 1 1 1 1 1 1 1 1
1              ...    ... 
2 3 4 5 6 7 8 9 16 
1 1 1 1 1  1 1
 1        ...      ...    ... 
2  4 4 8 8   16 16 
2 4 8

1 1 1 1
= 1     ...
2 2 2 2

Here below each bracket we have written the number of


summands it contains. It is clear that the latter sum tends to
infinity. It follows that lim S n   and, consequently, the
n 

harmonic series is divergent.


Let us consider some examples.
Example. Let us consider an infinite geometrical series:

1  q  q  q  ...  q  ...   q n .
2 3 n

n 0
We will prove that
 is convergent if q  1
q
n 0
n

is di vergent if q  1 .
Indeed, the sum of the first n terms of the series is
n 1 1  qn 1 qn
S n  1  q  q  q  ...  q 
2 3
  .
1 q 1 q 1 q
If q  1 then lim q n  0 , and, consequently,
n

 1 qn  1
lim S n  lim    .
n n 1  q 1 q  1 q

Thus, an infinite geometric series with q  1 is convergent and
its sum is given by
104
1
S  lim S n 
.
1 qn

If q  1 then lim q n  , and hence, lim S n   , that is , the


n n 

series is divergent.
Example. The series

 (1)
n 1
n1
 1  1  1  1  ...  (1) n1  ...

is divergent because its general term un  (1) n1 has no limit


as n   and hence does not tend to zero.

Properties of Convergent Series

Theorem 1. Let C be an arbitrary number.


 
If a series  u n is convergent, the series  cun is also
n 1 n 1
convergent and
 
 cun  c  u n
n 1 n 1
Proof. Let us designate the n th partial sums of the given series
as follows
S n  u1  u 2  ...  u n
S n/  cu1  cu2  ...  cun
Then

S n/  c  S n

If the series  u n is convergent we have lim S n  S
n 1 n

Consequently,

105
lim S n/  lim(c S n )  c lim S n  c  S
n n n

that is the series  cun is also convergent and its sum is equal
n 1
to c  S .
The property has been proved.
 
Theorem 2. If  u n and
n 1
v
n 1
n
are two convergent series the

series

 (u n  vn )  (u1  v1 )  (u 2  v2 )  ...  (u n  vn )  ...
n 1
formed of the sums of the corresponding terms of the given
series is also convergent and
  
 (u n  vn )   u n   vn
n 1 n 1 n 1
/ . //
Proof. Let S and S be the n th partial sums of the two given
n n

series, and let  n be the n th partial sum of the resultant series:


n
S n/  u1  u 2  ...  u n   u k
k 1
n
S n//  v1  v2  ...  vn   vk
k 1
n
 n  (u1  v1 )  (u 2  v2 )  ...  (u n  vn )   (u k  vk ) .
k 1
Then
 n  S n/  S n// .
 
Since the series  u n and  vn are convergent the limits
n 1 n 1

lim S n/ and lim S n// exist and both are finite. Assume that
n  n

lim S n/  S / and lim S n//  S // .


n n

106
Consequently, the limit lim n also exists and
n

lim n  lim(S  S )  lim S n/  lim S n//  S /  S // .


/
n
//
n
n n n n
which is what we wanted to prove.


Theorem 3. If the series  u n is convergent its any remainder
n 1

series rn   u k is also convergent and vice versa, if any
k  n 1

remainder series is convergent the series  u n itself is also
n 1
convergent.
In addition, if
 m 
S   uk , S m   uk , rm   u k
k 1 k 1 k  m 1
then

S  S m  rm
Proof. Let S n  u1  u 2  ...  u m  u m1  ...  u n be the nth

partial sum of the series  u n , and let S k( m)  u m1  ...  u mk
n 1
be the k th partial sum of the m th remainder series

rm   u k .
k  m 1
It is clear that
S n  S m  S k(m) where n  m  k .

If the series  u n is convergent and it has the sum S, i.e.
n 1
lim S n  S then at fixed m we obtain
n 

lim S n  lim( S m  S k( m) )  S m  lim S k( m)


n  n  n 

107
where k   because n  m  k   and m is fixed.
Consequently,
S  S m  lim S k( m)
k 
whence
lim S k( m)  S  S m
k 
that is the remainder series is convergent.
Now let us suppose that the remainder series is convergent and
it has the sum rm , i.e. lim S k( m)  rm . Then passing to the limit
k 

in the equality S n  S  S
m
(m )
k we get
lim S n  lim(S  S
m k
( m)
)  S m  lim S k( m)  S m  rm
n  n  k 

that is the series  u n is convergent.
n 1
The theorem has been proved.
The theorem 3 implies the following property.
Theorem 4. If a series is convergent the series obtained by
adding or deleting a finite number of terms is also convergent.

Positive Series

We now consider a series whose all terms are positive:


u1  u2  ...  un  ...
where un  0 for all n  1,2,3,.... . Such a series with positive
terms are called positive series.
Let us prove the following simple propositions.
Lemma 1. If the partial sums of a positive series are bounded
above, that is
Sn  M , M = constant
the series is convergent.
Proof. Since all the terms of the series are positive, its n th
partial sum S n is an increasing sequence:
108
Sn  Sn1 for any n
i.e. S1  S2  ...  Sn  Sn1  ...
Thus we have Sn  M and S n increases as n   . Now taking
into account that a monotone increasing and bounded above
sequence it has a limit, we get
lim S n  S  M .
n

that is the series is convergent.


The lemma has been proved.
Lemma 2. If a positive series is convergent its partial sums are
less than the sum of the series:
Sn  S .
It should be noted that if a positive series is divergent its partial
sums tend to infinity: lim S n   .
n

In this case we write  un   and say that the series is
n1
divergent to   .

Comparison Tests for Positive series

Theorem. Consider two positive series



 un  u1  u2  ...  un  ... (un  0) (1)
n1

 vn  v1  v2  ...  vn  ... (vn  0) (2)
n1
satisfying the condition that each term of series (1) does not
exceed the corresponding term of series (2), i.e.
un  vn n  1,2,...
Then:
1) If series (2) is convergent series (1) is also convergent.
2) If series (1) is divergent series (2) is also divergent.
Proof. We designate the nth partial sums of the given series as
109
n n
S n   uk ,  n   vk
k 1 k 1
It is obvious that
Sn   n .
If series (2) is convergent then  n  where   lim  n .
n

Consequently,
Sn   n  
which means that the partial sums of series (1) are bounded
above. Therefore according to the Lemma 1 series (1) is
convergent.
If series (1) is divergent its partial sums increase indefinitely:
Sn   as n   .
Since  n  Sn we also have  n   . Consequently, series (2)
is also divergent, which is what we had to prove.
Example. Let us consider the series
 1 1 1 1
 p  1  p  p  ...  p  ...
n 1 n 2 3 n
This series is convergent for p  1 and divergent for p  1 .
 1
Now if the take the series  p as the “comparison series”
n 1 n
whose convergence is already known the theorem we have
proved directly implies the following comparison test for
positive series.
Theorem. Consider a positive series

 un  u1  u2  ...  un  ... (un  0)
n1
Then:
c
1) if un  and p  1 the series is convergent
np
1
2) if  c  un and p  1 the series is divergent.
np
110
Here c is constant.

D’Alembert’s Test

Theorem (D’Alembert’s Test). Suppose we are given a


positive series

 un  u1  u2  ...  un  ... (un  0) (1)
n1
If there exists the limit
un1
lim d
n u
n
then the series is convergent for d  1 and divergent for d  1 .
Proof. Let d  1 . By the definition of a limit we have
u
lim n 1  d  1    0 N  N ( ) n  N
n  u
n

u n 1
there holds the inequality d  .
un
The latter inequality can be rewritten in the form
u
   n 1  d  
un
or
u
d    n 1  d  
un
We put d    q . Since d  1 and  is an arbitrarily
sufficiently small positive number we can choose  such that
q  d    1 . Thus we have
u n 1
 q  1 for all n  N
un
u N 1 u u N 3
i.e.  q, N  2  q,  q, ...
uN u N 1 uN 2
111
Consequently,
u N 1  q  u N
u N  2  q  u N 1  q 2u N
u N  3  q  u N  2  q 3u N
…………………….
Now let us consider two series
u N 1  u N  2  u N 3  ... (2)
u N q  u N q  u N q  ...
2 3
(3)
Since the terms of series (2) representing the remainder series
after the Nth term of the given series are smaller than the
corresponding terms of the convergent geometric series (3) (its
common ratio q is smaller than 1), the remainder series (2) is
convergent, and therefore the given series (1) itself is also
convergent.
Now let d  1 . Then there is a number N such that for all
n  N the inequalities
u n 1
 1. i.e. un1  un
un
hold. It follows that the terms of the series increase as
n   and since un  0 for all n, the necessary condition for
convergence is violated: lim u n  0 .
n 
Therefore, the series (1) is divergent
The theorem has been proved.
It should be noted that for d=1 D’Alembert’s test is
inapplicable: there exist both convergent and divergent series
for which d=1.
 1
Example. Let us consider the series  . Here
n 1 n!
1 1
u n  , u n 1  and hence
n! (n  1)!
112
u n1 n! n! 1
   .
u n (n  1)! n!(n  1) n  1
Since
u n 1 1
d  lim  lim  0 1
n 
un n   n 1
according to the D’Alembert’s Test the given series is
convergent.

Example. Let us consider the series  nn .
n 1 2
n n 1
Here we have un  n , un 1  n 1 and hence
2 2
un 1 n  1 2 n n  1
 n 1   .
un 2 n 2n
Therefore
un 1 1 n 1 1
d  lim   lim   1.
n  u 2 n  n 2
n
Consequently, the series is convergent.

Cauchy’s Root Test

Theorem (Cauchy’s Root Test). Suppose we are given a


positive series

 un  u1  u2  ...  un  ... (un  0) (1)
n 1
and let the limit
lim n un  C
n 
(finite or infinite) exists.
Then, if C  1 the series is convergent and if C  1 the series is
divergent.

113
Proof. Let C  1 . By definition of a limit we have
lim n un  C  1    0 N  N ( ) n  N n un  C  
n 
i.e.
   n un  C  
C    n un  C   for all n  N .
We put C    q . Since C  1 and  is an arbitrary sufficiently
small positive number we can choose  such that q  C    1 .
Thus we have
n u  q  1, i.e. u  q n  1 for all n  N .
n n

Now let us consider two series


uN  uN 1  uN  2  ... (2)
q N  q N 1  q N 2  ... (3)
For q  1 series (3) is convergent. According to the comparison
test the series (2) is also convergent. But if the remainder series
(2) is convergent the given series (1) itself is also convergent.
Now let C  1 . Then there is a number N such that for all
n  N the inequalities
n u  1, i.e. u  1
n n

hold, and hence lim un  0


n
Therefore since the necessary condition for convergence is
violated the given series (1) is divergent for C  1 . The
theorem has been proved.
It should be noted that for C  1 Cauchy’s root test is
inapplicable because in this case the test does not allow us to
judge upon the convergence of the series: there exist both
convergent and divergent series for which C  1 .
 1
Example. Let us consider the series  n .
n 1 n

114
1
Since lim n un  lim  0  1 the series is convergent.
n  n 
n
Example. Consider the series
1 1 1 1
 2  3  ...  n  ...
ln 2 ln 3 ln 4 ln (n  1)
1 1
Here un  n . Since lim n un  lim  0 1
ln (n  1) n  n   ln(n  1)

the series is convergent.

TEST
1. Find the sum of the series
1 1 1
  ...   ...
1 2 2  3 n(n  1)
A) 2 B) 1 C) 3 D) 0,5 E) 0,6
2. Find the sum of the series
1 1 1
  ...   ...
1 3 3  5 (2n  1)(2n  1)
1 1 1 1
A) B) C) D) E) 0
2 3 4 5
3. Find the sum of the series
1 1 1
  ...   ...
1 4 4  7 (3n  2)(3n  1)
1 1 1
A) B) C) D)1 E) 5
2 4 3
4. Find the sum of the series

115
5 13 3n  2 n
  ...   ...
6 36 6n
5 3
A) 1 B) 2 C) 3 D) E)
2 2
5. Test the series for convergence or divergence
1 1 1
  ...   ...
3! 5! (2n  1)!
6. Test the series for convergence or divergence
1 2 n
 2  ...  n  ...
2 2 2
7. Test the series for convergence or divergence
2 n 3n nn
1   ...   ...
2! 3! n!
8. Test the series for convergence or divergence
1 1 1
 2  ...  n  ...
ln 2 ln 3 ln (n  1)
9. Test the series for convergence or divergence
2 n
1 2  n 
    ...     ...
3 5  2n  1 
10. Test the series for convergence or divergence
4 n2
2 1 3 1  1
    ...  n 1    ...
3 32 2 3  n
Answers: 1)B 2)A 3)C 4)E 5)converges 6)converges
7) diverges 8) converges 9) converges 10) converges

116
Lecture 8
Alternating series
Definition. An alternating series is a series of the form


 (1) n 1un  u1  u2  u3  u4  ...
n 1

where un  0 .
In other words, an alternating series is a series whose terms are
alternately positive and negative.
We will prove a sufficient test for convergence of an
alternating series.
Theorem (Leibniz’ test). An alternating series converges if
the absolute values of its terms form a monotone decreasing
sequence tending to zero, that is, if the following two
conditions are fulfilled:
(1) un : u1  u2  u3  ...
(2) lim u n  0
n 

In other words, it means that if u n  and lim u n  0 then the


n 

series  (1) n1 un converges.
n 1
Moreover, its sum S is positive and is less than the first term,
i.e.
0  S  u1
and the absolute value of the remainder rn  S  S n of the series
is less than u n1 , i.e.
rn  S  S n  un1 .
Proof. Let us consider the sequence of partial sums S 2 m of the
series with even indices n=2m. Such a sum can be written as
S 2m  u1  u2  u3  u4  ...  u2m1  u2m
and grouping in pairs we obtain
117
S 2m  (u1  u2 )  (u3  u4 )  ...  (u2m1  u2m ) .
By the hypothesis, the expression in each bracket is positive
u1  u2  0, u3  u4  0, ..., u2m1  u2m  0 .
Since the number of these brackets increases together with m
the sequence S 2 m is positive and is increasing:
S 2 m  0 and S 2 m  .
Let us rewrite S 2 m grouping it by other way
S 2 m  u1  (u 2  u3 )  (u 4  u5 )  ...  (u 2 m  2  u 2 m 1 )  u 2 m 
 u1  (u 2  u3 )  (u 4  u5 )  ...  (u 2 m  2  u 2 m 1 )  u 2 m .
The sum of the expression in square brackets is also positive.
Therefore S 2 m  u1 , i.e. bounded above.
Thus the sequence of partial sums S 2 m of the series with even
indices is increasing and bounded above:
S 2 m  and 0  S 2 m  u1 .
Consequently, the sequence S 2 m possesses a positive limit
lim S 2 m  S  0
m 

and also, since 0  S 2 m  u1 then 0  S  u1 .


Let us now consider the sequence of partial sums S 2m1 with
odd indices:
S 2m1  S 2m  u2m1 .
Here passing to the limit as m   we obtain
lim S  lim
m 2 m1 m
(S 2 m  u2 m1 )  lim S  lim
m 2 m
u S
m 2 m1

since by the hypothesis lim u n  0 .


n 

Therefore lim S 2 m1  S , that is the sums S 2m1 with odd


m 

indices tend to the same limit S.


Thus we have that the sequences of partial sums both with even
and odd indices have a common limit S and, hence
118
lim S n  S
n 

that is, the series converges, and also S  u1 .


Now we have to prove the inequality
rn  S  S n  u n1 .
To this end it is should be noted that
S 2m  S  S 2m1 .
Indeed, on the one hand, as has already been shown S 2 m  and
lim S 2 m  S and hence and S 2m  S.
m 

On the other hand


S2m1  S2m1  (u2m  u2m1 )  S2m1
that is the sequence S2 m1 is decreasing and since
lim S 2 m1  S then S  S2 m1 .
m

Now from the inequality S 2m  S  S 2m1 it follows that


S  S2m  S2m1  S2m  u2m1
S2m1  S  S2m1  S2m  u2m
and it means that the inequality
S  Sn  un1
is valid for all n, i.e.
rn  un 1 .
Thus, the theorem has been completely proved.
Example 1. Let us consider the series
 1 1 1 1 1
 (1) n 1  1     ...  (1) n 1  ...
n 1 n 2 3 4 n
It is clear that all conditions of Leibniz’ test are fulfilled, i.e.
1 1
 and lim  0
n n n

and, hence the series converges.


2. Consider the series
119
 1 1 1 1 1
 (1) n1  1     ...  (1) n1  ...
n1 n! 2! 3! 4! n!
Here we have
1 1
 and lim  0 and hence the series converges.
n n  n!
3. Consider the series


1 1 1 1 1
 (1)
n 1
n 1

n(n  1) 1  2
2 2
 
23 3 4
2 2
 ...  (1) n1
n(n  1) 2
 ...

It is clear that
1 1 1 1 1
   ...   ...  
1 2 2
23 2
3 4 2
n(n  1) 2
n(n  1) 2
and
1
lim  0. Therefore the series is convergent.
n n( n  1) 2

Absolute and conditional convergence

Now let us consider series with terms of arbitrary signs.


Suppose we are given a series of the form

 un  u1  u2  ...  un  ... (1)
n1
whose terms can be of any sign.
Let us form the positive series

 un  u1  u2  ...  un  ... (2)
n1
whose terms are the absolute values of the terms of a given
series (1).
Definition. A series (1) is said to be absolutely convergent if
the series (2) is convergent.

120
 def 
 un - absolutely convergent   un - convergent.
n 1 n 1

Theorem. If the series  un is absolutely convergent the series
n 1
itself is also convergent
 
 un - absolutely convergent   un - convergent.
n 1 n1
Definition. A series (1) is said to be conditionally convergent
if the given series (1) is convergent whereas the series (2)
composed of the absolute values of its terms is divergent
 def 

 un - conditionally convergent   u n - convergent and


n 1 n 1

 un  divergent.
n 1
Example. Let us consider the series
 1
 (1) n1 p .
n1 n
 1
For p=1 we have  (1) n1 and, as we know, this series is
n 1 n
convergent (by Leibniz’ test) but not absolutely convergent
 1  1
since the series  (1) n1   is divergent as harmonic
n1 n n1 n
 1
series. Therefore the series  (1) n1 is conditionally
n 1 n
convergent.
The given series satisfies the conditions of the Leibniz’ test
for p  0 and therefore it converges for such p. For p  1 the
convergence will be absolute because, as we know, the series
 1
 p converges for p  1. But in the case p  1 the series
n 1 n

121
 1
 (1) n1 is conditionally convergent, i.e. it is convergent
n1 np
but not absolutely convergent.
Now we proceed to examination the fundamental distinction
between absolute and conditional convergence. It turns out that
some properties of finite sums can only be extended to
absolutely convergent series whereas conditionally convergent
series do not possess these properties.
Let us denote by

 un* (3)
n 1
the series composed of the same terms of the given series (1)
but taken in a different order.
Theorem. If the series (1) is absolutely convergent the series
(3) is also absolutely convergent and has the same sum.
 
 un - absolutely convergent   un* - absolutely convergent
n 1 n1
 
and  u   un  S .
*
n
n1 n1
In other words, this theorem states that any rearrangement of
the terms of an absolutely convergent series (1) does not affect
its sum, that is the new series (3) remains absolutely
convergent and has the same sum as before.
Conditionally convergent series do not possess this property. It
turns out if the series (1) is conditionally convergent (i.e.it is
convergent but not absolutely convergent) then for the given
any number A it is always possible to rearrange the terms of
the series (1) so that its sum becomes equal to the given
number A. This property is expressed by the following
Riemann’s Rearrangement theorem.

122

Theorem. Let  un be a conditionally convergent series, and
n 1
let A be a given real number. Then a rearrangement of the

terms of the series  un exists that converges to A.
n 1
Moreover, it is also possible to make the partial sums of the
series tend to   or   by rearranging its terms in an
appropriate manner.
The following theorems which we state without proof describe
some other important properties of absolutely convergent
series.

Theorem. If the series  un is absolutely convergent and C is a
n 1

given number then the series  Cu
n 1
n is also absolutely

convergent.
 
Theorem. If the series  un and  vn are absolutely
n 1 n 1

convergent then their sum  (un  vn ) is also absolutely
n1
convergent.
Another important property of absolutely convergent series is
related to the operation of multiplication. Let us introduce the
following definition.
Definition. Suppose we are given two convergent series

 un  U (4)
n1

 vn  V (5)
n1
The product of two convergent series (4) and (5) is the series
formed of all possible pairwise products of the terms of the
given series arranged as

123

u v
i , j 1
i j  (u1v1 )  (u1v2  u 2 v1 )  (u1v3  u 2 v2  u 3 v1 )  ... 

 (u1vn  u 2 vn 1  ...  u n 1v2  u n v1 )  ...


 
Theorem. If series  un and  vn are absolutely convergent
n 1 n 1
their product is also an absolutely convergent series and its sum
 
is equal to the product of the sums of series  un and  vn .
n 1 n 1

In other words, if  ui v j  S then S  U V .
i , j 1

TEST
1) Given the following series
 
1 1
I.  (1)n 1 II.  (1) n 1
n 1 2n  1 n 1 (2n  1)3

n 1
III.  (1)n 1
n 1 n
Determine which of them are convergent?

A) only I B) I and II C) only II D) III E) none


Test the following series for convergence and establish the
nature of the convergence:
1 1 1
2.   ...  (1)n 1  ...
ln 2 ln 3 ln( n  1)
1 1 1 1 1
3.   2  ...  (1) n 1  n  ...
2 2 2 n 2
1 1
4.  1   ...  (1) n  ...
2 n
124
1 8 n3
5.   ...  (1)n 1 n  ...
2 4 2
1 1 1
6. 1     ...
2 3 4
sin  sin 2 sin n
  ...   ...
7. 1 4 n2

 2n  1 
 n

8.  (1) n  
n 1  3n  1 

n
9.  (1) n
n 1 5n  2
1 4 7 10
10.     ...
2 5 8 11
Answers:
1) B
2) converges, but no absolutely
3) absolutely converges
4) converges, but no absolutely
5) absolutely converges
6) converges
7) absolutely converges
8) converges
9) diverges
10) diverges

125
Lecture 9
Functional Series
Now we proceed to series whose terms are not numbers but
functions.
Definition. An expression
f1 ( x)  f 2 ( x)  ...  f n ( x)  ... (1)
whose terms are functions of x, is called functional series.
Series (1) may converge for some values of x and diverge for
the other values.
Definition. A point x  x0 for which the number series

f1 ( x0 )  f 2 ( x0 )  ...  f n ( x 0 )  ...   f n ( x 0 )
n1
is convergent is called a point of convergence of series (1).
Definition. The collection of all the points of convergence is
called the domain of the convergence of the functional series.
In this case we also say that the functional series is convergent
in this domain. As a rule, the domain of convergence of a
functional series is some interval of the x-axis.
Let us denote by Sn (x) the sum of the first n terms of the series
(1):

S n ( x)  f 1 ( x)  f 2 ( x)  ...  f n ( x)  ...   f k ( x)
k 1

and by Rn (x) the remainder after n terms:



Rn ( x)  f n1 ( x)  f n2 ( x)  ...  f
k n1
k ( x) .

Each value x of the domain of convergence X is associated


with a certain value of the quantity lim S n ( x) . This quantity,
n

which is a function of x, is called the sum of the functional


series and is denoted S(x):
S ( x)  lim S n ( x) for every x  X ,
n

126
where

S ( x)  f1 ( x)  f 2 ( x)  ...  f n ( x)  ...   f n ( x) .
n1
Example: Let us consider the series

x2 x2 x2
x 
2
  ...   ...
1  x 2 (1  x 2 ) 2 (1  x 2 ) n
where x is a real number
This functional series is convergent for all x.
Indeed, for x  0 we obtain a convergent geometric series since
the common ratio is
1
r 1
1  x2
and hence, for x  0 the sum of the series is equal to

x2
S ( x)   1  x2.
1
1
1  x2
For x  0 all the terms turn into zero, and therefore the series is
convergent and the sum of the series is also zero: S (0)  0 .
Thus we have
1  x 2 for x  0
S ( x)  
 0 for x  0 .
It is clear that all the terms of the given functional series are
continuous functions and the series
converges at all point x. However y
we see that the sum S (x) of the
2
series is a discontinuous function
with a jump discontinuity at the 1
point x  0 (Fig.9.1).
-1 0 1 x
Fig 9.1
127
The considered example shows that how from the simple
continuous functions in limit processes are formed the
functions of the more complicated nature – discontinuous
functions.
To find out what conditions guarantee the possibility of
extending the properties of finite sums of functions to
functional series we need the following definition.
Let us represent the sum of the series as

S ( x)  Sn ( x)  Rn ( x)
whence
Rn ( x)  S ( x)  Sn ( x)

Definition. A convergent functional series f
n1
n ( x) is said to

be uniformly convergent in a certain domain X if for every


arbitrary small number   0 there is a positive integer
N ( ) dependent solely on  and independent of x such that for
n  N ( ) the inequality Rn (x)   is fulfilled for every x of the
domain X.

Theorem. If a functional series fn1
n ( x) formed of continuous

functions f n (x) is uniformly convergent in a domain X its sum


S (x) is also continuous function in that domain.
Let us prove the Weierstrass sufficient condition for uniform
convergence of a functional series.
Theorem (Weierstrass’ test). Let we are given two series:
a functional series

f
n1
n ( x)  f1 ( x)  f 2 ( x)  ...  f n ( x)  ...

whose terms are defined in a certain domain X;


and a positive number series
128

a
n1
n  a1  a2  ...  an  ... , an  0 .

If the inequality
f n ( x)  a n
is fulfilled for every x  X , and the number series
 

 an converges, then the functional series


n 1
f
n1
n ( x) absolutely

and uniformly converges in that domain.



Proof. Since the number series a
n 1
n converges for every given

  0 there is a positive integer N ( ) such that the inequality


a
k  n 1
k 

holds true.
It follows that for n  N ( ) and for every x  X there holds the
inequality
  
Rn ( x)   f k ( x) 
k  n 1

k  n 1
f k ( x)  a
k  n 1
k  .

And according to definition it means that the functional series


uniformly converges in the domain X, which is what we had to
prove.
Example. Using Weierstrass’ test show that the series
1 1
sin x  2 sin 2 2 x  2 sin 3 3x  ...
2 3
is uniformly convergent in (; ) .
Solution. Since
1 1 1
sin n nx  2  sin nx  2
n
2
n n n
and the series

129
1 1 1
12
 2  ...  2  ...
2 3 n
is convergent, the given series is uniformly convergent for any
value of x  (;) .

Power Series
Definition. A power series is a functional series of the form

a 0 a1 ( x  x0 )  a2 ( x  x0 ) 2  ...  an ( x  x0 ) n  ...


where a0 , a1 ,..., an ,... are constant called the coefficients of the
power series
In particular, if x0  0 the power series has the form
a 0  a1 x  a2 x 2  ...  an x n  ... . (1)
For series (1) we will prove the following fundamental theorem
on which the whole theory of power series is based.
Theorem (Abel’s theorem). 1) If power series (1) converges
at a point x 0  0 , it is absolutely convergent for every value of
x satisfying the condition x  x0 .
2) If power series (1) diverges for x  x 0 it also diverges for
any point x satisfying the condition x  x0 .

Proof. 1) Let the series a x
n0
n
n 0 be convergent.

Then its general term tends to zero: an x0n  0 as n   .


Therefore the set of values of these terms is bounded, that is
there exist a positive number M such that an x0n  M for any n.
Let us write down the series (1) as

130

a x
n 0
n
n
a 0  a1 x  a2 x 2  ...  an x n  ... 
2 n
 x  x  x
 a0  a1 x0     a2 x 02     ...  an x 0n     ...
 x0   x0   x0 
and form the series of the absolute values of its terms:
n n
 
 x 
x
 a n x   n
a x 
n 0 
 
 
n
 an x n  
n 0 n 0  x0  n 0 x0
(2)
2 n
x x x
a 0  a1 x0   a2 x02   ...  an x 0n   ...
x0 x0 x0
Since by the inequality proved an x0n  M for any n it follows
that every term of the latter series is less than the
x
corresponding term of the geometric series with ratio :
x0
2 n
x x x
M M  M   ...  M   ... (3)
x0 x0 x0
x
For x  x0 we have  1, and the geometric series (3) is
x0
convergent. Therefore series (2) that is the series of the
absolute values of the terms of the given power series is also
convergent, and hence series (1) is itself absolutely convergent.
The first part of the theorem has been proved.
Now let us proceed to proving the second part of the theorem.
Suppose the given series (1) diverges at some point x0 . Then
series (1) also diverges for any point x satisfying the condition
x  x0 .
Indeed, if the series were convergent for such a value of x, the
first part of Abel’s theorem would imply its convergence for
131
the values of the argument smaller than x in their absolute
values and, in particular, for x  x0 , which contradicts the
hypothesis. The theorem has been completely proved.
Let us proceed to investigate the domain of convergence of
series (1). There are three possible cases here:
(1) The domain of convergence consists of one point x  0 .
In other words, the series is divergent for all the points except
x=0.
Such a case can be demonstrated by the following example.
Example. Consider the series

n x
n0
n n
 1  x  22 x 2  33 x3  ...  n n x n  ...

Indeed, for any fixed x  0 we have nx  1 beginning with


sufficiently large value of n whence follows the inequality
n n x n  1 indicating that the general term of the series does not

tend to zero. That is n n x n  1  lim n n x n  0   n n x n is
n
n0
divergent.

Thus, the given series 
n0
n n x n is divergent for all the points

except x=0.
(2) The domain of convergence consists of all the points of
the x-axis.
In other words, the series is convergent for all the points
x  (; ) .
Definition. A power series which is convergent at any point
x  (; ) is called permanently convergent.
Let us consider an example of such a series.
Example. Consider the series

132

xn x2 x3 xn
1  n
 1  x    ...   ...
n 1 n 2 2 33 nn

x
For any x we have  1 beginning with a sufficiently large n.
n
Since
n 1 n 1 n2 n2
x x x x
 ,  , etc.
n 1 n n2 n
the absolute values of the terms of the series become,
beginning with the number n, less than the terms of the
x
convergent geometric series with common ratio 1.
n
Consequently, the given series is convergent for any x.
(3) The domain of convergence consists of more than one
point, and there are points of the axis Ox not belonging to
the domain of convergence.
Example. Consider the series

x
n0
n
 1  x  x 2  ...  x n  ... .

The given series is the geometric series with common ratio x.


This series is convergent for x  1 and divergent for x  1 . In
this example the Ox axis contains both points of convergence
and points of divergence of the series.
Abel’s theorem implies that all the points of convergence are
not farther from the origin than any point of divergence. It is
also clear that the points of convergence entirely cover a whole
interval with centre at the origin.
Thus, we can say that for every power series possessing both
points of convergence and points of divergence there is a

133
positive number R such that the series converges absolutely for
x  R and it diverges for x  R .
As for the values x  R and x   R of the argument, there can
be various possibilities in these cases: the series may converge
at both points or at one of them or at neither. Moreover, the
convergence at these points may be absolute or conditional.
Definition. The number R such that power series (1) is
convergent for all x satisfying the condition x  R and
divergent for all x satisfying the inequality x  R is called the
radius of convergence of the power series.
The interval ( R, R) is referred to as the interval of
convergence.
Now let us present a method for determining the radius of
convergence of a power series. To this end, we consider the
series

a x
n 0
n
n
 a 0  a1 x  a2 x 2  ...  an x n  ... 
(4)
 a0  a1  x  a2  x2  ...  an xn  ...
2 n

composed of the absolute values of the terms of series (1). Let


us apply D’Alembert’s test to the positive series (4). Suppose
that the limit of the ratio of two successive terms of this series
exists and that this limit has been found.
u a x n 1 a
lim n 1  lim n 1 n  lim n 1  x  L x .
n  u n  an x n   an
n

1
Then, if L x  1 , that is x  the series is convergent, now if
L
1
L x  1 , that is x  the series is divergent. It follows that
L
1 a
R   lim n
L n   an 1
134
is the radius of convergence of power series (1).
It may turn out that L  0 or L   .
If L  0 for all x  0 then R   . This means that series (1) is
everywhere convergent, that is it converges for all the values of
x  (;) , and hence (; ) is the interval of
convergence.
If L   for all x  0 then R  0 . This means that series (1) is
everywhere divergent (except at the point x  0 ).
Similarly we can determine the radius of convergence by using
Cauchy’s test. We have
lim n un  lim n an x n  x  lim n an  x  L
n n n

1
x L 1 x    an x n is convergent.
L n0

1
x  L  1  x    an x n is divergent.
L n 0
It follows that
1
R
lim n an
n
is the radius of convergence of power series (1).
Let us consider some examples.
(1) Find the radius of convergence of the series

xn 1 1 1
n0 n!
 1  x  x 2  x 3  ...  x n  ...
2! 3! n!
Solution. We have
n 1
u n1 x  n! x
  0
(n  1)! x n 1
n
un
for any value of x. Hence, the series is convergent for all x, that
is R   .
(2) Determine the radius of convergence of the series
135

xn x2 x3 xn
1  1 x    ...   ...
n 1 n 2 3 n
Solution. We have
u n1 x n1  n n
 x x.
un (n  1) x n
n 1
Consequently, by D’Alembert’s test, the series converges if
x  1 and diverges if x  1 . Therefore R  1 .
(3) Determine the radius of convergence of the series

x 2 n 1 x3 x5 x 2 n 1
n 1
( 1) n 1

(2n  1) 2
 x  
32 52
 ...  ( 1) n 1

(2n  1) 2
 ...

Solution. Here we obtain


x 2 n1  (2n  1) 2 2  2n  1 
2
u n1
 x   x
2

un (2n  1)  x2 2 n 1
 2n  1 
For x  1 the series is convergent and for x  1 it is divergent.
Therefore, the radius of convergence is equal to 1, R  1 .

TEST
1) Find the sum of the series
1. 1  x 2  x 4  x 6  ...
1
A) S ( x)  , x 1 B) S ( x)  1 C) S ( x)  x
1  x2
D) S ( x)  x2 E)0
2. Find the sum of the series
1  2 x  4 x 2  8 x3  ...

136
1 1
A) S ( x)  x2 B) S ( x)  ,x C) S ( x)  1
2x  1 2
D) S ( x)  0 E) S ( x)  2 x
3. Find the sum of the series
1 ( x  3) ( x  3)2
  ...   ...
3 9 27
1 1 1
A) S ( x)  B) S ( x)  C) S ( x)  , 0  x  6
2 3x x
D) S ( x)  0 E) S ( x)  5
4. Test the functional series

4 x 1 4 x  1 4 x 
2 3

       ...
7x  2 3  7x  2  5  7x  2 
for convergence at the points x  0, x  1
Find the interval of convergence of following power series
5. 10 x  100 x 2  ...  10n x n  ...

A) (0,2) B) 0,3 C) (1,1)

 1 1
D) (10,10) E)   , 
 10 10 
x2 n 1 x
n
6. x  ...  (1)  ...
2 n
A) (1,1] B) (0,1) C) (10,9) D) (1,3) E) (8,8]

x2 xn
7. x  ...   ...
20 n 10n 1
137
A) (1,1) B) [10,10) C) (9,9) D) (20,30) E) (2,2]

8. 1  x  ...  n! x n  ...
A) x  0 B) x  1 C) x  5 D) x  7 E) x  1

9. 1  3x  ...  (n  1)  3n 1  xn 1  ...
 1 1
A) (0,1) B) (3,3) C)   , 
 3 3
D) (2,2) E) (1,1)

x x2 xn
10.   ...   ...
1 2 2  3 n(n  1)

A) [0,1) B) [2,2] C)  7,5 D) [1,1] E) (5,4)


Answers:
1)A 2)B 3)C

4) at the point x  0 the series diverges


at the point x  1 the series converges

5)E 6) A 7) B 8)A 9)C 10)D

138
Lecture 10
Differential Equations
General Notions
Definition. A differential equation is an equation that
relates some function of one or more variables with its
derivatives.
The problem of forming and solving these equations is
widely encountered in physics and engineering. In biology and
economics, differential equations are used to model the
behavior of complex systems.
Definition. The process of solving a differential equation
is called integration of the differential equation. The
simplest ordinary differential equations can be integrated
directly by finding antiderivatives. These equations have the
form
d nx
 G(t )
dt n
where the derivative of x  x(t ) can be of any order, and the
right-hand-side may depend only on the independent variable t.
As an example, consider a mass falling under the influence of
constant gravity, such as approximately found on the Earth’s
surface. Newton’s law, F  ma, results in the equation
d 2x
m  mg
dt 2
where x is the height of the object above the ground, m is the
mass of the object, and g  9,8 meter/sec2 is the constant
gravitational acceleration. As Galileo suggested, the mass
cancels from the equation, and
d 2x
 g
dt 2
Here, the right-hand-side of the equation is a constant.
The first integration, obtained by antidifferentiation, yields

139
dx
 A  gt ,
dt
with A the first constant of integration, and the second
integration yields
1
x  B  At  gt 2 ,
2
with B the second constant of integration.
The two constant of integration A and B can then be
determined from the initial conditions. If we know that the
initial height of the mass is x0 , and the initial velocity is 0 ,
then the initial conditions are
dx
x(0)  x0 , (0)  0
dt
Substitution of these initial conditions into equations for
dx
and x allows us to solve for A and B. The unique solution
dt
that satisfies both the ordinary differential equation and the
initial conditions is given by
1
x(t )  x0  0t  gt 2
2

Basic definitions
Let us now introduce some definitions important for
further understanding. A differential equation is usually taken
in a form which connects an argument (or several arguments)
and an unknown function (or several unknown functions) with
its derivatives. If the unknown function depends on one
variable the differential equation is called ordinary.
Otherwise the equation is called a partial differential
equation. The highest order of the derivative of the unknown
function entering into an equation is called the order of the
differential equation.

140
The general form of a differential equation of the nth
order is
F ( x, y, y / , y // ,..., y ( n) )  0, (1)
where y  y(x) is the sought-for function. A function is called
a solution of a differential equation if it reduces the equation to
an identity when substituted into the equation.
Even the simplest examples indicate that a differential
equation has infinitely many solutions. For instance, taking a
simple equation of the form
y /  x 2 , y  y(x) (2)
we immediately find, by integrating, that
x3
y  C (3)
3
This is the general solution of equation (2).
It contains an arbitrary constant C and is the set of
solutions containing all solutions of the equation. Making the
arbitrary constant assume concrete numerical values we obtain
particular solutions of equation (2):
x3 x3 x3 2
y , y   6, y  
3 3 3 3
Similarly, the general solution of an equation of the form (1)
also contains n arbitrary constants, i.e. it has the form
y  y( x, c1 , c2 ,..., cn ) (4)
We often obtain the general solution in an implicit form
( x, y, c1 , c2 ,..., cn )  0 (5)
Relations (4) and (5) are also called general integrals of
equation (1). Particular solution can be obtained from (4) or
(5) if we make each of the arbitrary constants c1 , c2 ,..., cn take
on a certain concrete numerical value. The graph of every
particular solution is called an integral curve of the
differential equation.

141
To isolate a unique particular solution from the general
solution we must set some additional conditions. Such
conditions are often taken as so-called initial conditions. In the
general case of an equation of form (1) the initial conditions
are
y  y0 , y /  ( y / )0 , ..., y ( n 1)  ( y ( n 1) )0 for x  x0 (6)
Condition (6) for an equation of the first order of from
(2) means that for a certain value x  x0 we must assign a
value y  y0 . For instance, let it be necessary to isolate a
solution for which y(1)  2 . Then (3) implies

1 5
2
 C i.e. C  .
3 3
Hence, the sought – for particular solution has the form
x2  5
y
3
Definition. The problem of finding a particular solution
of a differential equation when certain initial conditions are
given is called the Cauchy problem (initial-value problem)

First – Order Differential Equations

The general first-order differential equation for the


function y  y (x) is written as
dy
 f ( x, y ) , (1)
dx
where f ( x, y ) can be any function of the independent variable
x and the dependent variable y.

142
Separable equations
dy
1) The differential equation of the form  f ( x, y ) is
dx
called separable, if f ( x, y )  h( x) g ( y ) that is
dy
 h( x ) g ( y ) (2)
dx
Let us rewrite the equation (2) as
dy
 h( x)dx
g ( y)
and then, integrating,
1
 g ( y) dy   h( x)dx  C
we obtain
G ( y )  H ( x)  C
dy y 2  1
Example. Solve 
dx x
Separating the variables we obtain
dy dx

y 1 x
2

using the techniques of integration of rational functions, we


get
dy 1 y 1
 y 2  1  2 ln y  1 ,
which implies

1 y 1
ln  ln x  C
2 y 1
2) The equation
M 1 ( x) N1 ( y )dx  M 2 ( x) N 2 ( y )dy  0 (3)

143
is also an equation with variables separable. If we divide
both side of equation (3) by N1 ( y ) M 2 ( x) , we get

M 1 ( x) N1 ( y ) M ( x) N 2 ( y )
dx  2 dy  0
N1 ( y ) M 2 ( x) N1 ( y) M 2 ( x)
or
M 1 ( x) N ( y)
dx  2 dy  0
M 2 ( x) N1 ( y )
We can construct the general solution of this equation:

M 1 ( x) N ( y)
M 2 ( x)
dx   2 dy  C
N1 ( y )

Homogeneous Equations

The differential equation

dy
 f ( x, y ) (1)
dx
is homogeneous if the function f ( x, y ) is homogeneous, that
1
is f (tx, ty )  f ( x, y) for any number t. Let us substitute t 
x
in this identity, then we have
 y
f ( x, y)  f 1; 
 x
The equation (1) becomes
dy  y
 f 1;  (2)
dx  x
This equation is easily integrated by means of the
substitution

144
y dy du
 u, y  ux, u x (3)
x dx dx
where u  ux  is a new unknown function which replaces y.
Substituting (3) into (2) we derive
dy
u   x  f (1, u )
dx
which is a separable equation

xdu du dx
 f (1, u )  u or 
dx f (1, u )  u x
Integrating, we find

du dx
 f (1, u )  u
   C.
x
Once solved, go back to the old variable y via the
equation y  ux .
A first – order ordinary differential equation in the form
M ( x, y )dx  N ( x, y )dy  0
is a homogeneous type if both functions M ( x, y ) and
N ( x, y) are homogeneous functions of the same degree n. That
is, multiplying each variable by a parameter  , we find

M (x, y)dx  n M ( x, y) and N (x, y)dx  n N ( x, y)


Thus
M (x, y ) M ( x, y )

N (x, y ) N ( x, y )
Example . Solve the equation

dy  2 x  5 y

dx 2x  y
Solution: it is easy to check, that
145
 2x  5 y
f ( x, y )  is homogeneous.
2x  y
y
we substitute  u , then
x
y  ux, y /  u  xu /
 2 x  5ux
and hence u  xu /  or
2 x  ux
 2  5u
u  xu / 
2u
which can be rewritten as
1   2  5u 
u/   u .
x 2u 
This is a separable equation
du 1  u 2  3u  2  u2 dx
    or  du  .
dx x  u2  (u  2)(u  1) x
Integrating, the latter we find

4du 3du dx
     ln C
u2 u 1 x
 4 ln u  2  3 ln u  1  ln x  ln C .
u 1
3

 ln x C (u  1) 3
ln or  xC
u2
4
(u  2) 4
Back to the function y, we get
3 4
y  y 
  1  xC   2  ,
x  x 
( y  x)  C ( y  2 x) 4
3

This is the general solution of the original differential


equation.

146
Linear Equations

A first order linear differential equation is an equation


of the form
y /  P( x) y  Q( x) , (1)
where P and Q are functions of x. If the equation is written in
this form it is called standard form. The equation is called
first order because it only involves the function y and first
derivatives of y.
Equation (1) is called homogeneous linear equation if
Q(x) is identically equal to zero.
Otherwise the equation is called non-homogeneous. To
solve the general non-homogeneous equation of the form (1)
we first investigate the associated homogeneous equation
y /  P( x ) y  0 (2)
The variables in equation (2) are easily separated:
dy dy
 P( x) y  0    P( x)dx
dx y
ln y    P( x)dx  lnC
It follows that the general equation of (2) has the
following form
y Ce 
 P ( x ) dx
.
We will use method of variation of parameters to find general
solution of non-homogeneous equation (1). The idea of method
of variation of parameters is to look for a particulars solution
such as
y  z ( x) e 
 P ( x ) dx
(3).
where z(x) is function. From this, the method got its name.
Differentiating (3), we find
y /  z / ( x) e   z( x) P( x)e 
 P ( x ) dx  P ( x ) dx
(4)
147
Substituting (3) and (4) into equation (1) we receive
z / ( x) e   z ( x) P( x)e   P( x) z ( x)e 
 P ( x ) dx  P ( x ) dx  P ( x ) dx
 Q( x )
whence

z / ( x) e 
 P ( x ) dx
 Q( x )
or

z / ( x)  Q( x)e 
P ( x ) dx

Integrating, find
z ( x )   Q ( x )e 
P ( x ) dx
dx  C1
Substituting this expression in formula (3), we get
general solution of linear equation (1):
y    Q( x)e  dx  C1 e 
P ( x ) dx  P ( x ) dx

 
Example. Solve y  y  e x /

Let us consider associated homogeneous equation


y/  y  0
It is separable equation. Thus
dy dy dy
y   dx     dx  ln C
dx y y

hence
ln y  x  ln C
or
y
ln  x  y  Ce x .
C
Let us substitute C(x) into C
Then y  C ( x)e x
Differentiating this expression, we find

148
y /  C / ( x)e x  C ( x)e x .
Substituting y / and y into given equation, we get
C / ( x)e x  C ( x)e x  C ( x)e x  e x
C / ( x)e x  e x or C / ( x)  1 .
Integrating, we find

C ( x)   dx C1  C ( x)  x  C1 .
So
y  ( x  C1 )e x
It is general solution of given equation.

Bernoulli Equation

Definition. Differential equation in the form


y /  P( x) y  Q( x) y n , (1)
where P(x) and Q(x) are continuous functions and n is
real number is called Bernoulli Equation.
First notice that if n  0 or n  1 then the equation is
linear and we already know now to solve it in these cases.
In order to solve equation (1), first let us divide the
equation by y n to get,
y  n y /  P( x) y1n  Q( x) (2)
1n
Using the substitution z  y (3) we convert this into a
differential equation in terms of z. Taking the derivative, we
get
z /  (1  n) y  n y / (4)
Now, plugging our substitutions (3) and (4) into the
differential equation (2) gives
1 /
z  P( x) z  Q( x)
1 n
149
This is a linear differential equation that we can solve
for z and once we have this in hand we can also get the solution
to the original differential equation by plugging z back into our
substitution and solving for y.
Example . Solve
4
y /  y  x3 y 2 (5)
x
Solution. The first thing that we need to do is get this
into the “proper” form and that means dividing everything by
y 2 . Doing this gives,
4
y 2 y /  y 1  x 3
x
The substitution and derivative that we will need here is:

z  y 1 , z /   y 2 y / .
With this substitution the differential equation becomes,

4
 z  z  x3 .
x
So, as noted this is linear differential equation that we
know how to solve. We have
4
z  z  x3 . (6)
x
Let us consider the corresponding homogeneous equation

4
z  z0
x
which is separable equation. Thus
dz 4 dz dx dz dx
 z  4    4  ln C .
dx x z x z x
Hence ln z  4 ln x  ln C  ln z  ln Cx 4
 z  Cx 4
150
Let us substitute C(x) instead of C
Then z  C ( x) x 4
Differentiating this expression, we find
z  C / ( x) x 4  4C ( x) x3
/

Substituting z / and z into (6), we get

4
C / ( x) x 4  4C ( x) x 3  C ( x)  x 4   x 3
x

1
C / ( x) x 4   x 3  C / ( x)    C ( x)   ln x  C1
x
so
z  ( ln x  C1 ) x 4
To get the solution in terms of y all we need to do is plug the
substitution back in. Doing this gives
y 1  x 4 (C1  ln x)

Exact First – Order Equations

Definition. The equation M ( x, y )dx  N ( x, y)dy  0 (1)


is an exact differential equation if there exists a function f of
two variables x and y having continuous partial derivatives
such that
f ( x, y ) f ( x, y)
 M ( x, y ) and  N ( x, y) (2)
x y
The general solution of the equation is
f ( x, y )  C . (3)
We know that if f has continuous second partials, then

M  2 f 2 f N
  
y xy yx x
151
This suggests the following test for exactness

Theorem. (Test for Exactness)


Let M and N have continuous partial derivatives. The
differential equation M ( x, y )dx  N ( x, y)dy  0 is exact if and
only if
M N
 (4)
y x
Let us integrate the first equation (2) with respect to the
variable x, we get
x
f ( x, y )   M ( x, y )dx   ( y )
x0

where x0 – abscissa of any point from domain of solution.

The function  ( y ) should be there, since in our integration, we


assumed that the variable y is constant.
Differentiating the last equation, with respect to y, we find

f x M
 dx   / ( y )  N ( x, y )
y x y 0

M N
But, since  , then
y dx
N
x

 x dx   ( y)  N ( x, y) .i.e.
/

x0

N ( x , y ) x   / ( y )  N ( x, y )
x

or
N ( x, y)  N ( x0 , y)   / ( y)  N ( x, y)

Hence  / ( y)  N ( x0 , y) , and it follows that

152
y

 ( y)   N ( x0 , y )dy  C1
y0

Therefore,
x y

f ( x, y )   M ( x, y )dx   N ( x0 , y )dy C1


x0 y0

here p( x0 , y0 ) is point, in neighborhood of which there is


solution of equation (1).
And the general solution is
x y

 M ( x, y)dx   N ( x0 , y)dy C
x0 y0

Example. Solve the differential equation


(2 xy  3x 2 )dx  ( x 2  2 y)dy  0
Solution. The given differential equation is exact
because
M 

y y

2 xy  3x 2  2 x  
N  2

x x
x  2y  
The general solution, f ( x, y )  C , is given by

f ( x, y )   M ( x, y )dx   (2 xy  3x 2 )dx  x 2 y  x 3   ( y )
f x, y   2
 x y  x 3    y   x 2    y   x 2  2 y  N x, y 
y y

Thus,  / ( y )  2 y , and it follows that


 ( y )   y 2  C1 .
Therefore, f ( x, y)  x 2 y  x3  y 2  C1
and the general solution is x 2 y  x3  y 2  C .

153
TEST
1.Find the general solution to the separable equation
xyy /  1  x 2
A) x 2  y 2  lnCx2 B) 1  y 2  C (1  x 2 )
C) y  Cx D) Cx  y  1 E) y  x 2  C
2. Find the general solution to the separable equation
1  2x
yy /  .
y
A) x 2  lnCy 2 B) y  3 C  3x  3x 2
y 1
C) y  C sin x D) Cx  E) y  e x  C .
y
3. Find the general solution to the separable equation:
y /  10x y .
A) 10 x  y  C B) 10  y  x  C
C) 10 x  10  y  C D) y  C sin x E) y  1  x 2  C .
4. Find the general solution to the homogeneous equation:
2 xy
y/  2 .
x  y2
y
x
A) ln y  C B) e x  Cy
y
C) y  Cx 2 D) x 2  y 2  Cy E) y  Ce2 x .
5. Find the general solution to the homogeneous equation:
y 2  x 2 y /  xyy / .
A) y  Ce x B) y  Cx 2
y

C) x  y  Cy
2 2
D) y  e E) e  Cy .
 Cx x

6. Find the general solution to the linear equation:


y /  2 xy  xe x .
2

154
2 x2 
A) y  e  x  C   B) y  Ce  x
2

 2
x2
C) y  5x 2  C D) y  C  E) y  Cxe x .
2
7. Find the particular solution to the linear equation with initial
1
condition: y /  y tan x  ; y0  0
cos x
A) y  cos x B) y  sin x
x
C) y  D) y  x E) y  x  5 .
cos x
8.Find the particular solution to the linear equation with initial
condition: xy /  y  e x  0 ; y1  1
ex (e x  1  e)
A) y  B) y  C) y  x
x x
ex
D) y  E) y  0
5
9.Find the general solution to the exact equation
e y dx  ( xe y  2 y)dy  0 ;
A) xe y  C B) y  Cx C) xe y  y 2  C
D) y 2  Cx 2 E) e y  x
10.Find the general solution to the exact equation.
3x 2e y dx  ( x3e y  1)dy  0 ;
A) xe y  C B) xe y  y 3  C C) y  Cx 2
D) y  Ce y E) x3e y  y  C

Answers: 1.A 2.B 3.C 4.D 5.E 6.A 7.C 8.B 9.C
10.E

155
Lecture 11
Differential equations of second – order.
Reduction of order
Some second-order equations can be reduced to first-
order equations, rendering then susceptible to the simple
methods of solving equations of the first order. The following
are three types of such second-order equations:
Type 1: Let
y //  f ( x) (1)
Integrating this equation (1), we obtain
y /   f ( x)dx  C1
Integrating this equation once again we get
y   dx  f ( x)dx  C1 x  C2
where C1  constant , C2  constant
Example 1. Solve the differential equation
y //  x  sin x
Solution. This equation relates to the first type
x2
y   ( x  sin x)dx  C1   cos x  C1
/

2
Then
 x2 
y     cos x  C1 dx  C2 
 2 
3
x
 sin x  C1 x  C2
6
Type 2: Second-order equations with the dependent
variable mussing.
This is an equation of the type
F ( x, y / , y // )  0 (2)

156
The dependent variable y does not explicitly appear in the
equation. This type of second-order equation is easily reduced
to a first-order equation by the transformation
y/  p .
This substitution obviously implies y //  p , and the
original equation becomes a first-order equation for p
F ( x, p, p / )  0
Suppose we have managed to integrate this equation and
have obtained its general solution p   ( x, C1 ) . Then we
have
y /   ( x, C1 )
and therefore the general solution of equation (2) is thus
obtained:
y    ( x, C1 )dx  C2
Example 2. Solve the differential equation
xy //  y /
Solution. Since the dependent variable y is missing, let
y  p and y //  p / .
/

These substitutions transform the given second-order


equation into the first-order equation
xp /  p
It is a separable equation. Separating the variables, we
obtain
dp dx

p x
Integrating, we have

dp dx
 p

x
Then ln p  ln x  ln C1 or ln p  ln xC1
157
and hence
p  xC1
x2
So y  C1 x  y   C1 x  C2  C1  C2
/

2
Type 3. Second-order nonlinear equations with the
independent variable missing.
Let it be of the form
F ( y, y / , y // )  0 (3)
The independent variable x does not explicitly appear in
the equation.
The method for reducing the order of these second-order
equations begins with the same substitution as for Type 2
equations, namely, replacing y/ as p. But instead of simply
writing y// as p/, the trick here is to express y// in terms of a first
derivative with respect to y.
Therefore we write
d  dy  dp dp dy dp
y //     p
dx  dx  dx dy dx dy
Now we deduce from equation (3) the equation
 dp 
F  y, p, p   0
 dy 

which is a first-order equation.


If we manage to integrate it and to find its general
solution p   ( y, C1 ) then we have
dy
  ( y , C1 )
dx
and therefore the general solution of equation (3) can be
directly written in the form

158
dy
  ( y, C )  x  C 2
.
1

Example 3. Solve the differential equation


1  ( y / ) 2  2 yy //
dp
Solution. The substitutions y /  p and y //  p
dy
transform this second-order equation for y into the following
first-order equation for p:
dp
1  p 2  2 yp
dy
Separating the variables, we obtain
2 pdp dy

1  p2 y
Integrating, we have
2 pdp dy
1 p 2

y
 ln C1 .
It follows that
ln(1  p 2 )  ln y  ln C1 or 1  p 2  C1 y
 p 2  C1 y  1  p  C1 y  1 .
Now, since p  y / , this last result becomes
dy
 C1 y  1 .
dx
Separating the variables, we get
dy
 C1 y  1   dx  C2 .
2
So C1 y  1  x  C2
C1

159
4C1 y  1
or 2
 ( x  C2 ) 2
C1

.
Second – Order Linear Differential Equations

A second-order linear differential equation has the form


d2y dy
2
 p( x)  q ( x) y  f ( x) , (1)
dx dx

where p( x), q( x) and f (x) are continuous functions. If


f ( x)  0 equation (1) is nonhomogeneous. If f ( x)  0

d2y dy
2
 p( x)  q( x) y  0 (2)
dx dx
equation (2) is called homogeneous linear equation.

Theorem 1. If y1 ( x ) and y2 ( x ) are both solutions of the


linear homogeneous equation (2) and C1 and C2 are any
constants, then the linear combination

y( x)  C1 y1 ( x)  C2 y2 ( x)

is also a solution of equation (2)


Proof. Since y1 and y 2 are solution of (2), we have
y1//  p( x) y1/  q( x) y1  0
and
y2//  p( x) y2/  q( x) y2  0
Therefore, using the basic rules for differentiation, we
have

160
y //  py /  qy  (C1 y1  C 2 y 2 ) //  p( x)(C1 y1  C 2 y 2 ) / 
q( x)(C1 y1  C2 y2 )  (C1 y1//  C2 y2// )  p( x)(C1 y1/  C2 y2/ ) 
 q( x)(C1 y1  C2 y2 )  C1 y1//  p( x) y1/  q( x) y1  
 C2 y 2//  p( x) y 2/  q( x) y 2   C1  0  C2  0  0
Thus, y  C1 y1  C2 y2 is a solution of equation (2). The other
fact we need is given by the following theorem. It says that the
general solution is a linear combination of two linearly
independent solutions y1 and y 2 .
This means that neither y1 nor y 2 is a constant multiple of
the other. For instance, the functions f ( x)  x 2 and g ( x)  5x 2
are linearly dependent, but f ( x)  e x and g ( x)  xe x are
linearly independent.
Theorem 2. If y1 and y 2 are linearly independent
solutions of equation (2), then the general solution is given by
y ( x)  C1 y1 ( x)  C2 y2 ( x)
where C1 and C 2 are arbitrary constants.
Definition. Let y1 ( x ) and y 2 ( x) be two differentiable
functions. The Wronskian W ( y1 , y2 ) , associated to y1 and y 2 ,
is the function
y ( x) y2 ( x)
W ( y1 , y2 )  1/ /
 y1 ( x) y2/  y1/ ( x) y2 ( x) (3)
y1 ( x) y2 ( x)
Theorem 3. If y1 ( x ) and y 2 ( x) are linearly dependent,
then Wronskian W ( y1 , y 2 )  0 .
Theorem 4. If y1 ( x ) and y 2 ( x) are linearly independent
solution of equation (2), then W ( y1 , y2 )  0 .
Theorem 5. If y1 ( x ) and y 2 ( x) are two solutions of
equation (2), then

161
 x 
W ( y1 , y2 )( x)  W ( y1 , y2 )( x0 ) exp   p(t )dt  .
 x 0 
Proof. Since y1 and y 2 are solutions of (2) we have
y1//  p( x) y1/  q( x) y1  0 (4)
y2//  p( x) y2/  q( x) y2  0 (5)
Now we multiply (4) by y 2 and multiply (5) by y1 . Subtract the
resulting two equations to obtain.
y1 y2//  y1// y2  p( x)( y1 y2/  y1/ y2 )  0 (6)
Recall the definition (3) and observe that
dW
 y1 y2//  y1// y2
dx
Hence (6) is the equation

dW
 p( x)W ( x)  0 (7)
dx
Separating the variables, we get
dW
  p( x)dx
W

Integrating, we have
x

lnW    p( x)dx  ln C
x0

or
x
W
ln    p( x)dx
C x 0

So

162
 x 
W ( y1 , y2 )x   C exp   p( x)dx 
 x 0 
where C  W ( y1 , y2 )( x0 )

Second-order linear homogeneous differential equation


with constant coefficients

We now study solutions of the homogeneous, constant


coefficient differential equation, written as
ay //  by /  cy  0 (1)
where a,b, and c are constants and a  0 . We “guess” a
solution of the form
y( x)  e kx (where k is a constant)
y /  kekx , y //  k 2 e kx
If we substitute these expressions into equation (1), we see that
y  e kx is a solution if
ak 2e kx  bkekx  cekx  0
or
(ak 2  bk  c)e kx  0
But e kx is never 0, since e kx  0 . Thus y  e kx is a
solution of equation (1) if k is a root of the equation
ak 2  bk  c  0 (2)
Equation (2) is called the auxiliary equation
(characteristic equation) of the differential equation (1).
It should be noted that it is algebraic equation that is
obtained from the differential equation by replacing y// by k2, y/
by k and y by 1. Sometimes the roots k1 and k2 of characteristic
equation can be found by factoring. In other cases they are
found by using the quadratic formula:

163
 b  b 2  4ac  b  b 2  4ac
k1  , k2  (3)
2a 2a
We distinguish three cases according to the sign of the
discriminant b 2  4ac
Case 1. b 2  4ac  0
In this case the roots k1 and k2 of the auxiliary equation are real
and distinct, so y1  e k1x and y2  e k2 x are two linearly
independent solution of equation (1). (Note that e k2 x is not a
constant multiple of e k1x ). Therefore, by theorem 2, we have
the following fact:
If the roots k1 and k2 of the auxiliary equation
ak 2  bk  c  0 are real and unequal, then the general
solution of
ay //  by /  cy  0 is
y  C1e k1x  C2e k2 x (4)
Example 1. Solve the equation y //  y /  6 y  0
Solution. The auxiliary equation is
k 2  k  6  (k  2)(k  3)  0
whose roots are k1=2, k2 =-3. Therefore by (4) the general
solution of the given differential equation is
y  C1e 2 x  C2e 3 x
Case 2. b 2  4ac  0
In this case k1=k2; that is, the roots of the auxiliary equation are
real and equal. Let us denote by k the common value of k1 and
k2. Then, from formula (3), we have
b
k so 2ak  b  0 (5)
2a
We know that y1  e kx is one solution of equation (1). We now
verify that y2  xekx is also a solution:

164
ay2//  by2/  cy2  a(2kekx  k 2 xekx )  b(e kx  kxekx )  cxekx 
 (2ak  b)e kx  (ak 2  bk  c) xekx  0  (e kx )  0  ( xekx )  0
The first term is 0 by equation (5); the second term is 0 because
k is a root of the auxiliary equation. Since y1  e kx and
y2  xekx are linearly independent solutions. Theorem 2
provides us with the general solution. If the auxiliary
equation ak 2  bk  c  0 has only one real root k, then the
general solution of ay  by  cy  0 is
// /

y  C1e kx  C2 xekx (6)


Example 2. Solve the equation y //  2 y /  y  0
Solution. The auxiliary equation k 2  2k  1  0 can be
factored as (k  1) 2  0
So the only root is k=1. By (6) the general solution is
y  C1e x  C2 xe x
Case 3. (b 2  4ac)  0
In this case the roots k1 and k 2 of the auxiliary equation
are complex numbers. We can write
k1    i , k 2    i ,
where  and  are real numbers.
 b 4ac  b 2 
In fact    ,   
 2a 2a 
Then, using Euler’s equation
ei  cos  i sin 
we write the solution of the differential equation as

165
y  C1e k x  C2 e k x  C1e ( i ) x  C2 e ( i ) x 
1 2

 C1ex (cos x  i sin x)  C2 ex (cos x  i sin x) 


 ex (C1  C2 ) cos x  i (C1  C2 ) sin x 
~ ~
 ex C1 cos x  C2 sin x 
~ ~
where C1  C1  C2 , C1  i(C1  C2 )
This gives all solutions (real or complex) of the
differential equation.
~ ~
The solutions are real when the constants C1 and C2 are
real. We summarize the discussion as follows.
If the roots of the auxiliary equation ak 2  bx  c  0
are the complex numbers k1    i , k 2    i , then
the general solution of ay //  by /  c  0 is
y  ex C1 cos x  C2 sin x (7)
Example 3. Solve the equation y //  4 y /  13y  0
Solution. The auxiliary equation is
k 2  4k  13  0
By the quadratic formula, the roots are
k1, 2  2  3i
By (11) the general solution of the differential equation
is
y  e 2 x (C1 cos3x  C2 sin 3x)

Initial-Value Problem
An initial-value problem for the second-order equation
(1) consists of finding a solution y of the differential equation
that also satisfies initial conditions of the form
y( x0 )  y0 y / ( x0 )  y1

166
where y0 and y1 are given constants.
Example 4. Solve the initial-value problem
y //  y /  6 y  0; y(0)  1 and y / (0)  0 .
Solution. From Example 1 we know that the general
solution of the differential equation is
y( x)  C1e 2 x  C2e 3 x .
Differentiating this solution, we get
y / ( x)  2C1e 2 x  3C2e 3 x .
To satisfy the initial conditions we require that
y (0)  C1  C2  1
y / (0)  2C1  3C2  0.
2
We have from second equation C2  C1 and from the first
3
2 3 2
C1  C1  1  C1  , C2  .
3 5 5
Thus, the required solution of the initial-value problem is
3 2
y  e 2 x  e 3 x
5 5
Example 5. Solve the initial-value problem
y //  y  0 y(0)  2 y / (0)  3
Solution. The auxiliary equation is
k 2  1  0 or k 2  1, whose roots are  i . Thus
  0,   1and since e 0 x  1 , the general solution is
y ( x)  C1 cos x  C2 sin x
Since y( x)  C1 sin x  C2 cos x
the initial conditions become
y(0)  C1  2 y / (0)  C2  3
Therefore, the solution of initial-value problem is
y ( x)  2 cos x  3 sin x
Summary: Solution of ay //  by /  c  0
167
Roots of ak 2  bk  c  0 General solution
k1 , k 2 - real and distinct y  C1e k1x  C2e k2 x
k1  k 2  k y  C1e kx  C2 xekx
k1 , k 2 - complex:   i y  ex (C1 cos x  C2 sin x)

TEST
1. Solve the differential equation y //  x3
x5 x4
A) y   C1 x  C2 B) y   C1 x  C2
20 4
C) y  x 3  C1 x 2  C2 D) y  3x 2  C1 x  C2
E) y  4 x 3  C2 x  C1
2. Solve the differential equation xy //  y /
A) y  C1 x  C2 B) y  C1 x 2  C2
C) y  C1 x 3  C2 x D) y  C1 x 4  C2 x 3
E) y  x
3. Solve the differential equation y //  y /  x
A) y  C1 x  C2 B) y  C1 x 2  C2
x2
C) y  C1e  C2  x 
x
D) y  x 2  C1e x
2
E) y  C1 xe  C2
x

y/
4. Solve the differential equation y  x
//

168
x3
A) y   C1 x  C2 B) y  C1e x  C2
6
x3
C) y  C1 x 3  C2 x 2 D) y   C1 x 2  C2
3
E) y  e x

5. Solve the differential equation yy //  ( y / ) 2


A) y  C1eC2 x B) y  C1e x  C2
x3
C) y  e x  C1 x  C2 D) y   C1
6
x3
E) y   C1 x  C2
3

6. Solve the differential equation with constant coefficients.


y //  y /  2 y  0
A) y  C1e x  C2 B) y  C1e x  C2e5 x
C) y  C1e  x  C2 D) y  C1e  x
E) y  C1e x  C2e2 x

7. Solve the differential equation with constant coefficients.


y //  6 y /  13y  0
A) y  C1e 3 x  C2e 2 x B) y  e 3 x (C1 cos 2 x  C2 sin 2 x)
C) y  C1e 3 x  C2 cos 2 x D) y  C1e 2 x  C2 sin 3x
E) y  x

8. Solve the differential equation with constant coefficients.


16 y //  8 y /  y  0
A) y  C1e 3 x  C2 B) y  C1e 4 x  C2e8 x

169
x

C) y  (C1  C2 x)e 4
D) y  C1 x  C2e x
E) y  C1e x  C2
9. Solve the initial-value problem y //  4 y /  3 y  0 ; y (0)  6 ;
y / (0)  10
A) y  2e x  3 B) y  4e5 x  6
C) y  2e x  2 D) y  4e x  2e3 x
E) y  4e5 x  2e x
10. Solve the initial-value problem y //  4 y /  29 y  0 ;
y (0)  0 ; y / (0)  15
A) y  6 cos 3x B) y  6 sin 5 x
C) y  3e 2 x  5 cos x D) y  e 2 x  5e5 x
E) y  3e 2 x sin 5x

Answers:
1)A 2)B 3)C 4)D 5)A
6)E 7)B 8)C 9)D 10)E

170
Lecture 12
Nonhomogeneous Linear Differential Equations
Let us consider second-order nonhomogeneous linear
differential equations with constant coefficients, that is,
equation of the form

ay//  by/  cy  f ( x) (1)


where a,b,c are constants and f(x) is a continuous function. The
related homogeneous equation
ay//  by/  cy  0 (2)
is called the complementary equation and plays an important
role in the solution of the original nonhomogeneous equation
(1).
Theorem. The general solution of the nonhomogeneous
differential equation (1) can be written as
y ( x)  y p ( x)  y c ( x) ,
where y p is a particular solution of equation (1) and y c is
the general solution of the complementary equation (2).
Proof. All we have to do is verify that if y is any solution
of equation (1), then y  y p is a solution of the complementary
equation (2).
Indeed
a( y  y p ) //  b( y  y p )  c( y  y p ) 
 ay //  ay p//  by /  by p/  cy  cy p 
 (ay //  by /  cy)  (ay p//  by p/  cy p ) 
 f ( x)  f ( x)  0

which is what we had to prove.


We know from last Lecture 11 how to solve the
complementary equation. Therefore, theorem 1 says that we
171
know the general solution of the nonhomogeneous equation as
soon as we know a particular solution y p . There are two
methods for finding a particular solution: The method of
undetermined coefficients is straightforward but works only for
a restricted class of functions f (x) . The method of variation of
parameters works for every function f (x) but it is usually more
difficult to apply in practice.

The method of Undetermined Coefficients

I. We first illustrate the method of undetermined


coefficients for the equation
ay//  by/  cy  f ( x)
where f (x) is a polynomial of the form
f ( x)  Pn ( x)  a0 x n  a1 x n1  ...  an
It is reasonable to guess there is a particular solution y p
that is a polynomial of the same degree as f (x) because if y is
a polynomial, then ay //  by /  cy is also a polynomial. We
therefore will look for y p of the form y p  Qn ( x) x r , where
Qn (x) is a polynomial of the same degree as f (x) with
unknown coefficients and r is number of the roots of the
auxiliary equation of (2), that is equal to zero.
Example 1. Solve the equation y //  y /  2 y  x 2 .
Solution. The auxiliary equation of
y //  y /  2 y  0 is
k 2  k  2  (k  1)(k  2)  0
with roots k1  1, k2  2.
So the solution of the complementary equation is
yc  c1e x  c2 e 2 x

172
Since f ( x)  x 2 is a polynomial of degree 2, we seek a
particular solution of the form
y p ( x)  Ax 2  Bx  C
Then
y /p ( x)  2 Ax  B and y //p ( x)  2 A
So, substituting into the given differential equation, we
have
2 A  (2 Ax  B)  2( Ax2  Bx  C )  x 2
or
 2 Ax2  (2 A  2B) x  (2 A  B  2C )  x2 .

Polynomials are equal when their corresponding coefficients


are equal. Thus
 2 A  1

2 A  2 B  0
2 A  B  2C  0

The solution of this system of equations is
1 1 3
A , B , C
2 2 4
A particular solution is therefore
1 1 3
y p ( x)   x 2  x 
2 2 4
and, by theorem 1, the general solution is
1 1 3
y  yc  y p  C1e x  C2 e  2 x  x 2  x  .
2 2 4
Example 2. Solve the equation
y //  y /  5x  3 .
Solution. The auxiliary equation of
y //  y /  0 is
k2 k  0
173
with roots k1  0 k 2  1
So the solution of complementary equation is
yc  C1  C2 e  x
Since f ( x)  5 x  3 is a polynomial of degree 1 and one of the
roots of auxiliary equation is 0 (r=1), then we look for a
particular solution of the form y p ( x)  ( Ax  B) x  Ax 2  Bx
Then y /p  2 Ax  B and y //p  2 A
So, substituting into the given equation, we obtain
2 A  2 Ax  B  5x  3
or
2 Ax  (2 A  B)  5 x  3
Polynomials are equal when their coefficients are equal. Thus
2 A  5

2 A  B  3
The solution of this system of equations is
5
A  ; B  2
2
Therefore, a particular solution is
5
y p ( x)  x 2  2 x
2
and, the general solution is
5
y  yc  y p  C1  C2 e  x  x 2  2 x
2
II. If the right side of equation (1) f (x) is of the form

ex Pn (x) , where Pn (x) is an n th degree polynomial we look for

y p in the form

y p  Q( x) ex x r ,

174
where Qn (x) is a polynomial of the same degree as Pn (x) with
unknown coefficients and r is number of the roots of the
auxiliary equation of (2), which coincides with  .
Example 3. Solve y //  4 y  e3 x

Solution. The auxiliary equation is k 2  4  0 with roots


 2i , so the solution of the complementary equation is
yc ( x)  C1 cos 2 x  C2 sin 2 x

For a particular solution we try y p ( x)  Ae 3 x . Then

y /p  3 Ae 3 x and y //p  9 Ae 3 x .

Substituting into the differential equation, we have


9 Ae 3 x  4 Ae 3 x  e 3 x
1
So 13 Ae 3 x  e 3 x and A 
13
Thus, a particular solution is
1 3x
y p ( x)  e
13
and the general solution is
1 3x
y ( x)  C1 cos 2 x  C2 sin 2 x  e .
13
Example 4. Solve the equation
y //  2 y /  3 y  ( x  2)e3 x .
Solution. The auxiliary equation is
175
k 2  2k  3  0 with roots k1  1, k 2  3 .
So the solution of the complementary equation is
yc ( x)  C1e  x  C2 e 3 x
Since x  2 is a polynomial of degree 1 and one of the
roots of auxiliary equation is coincided with 3 in exponent,
then we look for particular solution in the form
y p  x( Ax  B)e 3 x

or y p  ( Ax 2  Bx)e 3 x

Then
y /p  (2 Ax  B)e 3 x  3( Ax 2  Bx)e 3 x 
 (3 Ax 2  3Bx  2 Ax  B)e 3 x
and
y //p  (6 Ax  3B  2 A)e 3 x  3(3 Ax 2  3Bx  2 Ax  B)e 3 x 
 (9 Ax 2  12 Ax  9 Bx  6 B  2 A)e 3 x
Substituting into the differential equation, we have
(9 Ax 2  12 Ax  9 Bx  6 B  2 A)e 3 x 
 2(3 Ax 2  3Bx  2 Ax  B)e 3 x 
 3( Ax 2  Bx)e 3 x  ( x  2)e 3 x

or 9 Ax 2  12 Ax  9 Bx  6 B  2 A 
 6 Ax 2  6 Bx  4 Ax  2 B 
 3 Ax 2  3Bx  x  2

176
Simplifying, we get
8 Ax  (2 A  4 B)  x  2
whence
8 A  1

2 A  4 B  2
The solution of this system of equation is
1 7
A  and B 
8 16
Thus, a particular solution is
1 7 
y p ( x)   x 2  x  e 3 x
8 16 
and the general solution is

1 7 
y ( x)  C1e  x  C2 e 3 x   x 2  x e 3 x
8 16 
III. If the right side of equation (1) f (x) is of the form
M cos x  N sin x , we take as particular solution a function
of the form
y p ( x)  ( A cos x  B sin x) x r
where A,B are unknown coefficients and r = number of the
roots of the auxiliary equation of (2), coinciding with i
Example 5. Solve y //  y /  2 y  sin x
Solution. We try a particular solution
177
y p ( x)  A cos x  B sin x

Then y /p ( x)   A sin x  B cos x

y //p ( x)   A cos x  B sin x

So substitution in the differential equation gives


( A cos x  B sin x)  ( A sin x  B cos x) 
 2( A cos x  B sin x)  sin x
or
(3 A  B) cos x  ( A  3B) sin x  sin x
This is true if
 3 A  B  0

 A  3 B  1
The solution of this system is
1 3
A and B  
10 10
So a particular solution is
1 3
y p ( x)   cos x  sin x
10 10
In example 1 we determined that the solution of the
complementary equation is
yc  C1e x  C2 e 2 x
Thus, the general solution of the given equation is

178
1
y ( x)  C1e x  C 2 e  2 x  (cos x  3 sin x)
10
If f (x) is a sum of functions of the preceding types, we use the
easily verified principle of superposition, which that if
y p1 and y p2 are solution of

ay //  by /  cy  f1 ( x)

and ay //  by /  cy  f 2 ( x)
respectively, then y p1  y p2 is a solution of

ay //  by /  cy  f1 ( x)  f 2 ( x)

Example 6. Solve y //  4 y  xe x  cos 2 x

Solution. The auxiliary equation is k 2  4  0 with roots


 2 , so the solution of the complementary equation is
yc ( x)  C1e 2 x  C2 e 2 x

For the equation y //  4 y  xe x we try

y p1 ( x)  ( Ax  B)e x

Then y p/  ( Ax  B  A)e x
1

y p  ( Ax  2 A  B)e x
//
1

So substitution in the equation gives


( Ax  2 A  B)e x  4( Ax  B)e x  xe x

or (3 Ax  2 A  3B)e x  xe x
179
Thus
 3 A  1

 2 A  3 B  0
1 2
So A   , B   and
3 9
 1 2
y p1 ( x)    x  e x
 3 9

For the equation y //  4 y  cos 2 x , we try

y p2 ( x)  C cos 2 x  D sin 2 x

Substitution gives
 4C cos 2 x  4 D sin 2 x  4(C cos 2 x  D sin 2 x)  cos 2 x
or  8C cos 2 x  8D sin 2 x  cos 2 x
therefore  8C  1;  8D  0 and
1
y p2 ( x)   cos 2 x
8
By the superposition principle, the general solution is
1 2 1
y  yc  y p  y p  C1 e 2 x  C2 e 2 x   x  e x  cos 2 x
3 9 8
1 2

Summary: A particular solution of ax //  by /  cy  f ( x)


f (x) yp
I Pn (x) Qn ( x) x r , r is number of roots of
the auxiliary equation equals 0

180
II Pn ( x)ex Qn ( x) xx  x r , r is number of roots
of the auxiliary equation which
coincides with 
III M cos x  N sin x  A cos x  B sin xx r ,r is number
of roots of the auxiliary equation
which coincides with  i

The Method of Variation of Parameters


Suppose we have already solved the homogeneous equation
ay //  by /  cy  0
and written the solution as
y ( x)  C1 y1 ( x)  C 2 y 2 ( x) (3)
where y1 and y 2 are linearly independent solutions. Let us
replace the constants (or parameters) C1 and C2 in equation (3)
by arbitrary functions u1 ( x) and u 2 ( x ) . We look for a
particular solution of the nonhomogeneous equation
ay //  by /  cy  f ( x)
of the form
y p ( x)  u1 ( x) y1 ( x)  u2 ( x) y2 ( x) (4)

This method is called variation of parameters because


we have varied the parameters C1 and C2 to make them
functions. Differentiating equation (4), we get
y /p  (u1/ y1  u 2/ y2 )  (u1 y1/  u 2 y2/ ) (5)
181
Since u1 and u 2 are arbitrary functions, we can impose
two conditions on them. One condition is that y p is a solution

of the differential equation; we can choose the other condition


so as to simplify our calculations. In view of the expression in
equation (5), let us impose the condition that
u1/ y1  u2/ y2  0 (6)
Then
y //p  u1/ y1/  u 2/ y2/  u1 y1//  u 2 y2//

Substituting in the differential equation, we get


a(u1/ y1/  u2/ y2/  u1 y1//  u2 y2// ) 
 b(u1 y1/  u2 y2/ )  c(u1 y1  u2 y2 )  f ( x)
or
u1 (ay1//  by1/  cy1 )  u 2 (ay2//  by2/  cy2 ) 
(7)
 a(u1/ y1/  u 2/ y2/ )  f ( x)
But y1 and y 2 are solution of the complementary
equation, so
ay1//  by1/  cy1  0 and ay2//  by2/  cy2  0
and equation (7) simplifies to
a(u1/ y1/  u 2/ y2/ )  f ( x) (8)
Equations (6) and (8) form a system of two equations in
the unknown functions u1/ and u 2/ . After solving this system we

182
may be able to integrate to find u1 and u 2 and then the
particular solution is given by equation (4).
Example7. Solve the equation y //  y  tan x, 0  x  
2
Solution. The auxiliary equation is k 2  1  0 with roots
k1, 2   i , so the solution of y //  y  0 is
yc  C1 sin x  C2 cos x
Using variation of parameters, we look for a solution of
the form
y p ( x)  u1 ( x) sin x  u 2 ( x) cos x
Then
y /p  (u1/ sin x  u 2/ cos x)  (u1 cos x  u 2 sin x)
Set
u1/ sin x  u2/ cos x  0 (9)
Then
y //p  u1/ cos x  u 2/ sin x  u1 sin x  u 2 cos x
For y p to be a solution we must have
y //p  y p  u1/ cos x  u 2/ sin x  tan x (10)
Solving equations (9) and (10), we get
u1/ (sin 2 x  cos2 x)  cos x tan x
u1/  sin x , u1 ( x)   cos x
We seek a particular solution, so we don’t need a constant of
integration here.
Then from equation (9), we obtain
sin x / sin 2 x cos2 x  1 1
u 
/
2 u1     cos x 
cos x cos x cos x cos x

183
x 
So u 2 ( x)  sin x  ln tan  
2 4
Therefore
  x  
y p ( x)   cos x sin x  sin x  ln tan    cos x 
 2 4 
x 
  cos x ln tan  
2 4
and the general solution is

x 
y ( x)  C1 sin x  C2 cos x  cos x ln tan  
2 4

TEST
1. Solve the differential equation using the method of
undetermined coefficients.
2 y //  y /  y  2e x
x
A) y  C1e  x  C 2 e 2  e x B) y  C1e  x  e x

C) y  C2 e 2 x  x D) y  C1e  x  C2 e x
E) y  x  5
2. Solve the differential equation using the method of
undetermined coefficients.
y //  3 y /  2 y  10e  x

184
A) y  C1e  x  C2 e x B) y  C1  C2 e 2 x
5
C) y  C1e  2C 2 e D) y  C1e x  C2 e 2 x  e  x
x 5x

3
E) y  C1e x  2C2
3. Solve the differential equation using the method of
undetermined coefficients. y //  7 y /  6 y  sin x

A) y  C1 sin x  C2 cos x
5 7
B) y  C1e6 x  C2e x  sin x  cos x
74 74
5
C) y  C1 cos 6 x  C2 x  sin x
74
D) y  sin x E) y  cos x
4. Solve the differential equation using the method of
undetermined coefficients. y //  3 y /  2 y  x 2
1 2
A) y  C1e 2 x  C2 e  x B) y  x
2
1 2 3 7
C) y  C1e  2 x  C2 e  x  x  x
2 2 4
7 3
D) y  C1 x 2  C 2 x  E) y  e  2 x  x
4 2
5. Solve the differential equation using the method of
undetermined coefficients. y //  2 y /  sin 4 x

185
A) y  C1 sin 4 x  C 2 cos 4 x B) y  C1e x  sin x

C) y  C1e x  C2 e 2 x D) y  C1 sin 4 x
1 1
E) y  C1  C2 e 2 x  cos 4 x  sin 4 x
40 20
6. Solve the differential equation using the method of
undetermined coefficients. y //  6 y /  9 y  2 x 2  x  3
2 2 5 11
A) y  (C1  C2 x)e 3 x  x  x
9 27 27
2 2 2 2 5
B) y  C1  C2 x  x C) y  x  x
9 9 27
D) y  C1e3 x  C2 E) y  C1 x  C2 e 3 x
7. Solve the differential equation using the method of
undetermined coefficients. 2 y //  5 y /  e x
5x 5x
  1
A) y  C1  C 2 e 2
B) y  C1  C2 e 2
 ex
7
5x

C) y  C1 x  C 2 D) y  C1  e x
E) y  e 2

8. Solve initial-value problem using the method of


undetermined coefficients. y //  2 y /  2e x ,

y(1)  1, y / (1)  0

A) y  e 2 x 1 B) y  e  1 C) y  2e x  5e  x

D) y  e 2 x 1  2e x  e  1 E) y  6 x

186
9.Solve initial-value problem using the method of
undetermined coefficients. y //  y  4e x , y(0)  4, y / (0)  3 .

A) y  2 cos x B) y  5 sin x C) y  2 cos x  5 sin x  2e x

D) y  2e x E) y  2 cos x  2e x
10. Solve the differential equation using the method of
undetermined coefficients. 2 y //  5 y /  29 cos x .
A) y  C1  C 2 x B) y  2 sin x  29 cos x

C) y  C1 sin x  C2 D) y  6 sin x  3 cos x


5x

E) y  C1  C 2 e 2
 5 sin x  2 cos x
Answers:
1)A 2)D 3)B 4)C 5)E
6)A 7)B 8)D 9)C 10)E

187
Lecture 13
PROBABILITY

Probability is a key part of mathematics. It plays an


important role in the money markets, in weather reporting, and
many other areas of human life.
In everyday speech, the word “probable” is used in vague
senses. For example, we say “it will probably rain tomorrow”,
“the patient will probably recover (or die)”, “our football team
will probably win”, “I will probably get my salary tomorrow”
and so on.
The concept of probability is important and useful
because it gives us more than just a reflection of past
experience.

Events. Types of Events

The most commonly used term in Probability theory is


“event”. In everyday life we often say that a certain event is
highly probable whereas some other event is improbable. The
“event” can be almost any observable phenomenon.
All events that we may observe or that may occur in our
daily life one can subdivide into three following types: certain,
impossible and random events.
A certain event is an event which occurs when a certain
set of conditions is realized.
An impossible event is one which cannot occur when a
certain set of conditions is realized.
A random (stochastic) event is one that may occur or
may not occur when a certain set of conditions is realized.
Every realization of the indicated set of conditions is
called a trial or an experiment.
Let us explain first what we mean when we say “a trial”
or “an event”. For instance, when tossing a coin we can
188
consider the fact that it shows heads to be an event. In this case
tossing the coin serves as a trial. (an experiment).
Let us make the following conclusions and give
definitions.
A trial or an experiment is a process that can be
repeated and gives rise to a number of outcomes.
An event is a collection (or set) of one or more
outcomes.
A sample space is the set of all possible outcomes of an
experiment. Events are denoted by capital letters.
The events are called equally likely (or equally probable)
if there is no ground to believe that some one of them is more
probable than the others.

Empirical Probability

A probability that is estimated from observations is


called empirical or experimental probability.
Out of 205468 births is Azerbaijan in 2015, there were
3146 multiple births (twins, triplets, etc.). The fraction of
3146
multiple births is or about 0.015; this is an estimate of
205468
probability of a pregnancy being a multiple birth in Azerbaijan.
The probability of an event A occurring is denoted by
P(A). From observation, we can estimate P(A) according to the
following definition.
Definition.

number of times A occurs


P( A)  .
number of times A could have occurred
Notice, that, according to this definition, P(A) must be between
0 and 1 inclusive.

189
Obviously, if we are given a probability and the number of
times the event could have occurred, we can estimate how
many times we think that the event might occur.

Theoretical Probability

We can always estimate the probability of an event


empirically by performing an experiment repeatedly, but in
some cases it is possible to predict the probability before doing
any experiments.
Definition.
If A is any event, the probability that A will occur is
given by

number of times A occurs in the sample space


P ( A) 
number of items in the sample space
or

number of ways event A can occur


P ( A) 
total number of passible outcomes

assuming that all the possible outcomes are equally likely.

m
In the summary form: P(A) 
n
where m is the number of outcomes favourable to A, and n is
the number of all the possible outcomes.

Properties of probabilities
Let us now list some properties of probabilities:
1. The probability of a random event is between 0 and 1.
The event A is random  0  P( A)  1 .
2. The probability of a certain (sure) event is 1.
190
The event A is certain  P( A)  1 .
3. The probability of an impossible event is zero.
The event A is impossible  P( A)  0 .
4. The probability that event A will not occur (is denoted
by A ) is 1  P( A) .
5. The more likely that an event will occur, the higher its
probability (the closer to 1 it is); the less likely that an event
will occur, the lower its probability (the closer to 0 it is).

0 1

Our football Flipping Snowing at


team will Heads on a least once
become the fair coin during
World November in
champion Snowing at Moscow Karabakh is
least once the part of
during Azerbaijan
January in
Baku

Let us now consider some examples.


Example 1.
A fair six sided die is thrown once and the number
landing face up is recorded.
(a) Find the probability of the die landing with the
number 6 face up.
(b) Find the probability of throwing an odd number.
(c) Find the probability of throwing a number 8.

191
(d) Find the probability of throwing a number less than
15.
Solution:
(a) There are six faces and one has the number 6 on it, so the
probability of landing with the number 6 face up is
1
P ( 6)  .
6
(b) The odd numbered faces are 1,3, and 5, so the the
3 1
probability of throwing an odd number is P(odd )   .
6 2
(c) There are no faces with the number 8, so the probability of
0
landing with the number 8 face up is P(8)   0 .
6
In the given case, the event is impossible.
(d) Since the number of any face of the die less than 15, it
follows that the number of outcomes favourable to this
event is equal to the number of all the possible outcomes.
6
Consequently, P (number <15)=  1 . In this case, the
6
event is certain.
Example 2.
A fair spinner below has eight faces, that is there are
eight possible outcomes. We spin the spinner once and the
number indicated on it is recorded.

24 6
20 8

17 11
15 14

192
Find the following probabilities:
(a) P(8)
(b) P(even)
(c) P(odd)
(d) P(prime number)
(e) P(number <10)
(f) P(number >16)

Solution.
Since each region is the same size, it is equally likely that
the spinner will stop in any of the eight regions.
(a) There is 1 chance in 8 that it will stop in the region marked
8. So we say that the probability of spinning a 8 is one
1
eighth and write P (8)  .
8
(b) There are 5 chances in 8 that the spinner will land in a
5
region with an even number in it, so P(even)  .
8
(c) There are 3 chances in 8 that the spinner will land in a
3
region with an odd number in it, so P(odd )  .
8
(d) There are 2 chances in 8 that the spinner will land in a
region with a prime number in it, so
2 1
P (prime number)   .
8 4
(c) There are 2 chances in 8 that the spinner will stop in a
region with a number less than 10 in it, so
2 1
P(number <10)   .
8 4
(f) There are 3 chances in 8 that the spinner will stop in a
region with a number greater than 16 in it, so

193
3
P(number >16)  .
8

Example 3.
A box contains 10 balls labeled with the numbers from 1
to 10. We draw one ball. What is the probability that the
number of the drawn ball:
(a) does not exceed 10
(b) prime number
(c) a factor of 10

Solution.

10
(a) P(  10 )   1 , the event is certain.
10
4 2
(b) P(prime number) = P(2,3,5,7)   .
10 5
4 2
(c) P(factor of 10) = P(1,2,5,10)   .
10 5
Example 4.
There are 12 balls in the urn, of which 3 are green, 4 red
and 5 blue. What is the probability of drawing.
(a) a green ball,
(b) a red ball,
(c) a blue ball.
Solution.
3 1
(a) P(green)  
12 4
4 1
(b) P(red)  
12 3
5
(c) P(blue)  .
12

194
Counting Techniques

The probability formula for equally likely outcomes uses


the size of the sample space. In some simple experiments, this
may be easily determined, but some experiments require more
sophisticated counting techniques.
The basis of most counting techniques is the Counting
Principle.
Counting Principle. If two jobs need to be completed
and there are m ways to do the first job and n ways to do the
second job, then there are m  n ways to do one job followed by
the other.
This principle can be extended to any number of jobs or
experiments.
Counting Principle (General Case)
Consider a set of k experiments. Suppose the first
experiment has n1 outcomes, the second has n2 outcomes, and
so on. Then the total number of outcomes is n1  n2  ...  nk for all
k experiments.
Example. How many integers between 100 and 1,000
consist of three different odd digits?
Solution. We are looking for 3-digit numbers, such as
157,359, and 753, all of whose digits are odd and all of which
are different. The best way to answer this question is to use the
counting principle. Think of writing a 3-digit number as three
jobs that need to be done. The first job is to select one of the
five odd digits (1,3,5,7,9) and use it as the digit in the hundreds
place. This can be done in 5 ways. The second job is to select
one of the four remaining odd digits for the ten place. This can
be done in 4 ways. Finally, the third job is to select one of the
three odd digits not yet used to be the digit in the ones place.
This can be done in 3 ways. So, by using the counting
principle, the total number of ways to write a 3-digit number
with three different odd digits is 5  4  3  60 .
195
Combinatorics

Many problems in probability theory require that we


count the number of ways that particular event can occur. For
this, we need to know some concepts of combinatorics
(factorial, permutations, combinations).
Combinatorics is a branch of mathematics concerning the
study of finite or countable discrete structures. The
combinatorics studies the number of selections (or variants, or
ways) subordinated to the certain conditions. Combinatorial
problems arise in many areas of mathematics, notably in
probability theory. Combinatorics also has many applications
in computer science and statistical physics.
Factorial.
Definition. Let n be a positive integer. We define n!
(read “n factorial”) by
n! n(n  1)(n  2)  ...  3  2  1 .
By convention, we set 0!  1 .
The factorial n! gives the number of ways in which n
distinct objects can be permuted.
Permutations.
Definition. The different arrangements or selections of a
given number of distinct objects are called permutations.
The number of all the possible permutations of n distinct
objects is
Pn  n!

Arrangements (Permutations)
Definition. The different arrangements or selection of a
given number of distinct objects (things) taken k at a time are
called arrangements (or permutations).

196
The number of all the possible arrangements of n things,
taken k at a time, is given by:
n!
Ank  Pn,k  n(n  1)(n  2)...(n  k  1) 
(n  k )!
In particular An  Pn,n  n!, i.e. An  Pn .
n n

Combinations.
Definitions. Each of the different groups or selections of
a given number of objects, and which can be formed by taking
some or all of a number of objects is called a combination.
The number of all combinations of n things, taken k at a
time is given by:
n n!
Cnk    
 k  k!(n  k )!

Note. It is useful to remember:


 In permutations the order matters
 In combinations the order does not matter
Ak n!
Properties: 1. C nk  n 
Pk k!(n  k )!
2. Ank  Cnk  Pk
3. Cn0  Cnn  1
4. Cnk  Cnnk
5. Cnk  Cnk11  Cnk1

Example 1.
There are 10 balls in the box, of which 6 are green and 4
blue. We draw two balls. What is the probability that they are
(a) 2 green balls
(b) 2 blue balls

197
(c) 1 green, 1 blue
Solution.
C 2 15 1
(a) P(2 green) = 62  
C10 45 3
C 42 6 2
(b) P(2 blue) = 2
 
C10 45 15
C61  C 41 6  4 8
(c) P(1 green, 1 blue) =  
C102 45 15
Example 2.
A bag contains 4 red, 3 blue and 8 green beads.
Five beads are selected at random from the bag without
replacement. Find the probability that they are
(a) 5 green beads,
(b) 2 red and 3 green,
(c) 4 red and 1 blue
(d) 1 red, 2 blue and 2 green
Solution.
C5 8
(a) P(5 green) = 85 
C15 429
C 42  C83 16
(b) P(2 red and 3 green) = 
C155 143
C 44  C31 1
(c) P(4 red and 1 blue) = 5

C15 1001
C 41  C32  C82 16
(d) P(1 red, 2 blue and 2 green) = 
C155 143
Example 3.
Six students from a group of 30 have passed the test with
excellent marks, ten students with good marks, and nine
students have got satisfactory marks. What is the probability

198
that three students chosen randomly from this group have got
failing marks for the test?
Solution. The number of students that have got failing
marks is 30  (6  10  9)  5 . Then the number of cases
favourable to the event is determined by the equation m  C53 ,
i.e. m=10. The number of all events here is
30!
n  C303   28  29  5
3!27!
m C53 10 1
Therefore P   3   .
n C30 28  29  5 406
Example 4.
There are 1000 tickets in the lottery, of which 500 are
winning tickets and 500 are non-winning. We buy two tickets.
What is the probability of both tickets being winning?
Solution. The number of all possible outcomes here is
1000!
n  C1000
2
  999  500
2! 998!
The number of outcomes favourable to the event here is
500!
m  C500
2
  499  250 .
2! 498!

Thus, we have
2
m C500 499  250 499
P  2   .
n C1000 999  500 1998

Probability Addition and Multiplication Rules

Now we shall generalize the sample space method to


efficiently calculate probabilities of combined events.

199
Definition. The union of event A and event B is written
A  B and represents the event that either A or event B occurs
or both occur.
Definition. The intersection of event A and B is written
A B and represents the event that both A and B occur.
We can use Venn diagrams to illustrate the concepts of
union and intersection. Events can be represented graphically
by a Venn diagram. Venn diagrams are named after English
mathematician John Venn (1834-1923). We use set notation to
identify different areas on a Venn diagram.
A rectangle represents
the sample space and it A B S
contains closed curves that
represent events. It includes
all the possible outcomes of
an experiment so the
probability of the sample space is 1 or P(S)=1.

A B A B

A B S A B S

A B
A B S A B S

200
We are also interested in events not occurring. For
example, A (or A/ ) is the complement of A and is the event “A
does not occur”, P( A )  1  P( A) .
Venn diagrams can help us solve probability problems.
Theorem (Probability Addition Rule)
P( A  B)  P( A)  P( B)  P( A  B) .
It is useful to remember that we can rearrange this if we
want to find the probability of the intersection.
P( A  B)  P( A)  P( B)  P( A  B)
Definition. Two events are said to be exclusive
(mutually exclusive) if they cannot occur at the same time.
Obviously, if there is no possibility of A and B occurring
at the same time, that is A and B are mutually exclusive, then
P( A  B)  0 and the above formula reduces to
P( A  B)  P( A)  P( B) .
Definition. The event consisting in the non-occurrence of
the event A is said to be an event opposite, or contrary, to the
event A and is denoted A (or A/ ) .
The union of the events A and A yields a certain event,
and hence, P( A  A )  1, and since the events A and A are
mutually exclusive, we have
P( A)  P( A )  1 or P( A )  1  P( A) .
Example 1.
A and B are two events and P(A) =0.6, P(B)=0.5 and
P( A  B)  0.8 . Find the following:
(a) P( A  B)
(b) P(A ) A B S
(c) P( A  B) 0.3 0.3 0.2

(d) P( A  B)
0.2
Solution.

201
(a) P( A  B)  P( A)  P( B)  P( A  B)
P( A  B)  0.6  0.5  0.8
P( A  B)  0.3
(b) P( A )  1  P( A)
P( A )  1  0.6  0.4
(c) P( A  B)  (0.2  0.2)  0.3  0.7
(d) P( A  B)  P( A )  P( B)  P( A  B)
P( A  B)  0.4  0.5  0.7  0.2
Example 2.
A and B are two events and P(A) =0.4, P(B)=0.3 and
P( A  B)  0.1 . Find the following:
(a) P( A  B)
(b) P(B ) A B S
(c) P( A  B ) 0.3 0.1 0.2

(d) P( A  B ) 0.4
Solution.
(a) P( A  B)  P( A)  P( B)  P( A  B)
P( A  B)  0.4  0.3  0.1  0.6
(b) P( B )  1  P( B)
P( B )  1  0.3  0.7
(c) P( A  B )  0.3
(d) P( A  B )  P( A)  P( B )  P( A  B )
P( A  B )  0.4  0.7  0.3  0.8

202
Example 3.
C and D are two events and P(C) =0.4, P(D)=0.5 and
P(C  D)  0.6 . Find the following:
(a) P(C  D)
(b) P(C ) C D S
0.3 0.2
(c) P(C  D ) 0.1

(d) P(C  D) 0.4


Solution.
(a) P(C  D)  P(C )  P( D)  P(C  D)
P(C  D)  0.4  0.5  0.6  0.3
(b) P(C )  1  P(C )
P(C )  1  0.4  0.6
(c) P(C  D )  0.8
(d) P(C  D)  P(C )  P( D)  P(C  D)
P(C  D)  0.6  0.5  0.2  0.9
Example 4.
There are 20 students in a certain tutor group at Baku-
Oxford School. There are 12 students in the tutor group
studying French, 11 students studying Spanish and five
students studying both Spanish and French.
Find the probability that a randomly chosen student in the
tutor group
(a) studies Spanish
(b) studies Spanish and French
(c) studies Spanish but not French
(d) does not study Spanish or French
Solution.
Draw a Venn diagram to represent this information.
There are 20 students in the sample space. A student is chosen
at random so all outcomes are equally likely.

203
11
(a) P( S )   0.55
20
5 F S
(b) P( S  F )   0.25
20 7 5 6
6
(c) P( S  F )   0.3 2
20
2
(d) P( S  F )   0.1
20
Example 5.
A card is chosen at random from a pack of 52 playing
cards. H is the event ‘the card chosen is a Heart’ and Q is the
event ‘the card chosen is a Queen! Find the following:
(a) P (Q ) (d) P( H  Q)
(b) P(H ) (e) P(H )
(c) P( H  Q) (f) P(Q  H )

Solution.
Draw a Venn diagram to represent this information.
There are 52 cards in the sample space. A card is chosen at
random so all outcomes are equally likely.
4 1
(a) P(Q)  
52 13
13 1 H Q
(b) P( H )   12 1 3
52 4
1
(c) P( H  Q)  36
52
16 4
(d) P( H  Q)  
52 13
1 3
(e) P( H )  1  
4 4

204
12 3
(f) P(Q  H )  
52 13

Let us now consider two events A and B. The probability


of an event B may be different
if we know that a dependent A B
event A has already occurred.
a–i i b–i
Consider the Venn diagram.
The event A has happened, so
S
we are considering probabi-
lities inside this closed curve. We are looking for the
probability of B given A has happened so we divide i by a .
Thus
P( B  A)
P( B given A) 
P( A)

Definition. The probability of B given A, written


P( B | A) , is called the conditional probability of B given A
and so:
P( B  A)
P( B | A)  . (1)
P( A)
In other words, the conditional probability of the event
B relative to the event A is understood as the probability of the
occurrence of the event B defined relative to the hypothesis that
the event A has occurred.
Similarly, we have
P( A  B)
P( A | B)  . (2)
P( B)
(1) and (2) give us a useful formula for the probability of the
intersection P( A  B) .
Theorem (Probability Multiplication Rule)
P( A  B)  P( B)  P( A| B)  P( A)  P( B | A) .
205
Definition. Two events A and B are said to be
independent if one event has no effect on another.
Therefore if A and B are independent, the probability of A
happening is the same whether or not B has happened. In other
words, if A and B are independent, the conditional probability
of one of them relative to the other is equal to the absolute
(inconditional) probability of the first event:

P( B | A)  P( B) and P( A| B)  P( A)
In this case the following equalities hold true:
P( B | A )  P( B | A)  P( B) ,
P( A| B )  P( A | B)  P( A) .
Thus, it is useful to remember that A, B independent
 A, B; A, B ; A, B are all independent pairs.
Theorem. The probability of the intersection of
independent events is equal to the product of their
probabilities:
A and B are independent  P( A  B)  P( A)  P( B)
Example 6.
A and B are two events such that
P( A)  0.2, P( B)  0.6 and P( A| B)  0.3. Find the following
(a) P( B | A)
(b) P( A  B )
(c) P( A  B)
Solution. First we find
P( A  B)  P( A | B)  P( B)  0.3  0.6  0.18
Now we draw a Venn diagram. This will help to find any
other probabilities in the question.

206
P( B  A) 0.18
(a) P( B | A)    0.9
P( A) 0.2
(b) P( A  B )  0.38 A B
(c) P( A  B)  0.42 0.02 0.18 0.42

0.38

Example 7.
A and B are two events such that P( A)  0.6, P( B)  0.5
and P( A  B)  0.4. Find the following
(a) P( A  B)
(b) P( B | A)
A B
(c) P( A | B)
0.2 0.4 0.1
(d) P( A | B )
0.3

Solution. We draw the Venn diagram for better


understanding although we can solve the problem without it.
(a) P( A  B)  P( A)  P( B)  P( A  B)
P( A  B)  0.6  0.5  0.4  0.7
P( B  A) 0.4 2
(b) P( B | A)     0.(6)
P( A) 0.6 3
P( A  B) 0.4
(c) P( A| B)    0.8
P( B) 0.5
P( A  B ) 0.2
(d) P( A| B )    0.4
P( B ) 0.5
Example 8.
A and B are mutually exclusive and P( A)  0.3 and
P( B)  0.4 . Find
207
(a) P( A  B)
(b) P( A  B )
(c) P( A  B )
Solution. It is clear that when events A and B are
mutually exclusive they
have no outcomes in com- A 0.4
mon. Therefore remem- B
0.3
bering it we draw the cor-
responding Venn diagram 0.3

(a) Since P( A  B)  0 we have


P( A  B)  P( A)  P( B)  0.3  0.4  0.7
(b) P( A  B )  P( A)  0.3
(c) P( A  B )  1  P( A  B)  1  0.7  0.3
Example 9.
2 1 4
P( A)  , P( A | B)  , and P( A  B)  .. Find P( B) .
3 5 5
Solution. P( A  B)  P( A)  P( B)  P( A  B) 
 P( A)  P( B)  P( A| B)  P( B) 
 P( A)  P( B)  (1  P( A | B))
Now using the given values we finally obtain
4 2  1 1
  P( B)1    P( B) 
5 3  5 6
Example 10.
Events A and B are
1
independent and P( A) 
4
A B
1
and P( B)  . Find 6 1 3
7 28 28 28
(a) P( A  B) 18
28

208
(b) P( A  B )
(c) P( A  B )
Solution. A and B are independent, so
1 1 1
(a) P( A  B)  P( A)  P( B)    .
4 7 28
If A and B are independent, then so are A and B , and
hence
1 6 6 3
(b) P( A  B )  P( A)  P( B )     .
4 7 28 14
If A and B are independent, then so are A and B , and
therefore
3 6 18 9
(c) P( A  B )  P( A )  P( B )     .
4 7 28 14
Example 11.
A box contains 6 white and 8 red balls. We draw two
balls (without replacing the drawn ball into the box). Find the
probability of both balls being white.
Solution. Suppose the event A is the drawing of a white
ball in the first trial and the event B is the drawing of a white
ball in the second trial. By the probability multiplication rule
for the case of dependent events we have
6 5 15
P( A  B)  P( A)  P( B | A)   
14 13 91
Example 12.
The probability of winning a certain game is 0.2.
Suppose the game is played on two different occasions. What
is the probability of
(a) winning both times?
(b) losing both times?
(c) winning once and losing once?

209
Solution.
(a) The results of the two different trials are independent,
so the probability of winning both times can be found
by multiplying the probability of winning each time.
P(winning both games)= 0.2  0.2  0.04
(b) Since losing is the compliment of winning, the
probability of losing is 1  0.2  0.8 . The probability
of losing both times can be found by multiplying the
probability of losing each time
P(losing both games)= 0.8  0.8  0.64
(c) The complement of the event (winning once and
losing once ) is the set of the two events in parts (a)
and (b).
The event in parts (a) and (b) are mutually exclusive,
because it is impossible to win both times and lose
both times, so their probabilities may be added.
P(winning once and losing once)=
= 1  (0.04  0.64)  0.32 .
TEST1
1.Two fair six-sided dice numbered 1 to 6 are rolled.
Find the probability that
(a) the sum is 8
1 5 1 7 4
A) B) C) D) E)
9 36 6 36 9
(b) the product is greater than or equal to 8
7 5 2 11 7
A) B) C) D) E)
12 9 3 18 20
(c) the product is 24 or 12
5 2 4 1 1
A) B) C) D) E)
36 3 9 9 6
(d) the maximum value is 4

210
7 1 2 1 4
A) B) C) D) E)
36 6 9 4 9
(e) the larger value is more than twice the other value
1 1 1 1 1
A) B) C) D) E)
3 5 4 6 9

1.A box contains 15 balls, of which 4 are red, 5 yellow


and 6 green. We draw two balls. What is the probability of
getting
(a) 2 green
2 3 1 2 2
A) B) C) D) E)
7 8 7 21 35
(b) 2 red
1 2 1 3 1
A) B) C) D) E)
15 35 21 28 7
(c) 2 yellow
2 4 4 1 2
A) B) C) D) E)
21 15 21 15 35

(d) 1 red and 1 yellow


1 2 2 5 4
A) B) C) D) E)
7 21 35 21 21
(e) 1 green and 1 red
2 4 8 1 2
A) B) C) D) E)
35 21 35 7 21

3.A bad contains 5 blue, 7 white and 8 pink beads. Six


beads are selected at random from the bag without
replacement. Find the probability that they are
(a) 6 pink beads,

211
5 11 3 7 4
A) B) C) D) E)
1938 2584 3876 9690 1646
(b) 3 blue and 3 white
35 7 17 3 5
A) B) C) D) E)
3876 1938 6480 646 2472

(c) 4 white and 2 pink


7 17 49 15 31
A) B) C) D) E)
9690 2124 1938 3742 3876

(d) 5 pink and 1 blue


3 5 4 6 7
A) B) C) D) E)
875 938 741 893 969

(e) 2 blue, 2 white and 2 pink


52 49 3 15 14
A) B) C) D) E)
969 323 263 464 969

4.We toss a coin twice. What is the probability of the


coin falling heads up both times?

1 3 1 2 1
A) B) C) D) E)
2 4 3 3 4

5. Three coins are tossed at the same time. Find the


probability of obtaining;

(a) three heads


3 1 1 3 5
A) B) C) D) E)
8 4 8 4 8
212
(b) two heads and one tail
3 3 2 1 1
A) B) C) D) E)
4 8 3 8 4
(c) no heads
1 1 1 1 1
A) B) C) D) E)
8 6 4 7 3
(d) at least one head
3 5 7 5 1
A) B) C) D) E)
4 6 8 8 8

TEST2
1. P(A)=0.4, P(B)=0.3 and P( A  B)  0.2 . Find P( A  B)

A) 0.2 B) 0.3 C) 0.4 D) 0.5 E) 0.6

2. P(C )  0.7 , P( D)  0.5 and P(C  D)  0.3 . Find P(C  D)

A) 0.4 B) 0.9 C) 0.6 D) 0.8 E) 0.7

3. P( A)  0.24 , P( B)  0.18 and P( A  B)  0.31 . Find


P( A  B)
A) 0.18 B) 0.06 C) 0.14 D) 0.09 E) 0.11

4. If P( A)  0.2 , P( A  B)  0.1 and P( A  B)  0.7 . Find


P(B )
A) 0.2 B) 0.6 C) 0.4 D) 0.3 E) 0.5

5. Find P(C  D) if P(C )  0.37 , P( D)  0.43 and C and D


are mutually exclusive.
A) 0.63 B) 0.7 C) 0.06 D) 0.57 E) 0.8

213
6. 20% of teams in a football league have Italian players and
25% have Brazilian players. 60% have neither Italian nor
Brazilian players. What percentage have both Italian and
Brazilian players?
A) 5% B) 8% C) 12.5% D) 4% E) 9.5%

7. 90% of students in a university have Facebook account, and


three out of five have a Twitter account. One-twentieth of
students have neither a Facebook account nor a twitter
account. What percentage of students are on both Facebook
and Twitter?
A) 45% B) 64% C) 55% D) 38% E) 74%

8. A and B are two events such that P( A)  0.3 , P( B)  0.7


and P( A | B)  0.4 . Find.
(a) P( B | A)
14 3 11 18 7
A) B) C) D) E)
15 7 14 23 8
(b) P( A | B )
A) 0.18 B) 0.25 C)0.32 D) 0.28 E) 0.15

(c) P( A  B)
A) 0.24 B) 0.42 C) 0.35 D)0.21 E) 0.14
1 1
9. The events A and B are such that P( A)  , P( B)  and
3 4
1
P( A  B)  . Find P( A | B )
2
1 5 1 7 1
A) B) C) D) E)
3 12 6 18 12

10. Event A and B are mutually exclusive and P( A)  0.2 ,


P( B)  0.4 . Find
214
(a) P( A  B)
A) 0.8 B)0.4 C) 0.6 D) 0.7 E) 0.55

(b) P( A  B )
A) 0.4 B)0.6 C) 0.1 D) 0.3 E) 0.2

(c) P( A  B )
A) 0.2 B)0.4 C) 0.5 D) 0.6 E) 0.8

Answers
Test 1
1. (a) B 2. (a) C 3.(a) D 4.E 5.(a) C
(b) D (b) B (b) A (b) B
(c) E (c) A (c) C (c) A
(d) A (d) E (d) E (d) C
(e) A (e) C (e) B
Test 2
1.D 2.B 3.E 4.C 5.E
6.A 7.c 8(a) A,(b)D,(c) B 9A 10.(a), (b)E,(c)B

215
Lecture 14
Bernoulli’s Formula. The Most Probable Number of
Occurrences of an Event
If n independent trials are performed in each of which the
probability of occurrence of the event A is the same and is
equal to p, then the probability of the event A occurring m
times in these n trials is expressed by Bernoulli’s formula:
Pm,n  Cnm p m q n  m
where q  1  p . Thus we have
Po,n  Cno p o q n o  q n ,
P1,n  Cn1 p1q n 1  npq n 1 ,
n(n  1) 2 n  2
P2,n  Cn2 p 2 q n  2  p q ,
1 2
................................................
Pn, n  p n
Definition. The number m0 is called the most probable
number of occurrences of the event A in n trials if at m  m0
the value of Pm,n is not less than all the other values of Pm,n ,
i.e.
Pm0 ,n  Pmi ,n at mi  m0 .
If p  0 and p  1 , the number m0 can be determined
from the double inequality
np  q  m0  np  p .
The difference between the end point values in this
double inequality is 1:
(np  p)  (np  q)  p  q  1 .
If np  p is not an integer, then the double inequality
specifies only one most probable value m0 . Now if np  p is an
integer, then there are two most probable values,
216
m0/  np  q and m0//  np  p
Theorem. The probability that the event will occur (in n
independent trials):
a) less than k times (  k ) is found by the formula
Po,n  P1,n  ...  Pk 1,n
b) more than k times (  k ) is found by the formula
Pk 1,n  Pk  2,n  ... Pn,n
c) no less than k times (  k ) is found by the formula
Pk ,n  Pk 1,n  Pk  2,n  ... Pn,n
d) no more than k times (  k )is found by the formula
Po,n  P1,n  ...  Pk 1,n  Pk ,n .
Let us consider some examples.
Example 1. The probability of occurrence of the event A
is 0.4. What is the probability of the event A occurring not
more than three times in 10 trials?
Solution. Here p=0.4, q=0.6. We have:
probability of occurrence of event A 0 times is
P0,10  q10 ;
probability of occurrence of event A 1 time is
P1,10  10 pq 9 ;
probability of occurrence of event A 2 times is
P2,10  45 p 2 q 8 ;
probability of occurrence of event A 3 times is
P3,10  120 p 3 q 7 .
The probability of the event A occurring not more than
three times is equal to
P  P0,10  P1,10  P2,10  P3,10
that is
P  q10  10 pq9  45 p 2q8  120 p3q7

217
or
P  q7 (q3  10 pq2  45 p 2q  120 p3 )
Putting p=0.4, q=0.6, we obtain
P  0.67 (0.216  1.44  4.32  7.68)  0.38 .
Example 2. Determine the probability of the fact that in a
family having five children there are three girls and two boys.
The probabilities of a girl and a boy being born are assumed to
be equal.
Solution. The probability of a girl being born is p=0.5,
then q  1  p  0.5 (the probability of a boy being born).
Therefore, the sought-for probability is
5! 5
P3,5  C53 p 3 q 2  (0.5)3 (0.5) 2  .
3!2! 16

Example 3. Under the conditions of the previous


problem find the probability of the fact that there will not be
more than three girls among the children born to the family.
Solution. We have
5 5
1 1 1 5
P0,5     ; P1,5  5     ;
 2  32  2  32

5 5
1 5 1 5
P2,5  10     ; P3,5  10     .
 2  16  2  16
13
P  P0,5  P1,5  P2,5  P3,5  .
16
Example 4. Two chess-players of the same level play
chess. What is more probable to win: two games out of four or
three games out of six?
Solution. As players of the same level, then the
probability of winning is equal to 0.5; then the probability of
loss is 0.5. Let us find the probability of the fact that two
218
games out of four will be won. Applying the Bernoulli’s
formula we get
6
P2, 4  C42 p 2 q 2  .
16
Now we find the probability of the event that three games
out of six will be won. We have
5
P3, 6  C63 p 3 q 3  .
16
Since P2, 4  P3,6 , then of four games to win two is more
likely than three games out of six.
Example 5. We toss a coin five times. Find the
probability that the Heads comes up
(a) less than two times.
(b) no less than two times.
Solution.
1 5 3
(a) P0,5  P1,5   
32 32 16
13
(b) P2,5  P3,5  P4,5  P5,5  1  ( P0,5  P1,5 ) 
16

Example 6. We toss a coin eight times. Find the


probability that the Heads comes up no more than three times.
Solution. We have
P  P0,8  P1,8  P2,8  P3,8 
8
1
 
   C80  C81  C82  C83 
2
8
1 93
   (1  8  28  56)  .
2 256
Example 7. There are 10 white and 40 red balls in the
box. We successively draw 14 balls, register the colour of each

219
drawn ball and replace it into the box. Determine the most
probable number of occurrences of a white ball.
10 1 4
Solution. Here n  14, p   , q  1 p  .
50 5 5
Using the double inequality
np  q  m0  np  p
at the values of n, p and q indicated, we obtain
14 4 14 1
  m0   ,
5 5 5 5
i.e. 2  m0  3 .
Thus, the problem admits of two solutions:
m0/  2, m0//  3
Example 8. The probability of a marksman hitting the
target is 0.8. He fires 20 shots. Determine the most probable
number of times he hits the target.
Solution. Here n  20, p  0.8, q  0.2 . It follows that
20  0,8  0,2  m0  20  0,8  0,8
i.e 15.8  m0  16.8 .
Since m is an integer, we have m0  16 .
Example 9. As a result of long observations it was
established that the probability of rain falling on October 14 in
3
Baku is . Determine the most probable number of rainy days
8
on October 14 in Baku for 10 years.
3 5
Solution. Here n  10, p  , q  .
8 8
Thus we have
3 5 3 3
10    m0  10  
8 8 8 8
1 1
or 3  m0  4 , i.e. m0  4 .
8 8
220
Example 10. A balloon manufacturer claims that 95% of
his balloons will not burst when blown up. If you have 20 of
these balloons to blow up for a birthday party what is the
probability that none of them burst when blown up?
Solution. To solve this problem we use the Bernoulli’s
formula. Here m  n  20, p  0.95, q  0.05 . We have
P20 , 20  C2020 p 20 q 0  0.9520  0.3584...  0.358 .

Total Probability Formula


Definition. Let the events B1 , B2 ,..., Bn be pairwise
mutually exclusive (neither two of them can take place
simultaneously). The n events B1 , B2 ,..., Bn are said to be form
a complete group of events if only one them can occur as a
result of some trial.
Theorem. If the event A is known to occur together with
one of the events B1 , B2 ,..., Bn forming a complete group of
mutually exclusive events, then the probability of the event A
can be found from the formula
P( A)  P( B1 )  P( A | B1 )  P( B2 )  P( A | B2 )  ... 
 P( Bn )  P( A | Bn )
n
or P( A)   P( Bi )  P( A | Bi )
i 1
This formula is known as the total probability formula.
Proof. According to the condition the event A can occur
together with one of the mutually exclusive events
B1 , B2 ,..., Bn . In other words the event A can be represented as a
union of the events
B1 A  B1  A, B2 A  B2  A, ... , Bn A  Bn  A , i.e.
A  B1 A  B2 A  ... Bn A
Using the probability addition rule, we obtain
221
P( A)  P( B1 A  B2 A  ... Bn A) 
 P( B1 A)  P( B2 A)  ... P( Bn A) .
Now, using the probability multiplication rule, we get

P( A)  P( B1 ) P( A | B1 ) 
 P( B2 ) P( A | B2 )  ...  P( Bn ) P( A | Bn ) .

The theorem has been proved.


Let us now assume that the event A can occur together
with one of the events H1 , H 2 ,..., H n forming a complete group
of mutually exclusive events. Since we do not know which one
of these events occurs they are called hypotheses.
Suppose the event A has occurred. Provided the event A
has occurred let us find the conditional probability of the event
H 1 , i.e. P( H1 | A) .
By the probability multiplication rule
P( AH 1 )  P( A)  P( H 1 | A)  P( H 1 )  P( A | H 1 )
whence
P( AH1 ) P( H1 )  P( A | H1 )
P( H1 | A)  
P( A) P( A)
Replacing P(A) by the total probability formula we
obtain
P( H1 )  P( A | H1 )
P( H1 | A)  .
P( H1 )  P( A | H1 )  ...  P( H n )  P( A | H n )
Similarly we can obtain the formulas for the conditional
probability of the other hypotheses.
Thus, provided the event A has occurred, the conditional
probability of the event H i (i  1,2,..., n) can be determined by
the formula

222
P( AH i ) P( H i )  P( A | H i )
P( H i | A)   n


P( A)
P( H i )  P( A | H i )
i 1
which is known as Bayes’ formula.
The probabilities P( H i | A) calculated by Bayes’ formula
are often called probabilities of hypotheses.
Example 1. We have four boxes. The first box contains 2
red and 8 yellow balls, the second, 3 red and 2 yellow balls, the
third,1 red and 1 yellow ball, and the fourth, 4 red and 6 yellow
balls. The event H i is the selection of the i-th box (i=1,2,3,4).
i
The probability of selecting of the i-th box is , i.e.
10
1 2 3 4
P( H1 )  , P( H 2 )  , P( H 3 )  , P( H 4 )  .
10 10 10 10
We randomly select one of the boxes and draw a ball
from it. Calculate the probability of the ball being red.
Solution. Suppose the event A is the drawing of a red
ball. If follows from the hypothesis that the conditional
probability of drawing a red ball from the first box is equal to
1
, i.e.
5
2 1
P( A | H1 )   ;
10 5
3 1 2
similarly, P( A | H 2 )  , P( A | H 3 )  , P( A | H 4 )  .
5 2 5
The probability of drawing a red ball can be found by the
formula of total probability:
P( A)  P( H1 )  P( A | H1 )  P( H 2 )  P( A | H 2 ) 
 P( H 3 )  P( A | H 3 )  P( H 4 )  P( A | H 4 ) 
1 1 2 3 3 1 4 2
         0.45 .
10 5 10 5 10 2 10 5
223
Example 2. There are three urns identical in appearance.
The first urn contains 24 green balls, the second urn contains
12 green and 12 red balls, and the third urn contains 24 red
balls. We draw a green ball from a randomly selected urn. Find
the probability of the ball being drawn from the first urn.
Solution. Assume that H1 , H 2 , H 3 are the hypotheses
consisting in selecting the first, the second and the third urn
respectively. Let A be the event of drawing a green ball. Since
the selection of any of the urns is equally likely then
1
P( H1 )  P( H 2 )  P( H 3 )  .
3
It is clear that the probability of drawing a green ball
from the first urn is P( A | H1 )  1 ; the probability of drawing a
12 1
green ball from the second urn is P( A | H 2 )   ; the
24 2
probability of drawing a green ball from the third urn is
P( A | H 3 )  0 .
Therefore, the desired probability P( H1 | A) (i.e. the
probability of the green ball being drawn from the first urn) can
found from Bayes’ formula
P ( H1 )  P ( A | H1 )
P( H1 | A)  
P ( H1 )  P ( A | H1 )  P ( H 2 )  P ( A | H 2 )  P ( H 3 )  P ( A | H 3 )
1
1
3 2
  .
1 1 1 1
1     0 3
3 3 2 3

Example 3. There are two boxes. The first box contains


8 blue and 7 red balls, the second, 4 blue and 6 red balls. We
draw one ball from the second box and place it into the first
one, then we randomly draw one ball from the first box.
What is the probability of the drawn ball being blue?
224
Solution. Let A be the event of drawing a blue ball. After
a ball was drawn from the second box and placed into the first,
two collections of balls have turned out in the first box:
(1) 8 blue and 7 red balls it contained prior to
replacement;
(2) one ball replaced from the second box.
The probability of appearance of a blue ball belonging to the
8
first collection is P( A | H1 )  , and from the second
15
4 2
collection, P( A | H 2 )   .
10 5
The probability that he randomly drawn ball belongs to the first
15 1
collection is P( H1 )  , and to the second, P( H 2 )  .
16 16
Using the total probability formula, we obtain
P( A)  P( H1 )  P( A | H1 )  P( H 2 )  P( A | H 2 ) 
15 8 1 2 21
      0.525 .
16 15 16 5 40
Example 4. The hospital receives an average of 50% of
patients with the disease F, 30% of patients with the disease L,
and 20% of patients with the disease C. The probability that the
patient will completely recovered from the illness F is equal to
0.7; for the illnesses L and C, these probabilities are equal to
0.8 and 0.9, respectively. The patient admitted to the hospital
was discharged healthy. Find the probability that the patient
was suffering from the disease F.
Solution. If follows from the hypothesis that
P( H1 )  0.5, P( H 2 )  0.3, P( H 3 )  0.2
where P( H1 ) is the probability of the fact that the patient in the
hospital is suffering from the disease F. The probabilities
P ( H 2 ) and P( H 3 ) are defined similarly. Let A be the event that
the patient admitted to the hospital was discharged healthy.
225
It is clear that
P( A | H1 )  0.7, P( A | H 2 )  0.8, P( A | H 3 )  0.9 .
Now using the total probability formula, we obtain
P( A)  P( H 1 )  P( A | H 1 )  P( H 2 )  P( A | H 2 ) 
 P( H 3 )  P( A | H 3 )  0.5  0.7  0.3  0.8  0.2  0.9 
 0.35  0.24  0.18  0.77
Therefore, the desired probability P( H 1 | A) (i.e. the
probability of the fact that the patient that had been discharged
was suffering from the disease F) can be found from the Bayes
formula
P( H1 )  P( A | H1 ) 0.35 5
P( H1 | A)    .
P( A) 0.77 7

TESTS
1. We toss a coin eight times. What is the probability that we
will have heads six times?

3 1 7 1 5
A) B) C) D) E)
32 8 64 16 36

2. We toss a coin six times. What is the probability that we will


have heads no more than three times?

21 5 7 15 45
A) B) C) D) E)
32 8 16 32 64
3. We toss a coin seven times. What is the probability that we
will have heads no less than five times?

9 17 5 29 15
A) B) C) D) E)
64 128 32 128 64

226
4. The probability of a marksman hitting the target is 0.7. He
fires 25 shots. Determine the most probable number of times he
hits the target.

A) 14 B) 15 C) 16 D) 17 E) 18

5. A balloon manufacturer claims that 90% of his balloons will


not burst when blown up. If you have 30 of these balloons to
blow up for a birthday party what is the probability that exactly
2 balloons burst?
A) 0.167 B) 0.228 C) 0.341 D) 0.284 E) 0.066

6. The hospital receives an average of 45% of patients with the


disease K, 35% of patients with the disease D, and 20% of
patients with the disease C. The probability that the patient will
completely recovered from the illness K is equal to 0.8; for the
illnesses D and C, these probabilities are equal to 0.6 and 0.4,
respectively. The patient admitted to the hospital was
discharged healthy. Determine the probability that the patient
was suffering from the disease C.
8
A) 0.74 B) 0.58 C) 0.87 D) E) 0.46
65

7. A box contains 2 balls. We place one red ball into the box
and then we randomly draw one ball from it. Find the
probability that the drawn ball being red if all possible
hypotheses about the initial balls in the box are equally likely.
2 1 3 1 5
A) B) C) D) E)
3 2 4 3 6

8. There are three boxes. The first box contains 5 green and 3
red balls, the second, 4 green and 6 red balls, and the third, 3
green and 7 red balls. The event Hi is the selection of the i-th
box (i=1,2,3). The probabilities of selecting the boxes are given
227
2 3 5
as follows: P( H1 )  , P( H 2 )  , P( H 3 )  . We
10 10 10
randomly select one of the box and draw a ball from it.
Calculate the probability of the ball being green.
A) 0.281 B) 0.415 C) 0.395 D) 0.243 E) 0.477

9. The first box contains 1 pink and 2 blue balls, the second,
100 pink and 100 blue balls. One ball was drawn from the
second box and placed into the first, and then one ball was
randomly drawn from the first box. Calculate the probability
that the drawn ball had previously belonged to the second box
if it is known to be pink.
1 2 1 3 5
A) B) C) D) E)
3 5 4 5 8

10.There are two urns. The first urn contains 3 red and 2 white
balls, the second, 24 red and 26 white balls. One ball was
drawn from the second urn and placed into the first, and then
one ball was randomly drawn from the first urn. Determine the
probability that drawn ball had previously belonged to the first
urn if it is known to be red.
15 25 24 18 17
A) B) C) D) E)
28 29 31 27 21
Answers
Test
1.C 2.A 3.D 4.E 5.B
6.D 7.A 8.C 9.A 10.B

228
Lecture 15
Random Variable and the Law of Its Distribution

Mathematics has many applications in most areas of


modern day life. If you construct a mathematical model of a
real world situation, you can learn about the real situation by
analyzing the mathematical model.
A mathematical model is a simplification of a real
world situation. It can be used to make predictions about a real
world problem. By analyzing the model an improved
understanding of the situation may be obtained.
Statistics play an important role in this process. In
statistics you collect observations or measurements of some
variable. Such observations are known as data.
Definition. A random variable is a variable quantity
which randomly assumes a certain numerical value resulting
from the outcome of a trial. This value depends on chance and,
generally speaking, varies as the trials are repeated. Examples
of random variables are the number of girls in a family, the
number of students attending a lecture, the duration of life of a
person and so on.
Definition. A variable is said to be discrete if it can take
only specific values in a given range.
Definition. A variable is said to be continuous if it can
take any value in a given range.
Example 1. State whether or not each of the following
variables is discrete or continuous.
(a) Number of people on a bus
(b) Time required to run 100m
(c) Number of boys in a family
(d) Weigh of watermelon
(e) The number of apples on the trees in an orchard
(f) Shoe size
(g) Length
229
(h) Time waiting in a queue.
It is clear that (a), (c), (e) and (f) are discrete variables
since these variables can’t take any value from a given range.
On the other hand, (b), (d), (g) and (h) are continuous variables
because these variables can assume any value in a given
interval.
We can define these concepts more precisely in terms of
mathematics.
A random variable is a quantity whose value depends on
chance. A random variable can assume one or another value
from a certain number set, but it is not known beforehand
which value it will assume. A random variable is designated by
capital letters X, Y,…, and the values that the random variable
can take are represented by the corresponding lower-case
letters x,y,…
Definition. If the values that the given random variable X
can assume form a discrete (finite or infinite) number series
x1, x2 ,..., xn ,..., then the random variable is called discrete.
Definition. If the values that the given random variable X
can assume fill up the whole finite or infinite interval (a,b) then
the random variable is said to be continuous.
To each value xn of a random variable X of a discrete
type there corresponds a definite probability pn. To specify a
discrete random variable completely, you need to know its set
of possible values and the probability with which it takes each
one; this information is best displayed in a table.

X x1 x2 … xn
P( X  x) … (1)
p1 p2 pn

Definition. The relationship establishing in one way or


another the connection between the possible values of a

230
random variable and their probabilities is called the Law of
distribution of a random variable.
The law of distribution of a random variable you can
specify in a table, analytically (i.e. by means of formulas) and
graphically.
Definition. A table representing the correspondence
between the possible values of variable and their probabilities
is called the Law of distribution of a discrete random
variable.
The table (1) is called the Law of distribution or the
probability distribution of a discrete random variable. The
total of all the probabilities in a probability distribution must
always equal 1. In this case p1  p2  ...  pn  1. This fact can
be useful if we do not have complete information about the
probabilities.
Example 2. A fair six-sided die is rolled. Write down the
probability distribution of the outcome of rolling a fair die.
Solution.

X 1 2 3 4 5 6
P( X  x) 1 1 1 1 1 1
6 6 6 6 6 6
Here the sum of all the probabilities is equal to 1.
6
1 1 1 1 1 1

i 1
Pi        1
6 6 6 6 6 6
Example 3. Three fair coins are tossed. The number of
heads X, is counted. Write down the probability distribution of
getting heads.
Solution. In this experiment we have eight possible
outcomes HHH, HHT, HTH, HTT, THH, THT, TTH, TTT.

231
Consequently,
Number of Heads (X) 0 1 2 3
P( X  x) 1 3 3 1
8 8 8 8
Here the sum of all the probabilities is equal to 1.
4
1 3 3 1

i 1
Pi      1
8 8 8 8
Example 4. A discrete random variable X has the
following probability distribution:
X 1 2 3 4
P( X  x) 1 k 1 3
5 4 10
Find the value of k.
Solution. For a discrete random variable the sum of all
the probabilities must always equal 1.
1 1 3 1
 k   1 k 
5 4 10 4
Example 5. Given P( X  x)  kx for x  1,2,3,4. Find k.
Solution. According to the condition we can make a
table to show the probability distribution

X 1 2 3 4
P( X  x) k 2k 3k 4k

1
We have k  2k  3k  4k  1  10k  1  k  .
10
Let us now consider a continuous random variable.
To each interval (a,b) belonging to the range of a random
variable of the continuous type there corresponds a definite
probability P(a  X  b) that the value assumed by the random
variable will fall in that interval.

232
It is convenient to represent the law of distribution of a
continuous random variable with the aid of the so-called
probability density function f (x) . The probability
P(a  X  b) of the fact that the value assumed by the random
variable X will fall in the interval (a,b) is defined by the
equality
b
P(a  X  b)   f ( x)dx .
a
Definition. The graph of the probability density function
f (x) is called a distribution curve.
In terms of geometry, the probability that the random
variable will fall in the interval (a,b) is equal to the area of the
corresponding curvilinear trapezoid bounded by the
distribution curve, the Ox axis and the straight lines x=a, x=b.

f (x)

o a b x
The probability density function f (x) possesses the
following properties:
1. f ( x)  0 (i.e f (x) is non-negative)

2.  f ( x)dx  1.

Moreover, if all the values of the random variable X
belong to the interval (a,b), the last property can be written as
b

 f ( x)dx  1 .
a

233
Probability Distribution Function

We have shown above that a continuous variable X can


be represented by using the probability density function f (x) .
However, this method is not the only one. The law of
distribution of a continuous random variable can be also
represented with the aid of the so-called probability
distribution function.
Definition. Let x be an arbitrary real number.
Consider the function
F ( x)  P( X  x)
This function F (x) is called the probability distribution
function of the random variable X.
The function F (x) defines the probability of the fact that
the random variable X will assume a value less than x.
The function F (x) exists both for discrete and for
continuous random variables. If f (x) is the probability density
function of the continuous random variable X, then

F ( x)   f (t )dt .


It follows from the last equality that f ( x)  F / ( x) .


The function f (x) is sometimes called a probability
distribution differential function, and the function F (x) , a
probability distribution integral function.
It should be noted some properties of a probability
distribution function F (x) .

Properties

1. All the values assumed by F (x) belong to the interval


[0,1]:

234
0  F ( x)  1 .
Proof. It follows from the definition of F (x) as a
probability. The probability is always non-negative number
which is not greater than 1.
2. F (x) is a non-decreasing function, F (x)  , i.e.
x2  x1  F ( x2 )  F ( x1 )
Proof. Let x2  x1 . The event, consisting in the fact that
X will assume a value less than x 2 , may be subdivided into two
following inconsistent events:
E1: X will assume a value less than x1 with the probability
P( X  x1 )
E2: X will assume a value satisfying the inequality
x1  X  x2 with the probability P( x1  X  x2 ) .
By the probability addition rule

P( X  x2 )  P( E1  E2 )  P( X  x1 )  P( x1  X  x2 )
whence
P( X  x2 )  P( X  x1 )  P( x1  X  x2 )
or
F ( x2 )  F ( x1 )  P( x1  X  x2 )  0 
F ( x2 )  F ( x1 )
and we get what had to prove.
3.The probability of the fact that a random variable X will
assume a value belonging to an interval (a,b) is equal to an
increment of the function F (x) on that interval:
P(a  X  b)  F (b)  F (a) . (*)
Proof. From the previous property we have
F ( x2 )  F ( x1 )  P( x1  X  x2 ) .
Putting x2  b and x1  a we obtain (*) .

235
4. The probability of the fact than the continuous random
variable X will assume a certain one value is equal to zero:
P ( X  x1 )  0 .
Proof. We have
P(a  X  b)  F (b)  F (a)
Putting a  x1 and b  x1  x we get
P( x1  X  x1  x)  F ( x1  x)  F ( x1 ) .
Let x  0 . Since X is continuous random variable,
F (x) is continuous. Consequently
x  0  F ( x1  x)  F ( x1 )  0  P( X  x1 )  0.
The property has been proved.
Note that, using this property, it is easy to prove the
following equalities:
P ( a  X  b)  P ( a  X  b) 
 P ( a  X  b)  P ( a  X  b )
For example, the equality
P ( a  X  b)  P ( a  X  b)
is proved as follows:
P ( a  X  b)  P ( a  X  b)  P ( X  b)  P ( a  X  b) .
5. if all the possible values of a random variable belong
to (a,b), then:
(1) F ( x)  0 for x  a
(2) F ( x)  1 for x  b .
Proof. (1) Let x1  a (Fig. 1).
Then the event x1 a b
X  x1 is impossible and
Fig.1
hence P ( X  x1 )  0
(2) Let x2  b (Fig.2).
a b x2
In this case the event
X  x2 is a certain one and Fig.2

236
therefore P( X  x2 )  1 .
6. If all the possible values of a continuous random
variable lie on the whole x-axis, then
lim F ( x)  0; lim F ( x)  1.
x  x 

Expectation of a Discrete Random Variable

Definition. The expectation (or mean value) of a


discrete random variable is the sum of the products of the
values of the random variables by the probabilities of these
values.
The expectation (or mean value) of a random variable X
is written E(X). The expectation is also known as mathematical
expectation, average, mean value, or mean. If the random
variable X has the following probability distribution

x x1 x2 … xn
P( X  x) p1 p2 … pn

then the expectation can be calculated as


E( X )  x1 p1  x2 p2  ...  xn pn
or
n
E ( X )   xi pi
i 1
Example 1. A discrete random variable X has probability
distribution

x 1 2 3 4
P( X  x) 0.4 0.3 0.2 0.1

Calculate E(X).

237
Solution. According to the definition we have to
calculate the product xP( X  x) for all possible values of x and
add up the results:
E ( x)  1  0.4  2  0.3  3  0.2  4  0.1  2 .
Example 2. A random variable X has probability
distribution P( X  x)  k ( x  1) for x  2, 3, 4, 5, 6. Find E(X).
Solution. According to condition we have the following
probability distribution

x 2 3 4 5 6
P( X  x) 3k 4k 5k 6k 7k

First we find k. Since for a discrete random variable the


sum of all the probabilities must add up to one we get
1
25k  1 or k   0,04
25
It follows that the probability distribution will be:

x 2 3 4 5 6
P( X  x) 0.12 0.16 0.2 0.24 0.28

Now we can easily find E(X):


E ( x)  2  0.12  3  0.16  4  0.2  5  0.24  6  0.28  4.4
Example 3. A random variable X has the following
probability distribution

x 1 2 3 4 5
P( X  x) 0.1 a b 0.2 0.1

Given E(X)=2.9, find the value of a and the value of b.


Solution. By definition
E ( x)  0.1  2a  3b  0.8  0.5  2.9

238
whence
2a  3b  1.5
On the other hand, the sum of all the probabilities must
be equal to one:
0.1  a  b  0.2  0.1  1  a  b  0.6
To determine a and b we have the following system of
two equations with two unknowns:
2a  3b  1.5

 a  b  0. 6
Solving this system, we find a=0.3 and b=0.3.
Note the following:
1. The expectation of a random variable, intuitively, is the
average value of repetitions of the experiment it
represents.
2. The expected value (expectation) of a random variable is
greater than the least value and less than the highest
possible value that the variable can take.
3. The expectation of a random variable need not be one of
the values that the variable can take.

Properties of Expectation
Note some important properties of expectation.
1. The expectation of a constant variable is equal to the
constant itself:
E (C )  C
Proof. We shall consider a constant C as a discrete
random variable which can assume only one possible value C
with the probability p=1. Hence,
E (C )  C  1  C
So, if C is a constant, then E (C )  C .
2. A constant factor can be taken out of the sign of the
expectation:
E (CX )  CE ( X )
239
Proof. Suppose the probability distribution of the
discrete random variable X is:

X x1 x2 … xn
P( X  x) p1 p2 … pn

Then the Law of distribution of the random variable CX


can be written as

CX Cx1 Cx 2 … Cxn
P p1 p2 … pn

The expectation of the random variable CX is calculated


as follows:
E (CX )  Cx1 p1  Cx 2 p2  ...  Cx n pn 
 C ( x1 p1  x2 p2  ...  xn pn )  CE ( X )
Thus,
E (CX )  CE ( X ) .
3. If X and Y are two independent random variables, then
E ( XY )  E ( X )  E (Y ) .
Proof. Suppose the independent random variables X and
Y are given by their probability distributions. To simplify the
calculations we consider a small number of possible values.

X x1 x2 X y1 y2
P( X  x) p1 p2 P(Y  y ) q1 q2

Then the law of distribution of the random variable XY


can be written as

240
XY x1 y1 x2 y1 x1 y 2 x2 y 2
P p1q1 p 2 q1 p1q 2 p2 q2

The expected value of the random variable XY can be


written as:
E ( XY )  x1 y1  p1q1  x2 y1  p2 q1  x1 y2  p1q2  x2 y2  p2 q2 
 y1q1 ( x1 p1  x2 p2 )  y2 q2 ( x1 p1  x2 p2 ) 
 ( x1 p1  x2 p2 )  ( y1q1  y2 q2 )  E ( X )  E (Y )

Thus,
E ( XY )  E ( X )  E (Y ) .
4. If X and Y are two random variables, then
E ( X  Y )  E ( X )  E (Y )
Example 1. Two independent random variables X and Y
are given by the following probability distributions

X 3 5 8 Y 4 7
P( X  x) 0.1 0.3 0.6 P(Y  y ) 0.2 0.8

Find E ( XY ) .
Solution. We find the expectations of each of the given
variables
E ( X )  3  0.1  5  0.3  8  0.6  6.6
E (Y )  4  0.2  7  0.8  6.4 .

Since the random variables X and Y are independent the


expectation E ( XY ) can be calculated as
E ( XY )  E ( X )  E (Y )  6.6  6.4  42.24 .

241
Example 2. Two fair six-sided dice numbered 1 to 6 are
thrown once. The random variables X and Y represent the
values on the upper face of each of the dice. Find E ( X  Y ) .
Solution. We have
X 1 2 3 4 5 6
P( X  x) 1 1 1 1 1 1
6 6 6 6 6 6

Y 1 2 3 4 5 6
P(Y  y ) 1 1 1 1 1 1
6 6 6 6 6 6

Now we find E ( X ) and E (Y ) .


1 1 1 1 1 1 7
E( X )  1  2   3   4   5   6  
6 6 6 6 6 6 2
7
It is clear that E (Y )  . Therefore
2
7 7
E ( X  Y )  E ( X  E (Y )    7 .
2 2

Variance of a Discrete Random Variable

Suppose two discrete random variables X and Y are given


by the following probability distributions

X -0.02 0.03 Y -700 300


P( X  x) 0.6 0.4 P(Y  y ) 0.3 0.7

Here we find the expectations of these variables:


E ( X )  0.02  0.6  0.03  0,4  0
E (Y )  700  0.3  300  0.7  0

242
The mathematical expectations of these variables are the
same and the possible values are different. Moreover, X can
take possible values close to the expectation, and Y can take
possible values far from the expected value.
This example shows that knowing only the mathematical
expectation of a random variable, it is impossible to judge what
the possible values it can take and how these values are spread
around the expectation. Therefore, along with the mathematical
expectation we use another characteristic that helps to estimate
how the possible values of a random variable are spread around
the expected value. This new characteristic is called the
variance of a random variable.
Let X be a random variable and E(X) its mathematical
expectation.
Definition. The difference X  E ( X ) between the
random variable X and its expectation E(X) is called the
deviation of the random variable.
Suppose the probability distribution of the random
variable X is:

X x1 x2 … xn
P( X  x) p1 p2 … pn

Then the deviation X  E( X ) has the following


probability distribution

X  E (X ) x1  E ( x) x2  E ( X ) … xn  E (X )
P p1 p2 … pn

Theorem. The expectation of the deviation is equal to


zero:
E ( X  E ( X ))  0

243
Proof. Using the properties of the mathematical
expectation and taking into account that E ( X ) - constant, we
obtain
E ( X  E ( X ))  E ( X )  E ( E ( X ))  E ( X )  E ( X )  0 .
Definition. The variance of a random variable is the
expectation of the square of the deviation of the random
variable its expectation.
The variance of X is usually written as Var (X) and by
definition
Var ( X )  E[ X  E ( X )]2 .
The variance of a random variable is spread of its values about
its expectation.
Theorem. The variance is found by the formula
Var ( X )  E ( X 2 )  ( E ( X )) 2 .
Proof. The expectation E ( X ) is constant and hence,
2 E ( X ) and E 2 ( X ) are also constant. Taking it into account and
using the properties of the mathematical expectation, we obtain


Var ( X )  EX  E ( X )  E X 2  2 XE ( X )  E 2 ( X ) 
2

 E ( X 2 )  2 E ( X ) E ( X )  E ( E 2 ( X )) 
 E ( X 2 )  2 E 2 ( X )  E 2 ( X )  E ( X 2 )  ( E ( X ))2 .

The theorem has been proved.

Properties of Variance

1. If C is a constant, then Var(C )  0 .


Proof. By definition
 
Var (C )  E C  E C 2  E (C  C ) 2   E (0)  0
So, Var (C )  0
2. If C is a constant, then
244
Var (CX )  C 2  Var ( X ) .
Proof. We have

  
Var(CX )  E (CX  E (CX ) ) 2  E C 2 ( X  E ( X ))2  
 C E ( X  E ( X ))   C
2 2 2
 Var( X ) .
3. If X and Y are independent random variables, then
Var ( X  Y )  Var( X )  Var(Y ) .
Proof. According to the formula for calculating the
variance we obtain
 
Var ( X  Y )  E ( X  Y )2  E( X  Y ) 2
Removing the brackets and using the properties of the
expectation we get

 
Var ( X  Y )  E X 2  2 XY  Y 2  E( X )  E (Y ) 
2

 E ( X 2 )  2 E ( X ) E (Y )  E (Y 2 )  E 2 ( X )  2 E ( X ) E (Y )  E 2 (Y ) 
 E ( X 2 )  E 2 ( X )  E (Y 2 )  E 2 (Y )  Var( X )  Var(Y ) .
4. If X and Y are independent random variables, then
Var ( X  Y )  Var( X )  Var(Y ) .
Proof. According to the property 3 we get
Var ( X  Y )  Var( X )  Var(Y ) .
Now using the property 2 we obtain
Var ( X  Y )  Var( X )  (1) 2  Var(Y )
or
Var ( X  Y )  Var( X )  Var(Y ) .

Example. A fair six-sided die is thrown. What is


Var ( X ) if X is outcome of one fair die?
Solution. We have
1 1 1 1 1 1 7
E( X )  1   2   3   4   5   6  
6 6 6 6 6 6 2

245
1 1 1 1 1 1 91
E( X 2 )  12   2 2   32   4 2   5 2   6 2  
6 6 6 6 6 6 6
2
91  7  35
Var ( X )  E( X )  E ( X ) 
2 2
   .
6 2 12

Expectation and Variance


of a Continuous Random Variable

The concepts of the expectation and the variance can be


extended to a continuous random variable. Suppose f (x) is the
probability density function of the random variable X.
Definition. The expectation of the continuous random
variable X is specified by the equality

E( X )   xf ( x)dx

(provided the value of the integral is finite).
Definition. The variance of the continuous random
variable X is specified by the formula

Var( X )   x  E ( X )
2
f ( x)dx .


Standard Deviation
Definition. The quantity   Var( X ) is called the
standard deviation of the random variable X.
The standard deviation is a measure of the spread of
values (scores) within of data. The standard deviation is a
measure that is used to quantify how the values of a set of data
are spread. This measures the average distance of data items
from the expectation.

246
Example. A discrete random variable X has the
following law of distribution:

X -5 2 3 4
P 0.4 0.3 0.1 0.2

Find Var(X) and  .


Solution. The variance can be calculated by the formula
Var( X )  E ( X 2 )  E ( X ) .
2

To this end first we find the expectation


E( X )  5  0.4  2  0.3  3  0.1  4  0.2  0.3 .
The discrete random variable X 2 has probability
distribution

X2 25 4 9 16
P 0.4 0.3 0.1 0.2

We find E ( X 2 )
E( X 2 )  25  0.4  4  0.3  9  0.1  16  0.2  15,3 .
Now we find the variance:
Var( X )  E ( X 2 )  E ( X )  15.3  (0.3)2  15.21 .
2

Therefore
  Var( X )  15.21  3.9 .

Uniform Distribution

The distribution of random variables whose all values lie


in an interval [a,b] and possess a constant probability density
on that interval is known as uniform distribution.
Let us find the probability density function f (x) of the
uniform distribution. It is known that all the possible values of
the random variable lie in the interval [a,b] on which the
247
function f (x) has constant value C. In other words, the random
variable X does not assumes the values out of the interval
[a,b].
Thus,
0, if x  a

f ( x)  C , if a  x  b
0, if x  b .

f(x)

o a x
b

Let us now find the constant C. Since all the possible


values of the random variable X belong to the interval [a,b],
the following equality holds true
b b

 f ( x)dx  1
a
or  Cdx  1 .
a

1
Since C (b  a)  1, we have C  and, hence
ba
 0, if x  a
 1

f ( x)   , if a  x  b
b  a
 0, if x  b .
Now we can easily find the mathematical expectation, the
variance and the standard deviation for a random variable with
uniform distribution. We have

248
1 b2  a2 a  b
b b
x
E ( X )   xf ( x)dx   dx    .
a a
ba ba 2 2
Thus,
ab
E( X )  ,
2
which should be expected because of the symmetry of
distribution.
To calculate the variance we use the formula
Var( X )  E ( X 2 )  E ( X ) taking into account the value
2

ab
E( X )  that has already been found. Thus, it remains to
2
calculate E ( X 2 ) . We have
b 2  ab  a 2
b b
x2
E ( X 2 )   x 2 f ( x)dx   dx  .
a a
b  a 3
It follows that
b 2  ab  a 2 (a  b) 2 (b  a) 2
Var( X )   
3 4 12
and, hence
ba
  Var( X )  .
2 3

Binomial Distribution

If the probability of occurrence of a random event in each


trial is equal to p, then, as is known, the probability of the event
occurring m times in n trials is specified by Bernoulli’s formula
Pm,n  Cnm p m q nm , where q  1  p . (*)
The distribution of the random variable X which can
assume n  1 values 0,1,2,..., n described by Bernoulli’s formula
is known as a binomial distribution.
249
The law of distribution was called “binomial” because of
the right side of the equality (*) we can consider as the general
term of the Newton’s binomial expansion:
( p  q) n  Cnn p n  Cnn1 p n1q  ...  Cn0 q n .
Thus, the first term p n of the expansion defines the
probability of the event occurring n times in n independent
trials; the second term n p n1q defines the probability of the
event occurring n-1 times;…; the last term q n defines the
probability of the event occurring not once (i.e. never).
The mathematical expectation and the variance for the
binomial distribution of a random variable X are determined
by the following formulas:
E ( X )  np and Var( X )  npq .

Poisson’s Distribution

The distribution of the random variable X which can


assume any integer non-negative values (0,1,2,..., n) described
by the formula

a m a
P( X  m)  e ,
m!
is known as Poisson’s distribution.
The mathematical expectation and the variance for the
Poisson’s distribution of a random variable X are determined
by the formulas:
E ( X )  a and Var( X )  a .

250
TEST
1. A discrete random variable X has probability distribution

X -3 2 4 5
P 0.4 0.3 0.1 0.2

Calculate E ( X ) .
A)0.8 B) -1.2 C)1 D)-0.8 E)1.2
2.The random variable Y has probability distribution

Y 1 2 5 8 k
P 0.2 0.3 0.1 0.1 0.3

and E (Y )  6.3 . Find the value of k.

A)13 B) 10 C)14 D)11 E)12

3. Given the list of the possible values of the discrete random


variable X:

X 4 6 k
P 0.5 0.3 a

Find the value of k and the value of a, if E ( X )  8 .


A) k  21; a  0.2 B) k  8; a  0.1 C) k  11; a  0.2
D) k  22; a  0.1 E) k  14; a  0.5

4. A random variable X has its probability distribution given by


P( X  x)  k ( x  3) , where x is 0,1,2 or 3. Find the exact
value of E ( X ) .

251
24 21 35 28 32
A) B) C) D) E)
18 18 18 18 18

5. The random variable X has probability distribution


X 1 2 3
P p1 p2 p3

Given E ( X )  2.3 and E ( X 2 )  5.9 . Find p1 , p2 and p3 .


A) p1  0.3; p2  0.2; p3  0.5 B) p1  0.2; p2  0.1; p3  0.7
C) p1  0.1; p2  0.4; p3  0.5 D) p1  0.2; p2  0.3; p3  0.5
E) p1  0.5; p2  0.3; p3  0.2

6. Given X and Y are independent random variables. Find the


variance of the random variable Z  2 X  3Y , if
Var( X )  4 and Var(Y )  5 .
A)23 B) 61 C)107 D)51 E)46

7. The random variables X and Y are independent. Given


Var( X )  7 and Var(Y )  8 . Find Var( Z ) , where
Z  4 X  3Y .
A)40 B) 52 C)104 D)128 E)184

8. A fair six-sided die with sides numbered 1,1, 2, 2, 2, 5 is


thrown. Find the variance of the score.

65 77 13 169 403
A) B) C) D) E)
36 24 6 36 36

9. The discrete random variable X has the probability


distribution

252
X -1 0 1 2
P( X  x) 0.2 0.5 0.2 0.1

Find Var( X )
A)0.74 B) 0.6 C)0.64 D)0.76 E)0.82

10. The random variable X has a standard deviation   0,2 .


Find Var(2 X  3).
A)0.8 B) 0.14 C)0.16 D)0.6 E)0.18
Answers
1.A 2.C 3.A 4.E 5.D
6.B 7.E 8.A 9.D 10.C

253
Bibliography

1.Hiroyuki Shima. Tsuneyoshi Nakayama. Higher Mathematics


for Physics and Engineering. Berlin 2010.
2.Michael Corral. Vector Calculus. Schoolcraft college. 2008.
3.Sheldon Axler. Linear Algebra Done Right. New York. 1997.
4. Donald Allen. Lectures on Linear Algebra and Matrices.
Texas A&M University 2003.

254
UDK 51.517(07)

Author: Azimova Gulshen Musa


Lectures in Higher Mathematics For Engineers.
Baku, ASOIU, 2017, 254 pages.

Editor: Head of General and Applied Mathematics


Department of Azerbaijan State Oil and Industry University,
Professor Dr.Sc.A.R Aliev.

Reviewers:
1. Professor of Applied Mathematics Department of Baku State
University Dr.Sc.H.M.Huseynov
2. Associate Professor of General and Applied Mathematics
Department of Azerbaijan State Oil and Industry University,
Ph.D. R.G. Azizova

The present book is based on lectures given by the author over


a number of years to engineering students at Azerbaijan State
Oil and Industry University. Each lecture is preceded by a short
theoretical introduction containing definitions and basic
notions used in that lecture. The most difficult theoretical
notions are developed to achieve their better understanding by
the students.
It should be noted that at the end of each lecture, students are
offered tests to solve. Mathematics develops a common culture
of thinking, teaches people to reason logically. The ability to
think logically, knowledge of mathematics and the correct use
of mathematics supply the person a powerful research method.

ASOIU order № 01-I/33 “14” september


255
256

You might also like